Вы находитесь на странице: 1из 36

I

Hilario Moncado v. People’s Court Officers of certain corporations, from which the documents, papers, things were seized by means of
FACTS: In an ejectment case filed before the justice of the peace court of Guijulngan, Negros Oriental, search warrants, have no cause of action to assail the legality of the contested warrants and of the
after trial in the absence of the defendants, rendered judgment in favor of plaintiff. The Court of First seizures made in pursuance thereof, for the simple reason that said corporations have their respective
Instance then sent notice by registered mail posted on September 8, 1939 the notice of receipt of personalities, separate and distinct from the personality of herein petitioners, regardless of the amount of
appealed case and the period for pleading. Defendants were informed of such mail by the postmaster but shares of stock or of the interest of each of them in said corporations, and whatever the offices they hold
the same was not claimed, CFI of Negros Oriental, upon motion of plaintiff, ordered defendants to vacate therein may be. Indeed, it is well settled that the legality of a seizure can be contested only by the party
the land and to pay, jointly and severally, the plaintiff the sum of PHP 18 as damages, plus costs. whose rights have been impaired thereby, and that the objection to an unlawful search and seizure
Defendant’s appealed the decision claiming that their failure to get notice from post office was due to is purely personal and cannot be availed of by third parties.
ignorance and that they were not served with copy of plaintiff’s motion for default.
CFI denied defendant’s motion on June 19, 1940. Officers of certain corporations can not validly object to the use in evidence against them of the
ISSUE: Was defendant’s failure to get notice valid and excusable? documents, papers and things seized from the offices and premises of the corporations adverted to
RULING: No, the Court ruled that in the first place, defendant’s are not entitled to a service of copy of the above, since the right to object to the admission of said papers in evidence belongs exclusively to the
motion that they are in default except when they file a motion to set aside the order of default, in which corporations, to whom the seized effects belong, and may not be invoked by the corporate officers in
event they are entitled to notice of all further proceedings (Rule 27, Section 9, Rules of Court). Second, proceedings against them in their individual capacity.
failure to get notice was not excusable neglect as claimed by defendants. Indeed, a notice, is deemed
served if delivered by registered mail and claimed within 5 days from first notice of postmaster (Rule 27, II
Section 8, Rules of Court).
Appealed order affirmed. The Constitution provides:

Stonehill vs. Diokno The right of the people to be secure in their persons, houses, papers, and effects against unreasonable
searches and seizures shall not be violated, and no warrants shall issue but upon probable cause, to be
Facts: determined by the judge after examination under oath or affirmation of the complainant and the witnesses
he may produce, and particularly describing the place to be searched, and the persons or things to be
Upon application of the prosecutors (respondent) several judges (respondent) issued on different dates a seized.
total of 42 search warrants against petitioners (Stonehill et. al.) and/or corporations of which they were
officers to search the persons of the petitioner and/or premises of their officers warehouses and/or Two points must be stressed in connection with this constitutional mandate, namely: (1) that no warrant
residences and to seize and take possession of the personal property which is the subject of the offense, shall issue but upon probablecause, to be determined by the judge in the manner set forth in said
stolen, or embezzled and proceeds of fruits of the offense, or used or intended to be used or the means provision; and (2) that the warrant shall particularly describe the things to be seized.
of committing the offense, which is described in the application as violation of Central Bank Laws, Tariff
and Customs Laws, Internal Revenue Code and the Revised Penal Code. Search warrants issued upon applications stating that the natural and juridical person therein named had
committed a "violation of Central Ban Laws, Tariff and Customs Laws, Internal Revenue (Code) and
Revised Penal Code." In other words, no specific offense had been alleged in said applications. The
Petitioners filed with the Supreme Court this original action for certiorari, prohibition and mandamus and averments thereof with respect to the offense committed were abstract. As a consequence, it
injunction and prayed that, pending final disposition of the case, a writ of preliminary injunction be issued was impossible for the judges who issued the warrants to have found the existence of probable cause, for
against the prosecutors, their agents and representatives from using the effect seized or any copies the same presupposes the introduction of competent proof that the party against whom it is sought has
thereof, in the deportation case and that thereafter, a decision be rendered quashing the contested performed particular acts, or committed specific omissions, violating a given provision of our criminal
search warrants and declaring the same null and void. For being violative of the constitution and the laws.
Rules of court by: (1) not describing with particularity the documents, books and things to be seized; (2)
money not mentioned in the warrants were seized; (3) the warrants were issued to fish evidence for General search warrants are outlawed because the sanctity of the domicile and the privacy of
deportation cases filed against the petitioner; (4) the searches and seizures were made in an illegal communication and correspondence at the mercy of the whims caprice or passion of peace officers.
manner; and (5) the documents paper and cash money were not delivered to the issuing courts for
disposal in accordance with law. To prevent the issuance of general warrants this Court deemed it fit to amend Section 3 of Rule 122 of the
former Rules of Court by providing in its counterpart, under the Revised Rules of Court that "a search
In their answer, the prosecutors (respondent) alleged; (1) search warrants are valid and issued in warrant shall not issue but upon probable cause in connection with one specific offense." Not satisfied
accordance with law; (2) defects of said warrants, were cured by petitioners consent; and (3) in any event with this qualification, the Court added thereto a paragraph, directing that "no search warrant shall issue
the effects are admissible regardless of the irregularity. for more than one specific offense."

The Court granted the petition and issued the writ of preliminary injunction. However by a resolution, the Seizure of books and records showing all business transaction of petitioners persons, regardless of
writ was partially lifted dissolving insofar as paper and things seized from the offices of the corporations. whether the transactions were legal or illegal contravened the explicit command of our Bill of Rights - that
the things to be seized be particularly described - as well as tending to defeat its major objective the
Issues: elimination of general warrants.

1.) Whether or not the petitioners have the legal standing to assail the legality of search warrants issued III
against the corporation of which they were officers.
Most common law jurisdiction have already given up the Moncado ruling and eventually adopted the
2.) Whether or not the search warrants issued partakes the nature of a general search warrants. exclusionary rule, realizing that this is the only practical means of enforcing the constitutional
injunction against unreasonable searches and seizures. In the language of Judge Learned Hand:
3.) Whether or not the seized articles were admissible as evidence regardless of the illegality of its
seizure. As we understand it, the reason for the exclusion of evidence competent as such, which has been
unlawfully acquired, is that exclusion is the only practical way of enforcing the constitutional privilege. In
Held: earlier times the action of trespass against the offending official may have been protection enough; but
that is true no longer. Only in case the prosecution which itself controls the seizing officials, knows that it warrant before wiretapping private phone lines. In the case of Silverman v. United States (1961), the
cannot profit by their wrong will that wrong be repressed. Supreme Court refined the Olmsteadtrespass doctrine by holding that an unreasonable search occurs
only if a "constitutionally protected area" has been intruded upon.
The non-exclusionary rule is contrary, not only to the letter, but also, to the spirit of the constitutional At his trial, Katz sought to exclude any evidence connected with these wiretaps, arguing that the
injunction against unreasonable searches and seizures. To be sure, if the applicant for a search warrant warrantless wiretapping of a public phone booth constitutes an unreasonable search of a "constitutionally
has competent evidence to establish probable cause of the commission of a given crime by the party protected area" in violation of the Fourth Amendment. The federal agents countered by saying that a
against whom the warrant is intended, then there is no reason why the applicant should not comply with public phone booth was not a "constitutionally protected area," therefore, they could place a wiretap on it
the requirements of the fundamental law. Upon the other hand, if he has no such competent evidence, without a warrant.
then it is not possible for the Judge to find that there is probable cause, and, hence, no justification for the
issuance of the warrant. The only possible explanation (not justification) for its issuance is the necessity ISSUE
of fishing evidence of the commission of a crime. But, then, this fishing expedition is indicative of the Does the warrantless wiretapping of a public phone booth violate the unreasonable search and seizure
absence of evidence to establish a probable cause. clause of the Fourth Amendment to the United States Constitution?
RULING: Yes
The Court held that the doctrine adopted in the Moncado case must be, as it is hereby, abandoned; that REASONING:
the warrants for the search of three (3) residences of herein petitioners, as specified in the Resolution of By a 7-1 vote, the U.S. Supreme Court agreed with Katz and held that placing of a warrantless wiretap on
June 29, 1962, are null and void; that the searches and seizures therein made are illegal; that the writ of a public phone booth constitutes an unreasonable search in violation of the Fourth Amendment. The
preliminary injunction heretofore issued, in connection with the documents, papers and other effects thus majority opinion, written by Justice Potter Stewart, however, did not address the case from the
seized in said residences of herein petitioners is hereby made permanent; that the writs prayed for are perspective of a "constitutionally protected area." In essence, the majority argued that both sides in the
granted, insofar as the documents, papers and other effects so seized in the aforementioned residences case were wrong to think that the permissibility of a warrantless wiretap depended upon the area being
are concerned; that the aforementioned motion for Reconsideration and Amendment should be, as it is placed under surveillance. "For the Fourth Amendment protects people, not places. What a person
hereby, denied; and that the petition herein is dismissed and the writs prayed for denied, as regards the knowingly exposes to the public, even in his own home or office, is not a subject of Fourth Amendment
documents, papers and other effects seized in the twenty-nine (29) places, offices and other premises protection . . . . But what he seeks to preserve as private even in an area accessible to the public, may be
enumerated in the same Resolution, without special pronouncement as to costs. constitutionally protected," the Court stated.
Alih vs Castro Building upon this reasoning, the Court held that it was the duty of the Judiciary to review petitions for
warrants in instances in which persons may be engaging in conduct that they wish to keep secret, even if
This case was in explaining Sec 3 of Art 2 of the 1987 Constitution regarding the supremacy of the it were done in a public place. The Court held that, in the absence of a judicially authorized search
civilians. warrant, the wiretaps of the public phone booth used by Katz were illegal. Therefore, the evidence against
him gathered from his conversations should be suppressed.
Facts; CONCURRENCE
Justice John Marshall Harlan's Concurrence: Test for Constitutionally Protected Searches
Zona was conducted by the men of Maj. Gen Castro in a compoud where petioners reside and conducted Although he agreed with the majority opinion of the Court, Justice Harlan went further to provide a test for
illegal search and thereafter seized guns from them. The order was carried on by his Castro's men with what is a constitutionally protected search. He said it was necessary to clarify when private actions,
the contention that the petitioners are involved in the latest killing of the town's mayor Cesar Climaco. conducted in a public place, may be constitutionally protected. Expanding upon the general principles
Issue; enunciated by the majority opinion, Justice Harlan proposed the following two-pronged test to address
this issue: "My understanding of the rule that has emerged from prior judicial decisions is that there is a
Is the warrantless search and seizure legal? twofold requirement, first that a person have exhibited an actual (subjective) expectation of privacy; and
Held; second, that the expectation be one that society is prepared to recognize as 'reasonable.'"
Both the Supreme Court and the lower federal courts have looked to this two-pronged test, and not the
The Supreme Court declared those seized in custodia legis and declared that the operation conducted by majority holding per se, to determine when private actions in public places may be constitutionally
Maj. Gen. Castro was ILLEGAL. The respondents have all the time to obtain a search warrant granted protected. In essence, this concurrence has come to be seen as the main point of the Katz decision, and
that they have about 10 trial courts. The SC also held the protection of the petitioner's human rights as it is the test that, typically, has been used when deciding upon the constitutionality of warrantless
stated in Art IV Sec 3 and 4 of the 1973 Constitution regarding illegal search and seizure. The wiretaps.
presumption of innocence of the petitioners should be observed and that they cannot be subjected to self-
incriminating instances like paraffin tests, photographing and finger printing. Jose Burgos vs. Chief of Staff
Facts:
As penned by J. Cruz in this case, "The Constitution is a law for rulers and people, equally in war and in Two warrants were issued against petitioners for the search on the premises of “Metropolitan Mail” and
peace, and covers with the shield of its protection all classes of men, at all times and under all “We Forum” newspapers and the seizure of items alleged to have been used in subversive activities.
circumstances. No doctrine, involving more pernicious consequences, was ever invented by the wit of Petitioners prayed that a writ of preliminary mandatory and prohibitory injunction be issued for the return
man than that any of its provisions can be suspended during any of the great exigencies of government." of the seized articles, and that respondents be enjoined from using the articles thus seized as evidence
against petitioner.
Katz v. United States, 389 U.S. 347 (1967)
The warrantless wiretapping of a public pay phone violates the unreasonable search and seizure Petitioners questioned the warrants for the lack of probable cause and that the two warrants issued
protections of the Fourth Amendment. indicated only one and the same address. In addition, the items seized subject to the warrant were real
FACTS properties.
The petitioner, Charles Katz, was charged with conducting illegal gambling operations across state lines
in violation of federal law. In order to collect evidence against Katz, federal agents placed a warrantless Issue:
wiretap on the public phone booth that he used to conduct these operations. The agents listened only to Whether or not the two warrants were valid to justify seizure of the items.
Katz's conversations, and only to the parts of his conversations dealing with illegal gambling transactions.
In the case of Olmstead v. United States (1928), the Supreme Court held that the warrantless wiretapping Held:
of phone lines did not constitute an unreasonable search under the Fourth Amendment. According to the The defect in the indication of the same address in the two warrants was held by the court as a
Court, physical intrusion (a trespass) into a given area, and not mere voice amplification (the normal typographical error and immaterial in view of the correct determination of the place sought to be searched
result of a wiretap), is required for an action to constitute a Fourth Amendment search. This is known as set forth in the application. The purpose and intent to search two distinct premises was evident in the
the "trespass doctrine." Partly in response to this decision, Congress passed the Federal issuance of the two warrant.
Communications Act of 1933. This Act required, among other things, federal authorities to obtain a
As to the issue that the items seized were real properties, the court applied the principle in the case seized cannot be returned and shall be destroyed, except the liquors, playing cards, distilled water and
of Davao Sawmill Co. v. Castillo, ruling “that machinery which is movable by nature becomes immobilized five bottles of Streptomycin.
when placed by the owner of the tenement, property or plant, but not so when placed by a tenant,
usufructuary, or any other person having only a temporary right, unless such person acted as the agent of PP VS SALANGUIT
the owner.” In the case at bar, petitioners did not claim to be the owners of the land and/or building on G.R. No. 133254-55,
which the machineries were placed. This being the case, the machineries in question, while in fact bolted FACTS:
to the ground remain movable property susceptible to seizure under a search warrant. On December 26, 1995, around 10:30 p.m., a group of about ten (10) policemen, along with one civilian
However, the Court declared the two warrants null and void. informer, went to the residence of the accused-appellant, Roberto Salanguit y Ko, a search warrant was
shown, and the police operatives started searching the house. They found heat-sealed transparent plastic
Probable cause for a search is defined as such facts and circumstances which would lead a reasonably bags containing a white crystalline substance, a paper clip box also containing a white crystalline
discreet and prudent man to believe that an offense has been committed and that the objects sought in substance, and two bricks of dried leaves which appeared to be marijuana. A receipt of the items seized
connection with the offense are in the place sought to be searched. was prepared, but the accused-appellant refused to sign it. Charges against Roberto Salanguit y Ko for
violations of Republic Act (RA) 6425, i.e. for possession of shabu and marijuana, (Criminal Cases Q-95-
The Court ruled that the affidavits submitted for the application of the warrant did not satisfy the 64357 and Q-95-64358, respectively) were filed, and after hearing, the trial court convicted him in
requirement of probable cause, the statements of the witnesses having been mere generalizations. Criminal Cases Q-95-64357 and Q-95-64358 for violation of Section 16 and 8, respectively.
The accused-appellant contended that the evidence against him was inadmissible because the warrant
Furthermore, jurisprudence tells of the prohibition on the issuance of general warrants. (Stanford vs. State used in obtaining it was invalid.
of Texas). The description and enumeration in the warrant of the items to be searched and seized did not ISSUES:
indicate with specification the subversive nature of the said items. 1. Was the warrant find invalid for failure of providing evidence to support the seizure of “drug
paraphernalia?
ASIAN SURETY and INSURANCE COMPANY, INC., petitioner 2. Whether the marijuana may be included as evidence in light of the “plain view doctrine.”
v RULING:
HON. JOSE HERRERA, respondent Yes. The warrant authorized the seizure of “undetermined quantity of shabu and drug paraphernalia.”
Facts: Evidence was presented showing probable cause of the existence of methamphetamine hydrochloride or
Petition to quash and annul a search warrant issued by respondent Judge Jose Herrera of the City Court shabu. The fact that there was no probable cause to support the application for the seizure of drug
of Manila, and to command respondents to return immediately the documents, papers, receipts and paraphernalia does not warrant the conclusion that the search warrant is void. This fact would be material
records alleged to have been illegally seized thereunder by agents of the National Bureau of Investigation only if drug paraphernalia was in fact seized by the police. The fact is that none was taken by virtue of the
(NBI) led by respondent Celso Zoleta, Jr. search warrant issued. If at all, therefore, the search warrant is void only insofar as it authorized the
On October 27, 1965, respondent Judge Herrera, upon the sworn application of NBI agent Celso Zoleta, seizure of drug paraphernalia, but it is valid as to the seizure of methamphetamine hydrochloride as to
Jr. supported by the deposition of his witness, Manuel Cuaresma, issued a search warrant in connection which evidence was presented showing probable cause as to its existence. In sum, with respect to the
with an undocketed criminal case for estafa, falsification, insurance fraud, and tax evasion, against the seizure of shabu from Salanguit’s residence, Search Warrant 160 was properly issued, such warrant
Asian Surety and Insurance Co., a corporation duly organized and existing under the laws of the being founded on probable cause personally determined by the judge under oath or affirmation of the
Philippines, with principal office at Room 200 Republic Supermarket Bldg., Rizal Avenue, Manila. deposing witness and particularly describing the place to be searched and the things to be seized. With
Armed with the search warrant Zoleta and other agents assigned to the Anti-graft Division of the NBI respect to, and in light of the “plain view doctrine,” the police failed to allege the time when the marijuana
entered the premises of the Republic Supermarket Building and served the search warrant upon Atty. was found, i.e., whether prior to, or contemporaneous with, the shabu subject of the warrant, or whether it
Alidio of the insurance company, in the presence of Mr. William Li Yao, president and chairman of the was recovered on Salanguit’s person or in an area within his immediate control. Its recovery, therefore,
board of directors of the insurance firm. After the search they seized and carried away two (2) carloads of presumably during the search conducted after the shabu had been recovered from the cabinet, as
documents, papers and receipts. attested to by SPO1 Badua in his deposition, was invalid. Thus, the Court affirmed the decision as to
Issue: Criminal Case Q-95-64357, accused-appellant Roberto Salanguit y Ko guilty of possession of illegal
Whether or not the search warrant is void. drugs S16 of R.A.No. 6425.
Ruling:
In the case at bar, the search warrant was issued for four separate and distinct offenses of : (1) estafa, (2) PLACER VS. JUDGE VILLANUEVA [126 SCRA 463; G.R. NOS. L-60349-62; 29 DEC 1983]
falsification, (3) tax evasion and (4) insurance fraud, in contravention of the explicit command of Section Tuesday, February 03, 2009 Posted by Coffeeholic Writes
3, Rule 126, of the Rules providing that: "no search warrant shall issue for more than one specific Labels: Case Digests, Political Law
offense."
PREMISES CONSIDERED, petition is hereby granted; the search warrant of October 27, 1965, is Facts: Petitioners filed informations in the city court and they certified that Preliminary Investigation and
nullified and set aside, and the respondents are hereby ordered to return immediately all documents, Examination had been conducted and that prima facie cases have been found. Upon receipt of said
papers and other objects seized or taken thereunder. Without costs. informations, respondent judge set the hearing of the criminal cases to determine propriety of issuance of
warrants of arrest. After the hearing, respondent issued an order requiring petitioners to submit to the
CASTRO VS PABALAN court affidavits of prosecution witnesses and other documentary evidence in support of the informations
Facts: Judge Pabalan ordered the issuance of a search warrant despite failure of the application of to aid him in the exercise of his power of judicial review of the findings of probable cause by petitioners.
Lumang or the warrant itself to specify the offense, to examine the applicant as well as his witnesses on Petitioners petitioned for certiorari and mandamus to compel respondent to issue warrants of arrest. They
the part of the Judge, and to describe with particularity the place to be searched and the things to be contended that the fiscal’s certification in the informations of the existence of probable cause constitutes
seized. Judge never refuted the assertions when required to answer. Application alleged that applicants sufficient justification for the judge to issue warrants of arrest.
wee informed and claimed that they verified the report that Maria Castro and Co Ling are in possession of
narcotics and other contraband in Barrio Padasil, Bangar, La Union without specifying the particular place
in the Barrio. No complete description of the goods and inquiry was brief. Upon actual search, it turned Issue: Whether or Not respondent city judge may, for the purpose of issuing warrants of arrest, compel
out that it was in Barrio Ma. Cristina and not in Padasil. the fiscal to submit to the court the supporting affidavits and other documentary evidence presented
during the preliminary investigation.
Issue: Whether or not the search warrant is validly issued.

Held: Search warrant issued illegal for violation of the 1935 Constitution and the Rules of Court because Held: Judge may rely upon the fiscal’s certification for the existence of probable cause and on the basis
the two basic requirements are not complied with: (a) no warrant shall issue but upon probable cause, (b) thereof, issue a warrant of arrest. But, such certification does not bind the judge to come out with the
the warrant shall particularly describe the things to be seized, thus, a general warrant. However, things warrant. The issuance of a warrant is not a mere ministerial function; it calls for the exercise of judicial
discretion on the part of issuing magistrate. Under Section 6 Rule 112 of the Rules of Court, the judge
must satisfy himself of the existence of probable cause before issuing a warrant of arrest. If on the face of However, after a review of the above resolution, Special Prosecution Officer Leonardo P. Tamayo
the information, the judge finds no probable cause, he may disregard the fiscal’s certification and require (hereafter, “SPO Tamayo”) recommended that both Rolando Narciso and Doris Teresa Ho be charged
submission of the affidavits of witnesses to aid him in arriving at the conclusion as to existence of with violation of Section 3 (e) of R.A. 3019. The resolution of GIO Labrador, as modified by the
probable cause. memorandum[5] of SPO Tamayo, was approved by Ombudsman Conrado M. Vasquez on May 5, 1992.
Thus, herein petitioners were charged accordingly before the Sandiganbayan in an information[6] filed on
Petition dismissed. May 18, 1992. Attached to the information were the resolution of GIO Labrador and the memorandum of
SPO Tamayo. The said information reads:
Maximo Soliven vs Ramon Makasiar
167 SCRA 393 – Political Law – Constitutional Law – President’s Immunity From Suit – Must Be Invoked “The undersigned Special Prosecution Officer III, Office of the Special Prosecutor, hereby accuses
by the President ROLANDO NARCISO and DORIS TERESA HO, President of National Marine Corporation, of violation of
Luis Beltran is among the petitioners in this case. He, together with others, was charged with libel by the Section 3(e) of RA 3019, as amended, committed as follows:
then president Corzaon Aquino. Cory herself filed a complaint-affidavitagainst him and others. Makasiar
averred that Cory cannot file a complaint affidavitbecause this would defeat her immunity from suit. He That on or about April 4, 1989, and for sometime prior and/or subsequent thereto, in the City of Manila,
grounded his contention on the principle that a president cannot be sued. However, if a president would Philippines and within the jurisdiction of this Honorable Court, the above-named accused ROLANDO
sue then the president would allow herself to be placed under the court’s jurisdiction and conversely she NARCISO, a public officer, being then the Vice-President of the National Steel Corporation (NSC), a
would be consenting to be sued back. Also, considering the functions of a president, the president may government-owned or controlled corporation organized and operating under the Philippine laws, and
not be able to appear in court to be a witness for herself thus she may be liable for contempt. DORIS TERESA HO, a private individual and then the President of National Marine Corporation (NMC), a
ISSUE: Whether or not such immunity can be invoked by Beltran, a person other than the president. private corporation organized and operating under our Corporation law, conspiring and confederating with
HELD: No. The rationale for the grant to the President of the privilege of immunity from suit is to assure one another, did then and there wilfully, unlawfully and criminally, with evident bad faith and through
the exercise of Presidential duties and functions free from any hindrance or distraction, considering that manifest partiality, cause undue injury to the National Steel Corporation (NSC), by entering without legal
being the Chief Executive of the Government is a job that, aside from requiring all of the office-holder’s justification into a negotiated contract of affreightment disadvantageous to the NSC for the haulage of its
time, also demands undivided attention. products at the rate of P129.50/MT, from Iligan City to Manila, despite their full knowledge that the rate
But this privilege of immunity from suit, pertains to the President by virtue of the office and may be they have agreed upon was much higher than those offered by the Loadstar Shipping Company, Inc.
invoked only by the holder of the office; not by any other person in the President’s behalf. Thus, an (LSCI) and Premier Shipping Lines, Inc. (PSLI), in the amounts of P109.56 and P123.00 per Metric Ton,
accused like Beltran et al, in a criminal case in which the President is the complainant cannot raise the respectively, in the public bidding held on June 30, 1988, thereby giving unwarranted benefits to the
presidential privilege as a defense to prevent the case from proceeding against such accused. National Marine Corporation, in the total sum of One Million One Hundred Sixteen Thousand Fifty Two
Moreover, there is nothing in our laws that would prevent the President from waiving the privilege. Thus, if Pesos and Seventy Five Centavos (P1,116,052.75), Philippine Currency, to the pecuniary damage and
so minded the President may shed the protection afforded by the privilege and submit to the court’s prejudice of the NSC in the aforestated sum. The said offense was committed by Rolando S. Narciso in
jurisdiction. The choice of whether to exercise the privilege or to waive it is solely the President’s the performance of his official functions as Vice-President of the National Steel Corporation.
prerogative. It is a decision that cannot be assumed and imposed by any other person.
CONTRARY TO LAW.”
HO VS PEOPLE OF THE PHILIPPINES (full text)
PANGANIBAN, J.: Acting on the foregoing information, the Sandiganbayan issued the now questioned warrant of arrest
against Petitioners Ho and Narciso. Petitioner Ho initially questioned the issuance thereof in an “Urgent
May a judge issue a warrant of arrest solely on the basis of the report and recommendation of the Motion to Recall Warrant of Arrest/Motion for Reconsideration” which was adopted by Petitioner Narciso.
investigating prosecutor, without personally determining probable cause by independently examining They alleged that the Sandiganbayan, in determining probable cause for the issuance of the warrant for
sufficient evidence submitted by the parties during the preliminary investigation? their arrest, merely relied on the information and the resolution attached thereto, filed by the Ombudsman
without other supporting evidence, in violation of the requirements of Section 2, Article III of the
The Case Constitution, and settled jurisprudence. Respondent Sandiganbayan denied said motion in the challenged
Resolution. It ratiocinated in this wise:
This is the main question raised in these two consolidated petitions for certiorari under Rule 65 of the
Rules of Court challenging the Sandiganbayan’s August 25, 1992 Resolution[1] which answered the said
query in the affirmative. “Considering, therefore, that this Court did not rely solely on the certification appearing in the information
in this case in the determination of whether probable cause exists to justify the issuance of the warrant of
The Facts arrest but also on the basis predominantly shown by the facts and evidence appearing in the
resolution/memorandum of responsible investigators/ prosecutors, then the recall of the warrant of arrest,
Both petitions have the same factual backdrop. On August 8, 1991, the Anti-Graft League of the or the reconsideration sought for, cannot be granted. More so, when the information, as filed, clearly
Philippines, represented by its chief prosecutor and investigator, Atty. Reynaldo L. Bagatsing, filed with shows that it is sufficient in form and substance based on the facts and evidence adduced by both parties
the Office of the Ombudsman a complaint[2] against Doris Teresa Ho, Rolando S. Narciso (petitioners in during the preliminary investigation. To require this Court to have the entire record of the preliminary
G.R. Nos. 106632 and 106678, respectively), Anthony Marden, Arsenio Benjamin Santos and Leonardo investigation to be produced before it, including the evidence submitted by the complainant and the
Odoño. The complaint was for alleged violation of Section 3 (g) of Republic Act 3019[3] prohibiting a accused-respondents, would appear to be an exercise in futility.”
public officer from entering into any contract or transaction on behalf of the government if it is manifestly
and grossly disadvantageous to the latter, whether or not the public officer profited or will profit thereby. Thus, these petitions.
After due notice, all respondents therein filed their respective counter-affidavits with supporting
documents. On January 8, 1992, Graft Investigation Officer Titus P. Labrador (hereafter, “GIO Labrador”) The Issue
submitted his resolution[4] with the following recommendations:
Petitioner Ho raises this sole issue:
“WHEREFORE, all premises considered, it is respectfully recommended that an information for violation
of Section 3 (g) of R.A. 3019 as amended be filed against respondent Rolando S. Narciso before the “May a judge determine probable cause and issue [a] warrant of arrest solely on the basis of the
Sandiganbayan. resolution of the prosecutor (in the instant case, the Office of the Special Prosecutor of the Ombudsman)
who conducted the preliminary investigation, without having before him any of the evidence (such as
It is likewise recommending that the case against the other respondents be DISMISSED for insufficiency complainant’s affidavit, respondent’s counter-affidavit, exhibits, etc.) which may have been submitted at
of evidence.” the preliminary investigation?”[7]
And clarifying the statement in People vs. Delgado[13] -- that the “trial court may rely on the resolution of
In his separate petition, Rolando S. Narciso adopts the foregoing and raises no other distinct issue. the COMELEC to file the information, by the same token that it may rely on the certification made by the
prosecutor who conducted the preliminary investigation, in the issuance of the warrant of arrest” -- this
Petitioners Ho and Narciso similarly contend that a judge, in personally determining the existence of Court underscored in Lim Sr. vs. Felix[14] that “[r]eliance on the COMELEC resolution or the Prosecutor’s
probable cause, must have before him sufficient evidence submitted by the parties, other than the certification presupposes that the records of either the COMELEC or the Prosecutor have been submitted
information filed by the investigating prosecutor, to support his conclusion and justify the issuance of an to the Judge and he relies on the certification or resolution because the records of the investigation
arrest warrant. Such evidence should not be “merely described in a prosecutor’s resolution.” Citing sustain the recommendation.” We added, “The warrant issues not on the strength of the certification
People vs. Inting,[8] petitioners insist that the judge “must have before him ‘the report, the affidavits, the standing alone but because of the records which sustain it.” Summing up, the Court said:
transcripts of stenographic notes (if any), and all other supporting documents which are material in
assisting the judge to make his determination.’” “We reiterate the ruling in Soliven vs. Makasiar that the Judge does not have to personally examine the
complainant and his witnesses. The Prosecutor can perform the same functions as a commissioner for
The Court’s Ruling the taking of the evidence. However, there should be a report and necessary documents supporting the
Fiscal’s bare certification. All of these should be before the Judge.
The petitions are meritorious.
”The extent of the Judge’s personal examination of the report and its annexes depends on the
The pertinent provision of the Constitution reads: circumstances of each case. We cannot determine beforehand how cursory or exhaustive the Judge’s
examination should be. The Judge has to exercise sound discretion for, after all, the personal
“Section 2 [, Article III]. The right of the people to be secure in their persons, houses, papers, and determination is vested in the Judge by the Constitution. It can be as brief or as detailed as the
effects against unreasonable searches and seizures of whatever nature and for any purpose shall be circumstances of each case require. To be sure, the Judge must go beyond the Prosecutor’s certification
inviolable, and no search warrant or warrant of arrest shall issue except upon probable cause to be and investigation report whenever necessary. He should call for [the] complainant and [the] witnesses
determined personally by the judge after examination under oath or affirmation of the complainant and the themselves to answer the court’s probing questions when the circumstances of the case so require.”[15]
witnesses he may produce and particularly describing the place to be searched and the persons or things [underscoring supplied]
to be seized.” (Underscoring supplied.)
The above rulings in Soliven, Inting and Lim Sr. were iterated in Allado vs. Diokno[16] where we explained
In explaining the object and import of the aforequoted constitutional mandate, particularly the power and again what probable cause means. Probable cause for the issuance of a warrant of arrest is the
the authority of judges to issue warrants of arrest, the Court elucidated in Soliven vs. Makasiar[9]: existence of such facts and circumstances that would lead a reasonably discreet and prudent person to
believe that an offense has been committed by the person sought to be arrested.[17] Hence, the judge,
“What the Constitution underscores is the exclusive and personal responsibility of the issuing judge to before issuing a warrant of arrest, “must satisfy himself that based on the evidence submitted there is
satisfy himself of the existence of probable cause. In satisfying himself of the existence of probable sufficient proof that a crime has been committed and that the person to be arrested is probably guilty
cause for the issuance of a warrant of arrest, the judge is not required to personally examine the thereof.”[18] At this stage of the criminal proceeding, the judge is not yet tasked to review in detail the
complainant and his witnesses. Following established doctrine and procedure, he shall: (1) personally evidence submitted during the preliminary investigation. It is sufficient that he personally evaluates such
evaluate the report and the supporting documents submitted by the fiscal regarding the existence of evidence in determining probable cause.[19] In Webb vs. De Leon,[20] we stressed that the judge merely
probable cause and, on the basis thereof, issue a warrant of arrest; or (2) if on the basis thereof he finds determines the probability, not the certainty, of guilt of the accused and, in doing so, he need not conduct
no probable cause, he may disregard the fiscal’s report and require the submission of supporting a de novo hearing. He simply personally reviews the prosecutor’s initial determination finding probable
affidavits of witnesses to aid him in arriving at a conclusion as to the existence of probable cause.”[10] cause to see if it is supported by substantial evidence.
[underscoring supplied]
In the recent case of Roberts Jr. vs. Court of Appeals,[21] this Court’s application of the dictum laid down
We should stress that the 1987 Constitution requires the judge to determine probable cause “personally.” in Soliven -- affirmed and fortified in Inting, Lim Sr., Allado and Webb -- should lay to rest the issue raised
The word “personally” does not appear in the corresponding provisions of our previous Constitutions. in the instant petitions. In Roberts Jr., this Court, through Mr. Justice Hilario G. Davide, Jr., set aside the
This emphasis shows the present Constitution’s intent to place a greater degree of responsibility upon order of the respondent judge directing inter alia the issuance of warrants of arrest against the accused,
trial judges than that imposed under the previous Charters. reasoning that said judge did not personally determine the existence of probable cause, since he had
“only the information, amended information, and Joint Resolution as bases thereof. He did not have the
While affirming Soliven, People vs. Inting[11] elaborated on what “determination of probable cause” records or evidence supporting the prosecutor’s finding of probable cause.”
entails, differentiating the judge’s object or goal from that of the prosecutor’s.
In like manner, herein Respondent Sandiganbayan had only the information filed by the Office of the
“First, the determination of probable cause is a function of the Judge. It is not for the Provincial Fiscal or Ombudsman, the thirteen-page resolution of the investigating officer and the three-page memorandum of
Prosecutor nor for the Election Supervisor to ascertain. Only the Judge and the Judge alone makes this the prosecution officer, when it issued the warrant of arrest against the petitioners. The latter two
determination. documents/reports even had dissimilar recommendations -- the first indicting only Petitioner Narciso, the
second including Petitioner Ho. This alone should have prompted the public respondent to verify, in the
“Second, the preliminary inquiry made by a Prosecutor does not bind the Judge. It merely assists him to records and other documents submitted by the parties during the preliminary investigation, whether there
make the determination of probable cause. The Judge does not have to follow what the Prosecutor was sufficient evidence to sustain the Ombudsman’s action charging both petitioners with violation of Sec.
presents to him. By itself, the Prosecutor’s certification of probable cause is ineffectual. It is the report, 3(e) of Anti-Graft law. But in its initial justification of the issuance of the warrant, the Sandiganbayan
the affidavits the transcripts of stenographic notes (if any), and all other supporting documents behind the simply said:
Prosecutor’s certification which are material in assisting the Judge to make his determination.
“JUSTICE ESCAREAL:
“And third, Judges and Prosecutors alike should distinguish the preliminary inquiry which determines
probable cause for the issuance of a warrant of arrest from the preliminary investigation proper which xxx xxx xxx
ascertains whether the offender should be held for trial or released. Even if the two inquiries are
conducted in the course of one and the same proceeding, there should be no confusion about the But in this particular case we believe there is a prima facie case based on our examination of the
objectives. The determination of probable cause for the warrant of arrest is made by the Judge. The resolution because we believe, we think the Ombudsman will not approve a resolution just like that,
preliminary investigation proper -- whether or not there is reasonable ground to believe that the accused without evidence to back it up.”[22]
is guilty of the offense charged and, therefore, whether or not he should be subjected to the expense,
rigors and embarrassment of trial -- is the function of the Prosecutor.”[12] In attempting to further justify its challenged action, the public respondent explained in its assailed
Resolution:
is: he cannot rely solely and entirely on the prosecutor’s recommendation, as Respondent Court did in
“In the instant case, there are attached to the information, two (2) Memorandum/Resolution (sic) this case. Although the prosecutor enjoys the legal presumption of regularity in the performance of his
emanating from the Offices of the Ombudsman and the Special Prosecutor (Pp. 4-6, 7-19, respectively, official duties and functions, which in turn gives his report the presumption of accuracy, the Constitution,
Record) which clearly and indubitably established, firstly, the conduct of a due and proper preliminary we repeat, commands the judge to personally determine probable cause in the issuance of warrants of
investigation, secondly, the approval by proper officials clothed with statutory authority; and thirdly, the arrest. This Court has consistently held that a judge fails in his bounden duty if he relies merely on the
determination and ascertainment of probable cause based on the documentary evidence submitted by certification or the report of the investigating officer.
the complainant (Anti-Graft League of the Philippines), foremost among which is the Contract of
Affreightment entered into between National Steel Corporation (NSC), and National Marine Corporation True, in Webb vs. De Leon, we found that “the painstaking recital and analysis of the parties’ evidence
(NMC) and the COA-NSC audit report, together with the counter-affidavits of accused Rolando Narciso made in the DOJ Panel Report satisfied both judges that there [was] probable cause to issue warrants of
and NMC officials, among whom is accused-movant. Outlined in detail in the aforesaid Resolution of arrest against petitioners.” This statement may have been wrongly construed by the public respondent to
Titus P. Labrador, Graft Investigation Officer II, which was reviewed by Attys. Romeo I. Tan and Arturo mean that the narration or description of portions of the evidence in the prosecutor’s report may serve as
Mojica, Director, Community Coordination Bureau and Assistant Ombudsman, PACPO, [respectively,] are sufficient basis to make its own independent judgment. What it should bear in mind, however, is that,
the facts leading to the questioned transaction between NSC and NMC, together with an evaluation of the aside from the 26-page report of the DOJ panel, the sworn statements of three witnesses and counter-
propriety and legality of the bidding process involved therein and which revealed that there were affidavits of the petitioners in Webb were also submitted to the trial court, and the latter is presumed to
supposed non-compliance with proper bidding procedures. GIO Labrador’s findings and have reviewed these documents as well, prior to its issuance of the warrants of arrest.
recommendations, extensively set out in his thirteen-page resolution, is complemented by the three-page
Memorandum of Special Prosecution Officer II Leonardo P. Tamayo, both of which meticulously delved In the instant case, the public respondent relied fully and completely upon the resolution of the graft
into the merits and demerits of the evidence presented by the complainant and accused-respondents and investigation officer and the memorandum of the reviewing prosecutor, attached to the information filed
which resulted in their respective recommendation which led the Honorable Conrado M. Vasquez to before it, and its conjecture that the Ombudsman would not have approved their recommendation without
approve the recommendations of Deputy Special Prosecutor Jose de G. Ferrer and Special Prosecutor supporting evidence. It had no other documents from either the complainant (the Anti-Graft League of the
Aniano A. Desierto for the filing of the information in the case at bar. Philippines) or the People from which to sustain its own conclusion that probable cause exists. Clearly
and ineluctably, Respondent Court’s findings of “the conduct of a due and proper preliminary
xxx xxx xxx investigation” and “the approval by proper officials clothed with statutory authority” are not equivalent to
the independent and personal responsibility required by the Constitution and settled jurisprudence. At
“Considering, therefore, that this Court did not rely solely on the certification appearing in the information least some of the documentary evidence mentioned (Contract of Affreightment between National Steel
in this case in the determination of whether probable cause exists to justify the issuance of the warrant of Corporation and National Marine Corporation, the COA-NSC audit report, and counter-affidavits of
arrest but also on the basis predominantly shown by the facts and evidence appearing in the Rolando Narciso and NMC officials), upon which the investigating officials of the Ombudsman reportedly
resolution/memorandum of responsible investigators/ prosecutors, then the recall of the warrant of arrest, ascertained the existence of probable cause, should have been physically present before the public
or the reconsideration sought for, cannot be granted. More so, when the information, as filed, clearly respondent for its examination, to enable it to determine on its own whether there is substantial evidence
shows that it is sufficient in form and substance based on the facts and evidence adduced by both parties to support the finding of probable cause. But it stubbornly stood pat on its position that it had essentially
during the preliminary investigation. To require this Court to have the entire record of the preliminary complied with its responsibility. Indisputably, however, the procedure it undertook contravenes the
investigation to be produced before it, including the evidence submitted by the complainant and the Constitution and settled jurisprudence. Respondent Court palpably committed grave abuse of discretion
accused-respondents, would appear to be an exercise in futility.”[23] in ipso facto issuing the challenged warrant of arrest on the sole basis of the prosecutor’s findings and
recommendation, and without determining on its own the issue of probable cause based on evidence
In light of the aforecited decisions of this Court, such justification cannot be upheld. Lest we be too other than such bare findings and recommendation.
repetitive, we only wish to emphasize three vital matters once more: First, as held in Inting, the
determination of probable cause by the prosecutor is for a purpose different from that which is to be made WHEREFORE, the petitions are GRANTED and the assailed Resolution is SET ASIDE. The warrant
by the judge. Whether there is reasonable ground to believe that the accused is guilty of the offense issued by the Sandiganbayan (Second Division) on May 20, 1992 in Case No. 17674 for the arrest of
charged and should be held for trial is what the prosecutor passes upon. The judge, on the other hand, Petitioners Doris Teresa Ho and Rolando Narciso is hereby declared NULL AND VOID.
determines whether a warrant of arrest should be issued against the accused, i.e. whether there is a
necessity for placing him under immediate custody in order not to frustrate the ends of justice.[24] Thus, Webb v De Leon (Criminal Procedure)
even if both should base their findings on one and the same proceeding or evidence, there should be no Webb v De Leon
confusion as to their distinct objectives. GR No. 121234
August 23, 1995
Second, since their objectives are different, the judge cannot rely solely on the report of the prosecutor in
finding probable cause to justify the issuance of a warrant of arrest. Obviously and understandably, the FACTS:
contents of the prosecutor’s report will support his own conclusion that there is reason to charge the
accused of an offense and hold him for trial. However, the judge must decide independently. Hence, he On June 19, 1994, the National Bureau of Investigation filed with the DOJ a letter-complaint charging
must have supporting evidence, other than the prosecutor’s bare report, upon which to legally sustain his petitioners Hubert Webb, Michael Gatchalian, Antonio J. Lejano and 6 other persons with the crime of
own findings on the existence (or nonexistence) of probable cause to issue an arrest order. This Rape and Homicide of Carmela N. Vizconde, her mother Estrellita Nicolas-Vizconde, and her sister Anne
responsibility of determining personally and independently the existence or nonexistence of probable Marie Jennifer in their home at Number 80 W. Vinzons, St., BF Homes, Paranaque, Metro Manila on June
cause is lodged in him by no less than the most basic law of the land. Parenthetically, the prosecutor 30, 1991.
could ease the burden of the judge and speed up the litigation process by forwarding to the latter not only
the information and his bare resolution finding probable cause, but also so much of the records and the Forthwith, the DOJ formed a panel of prosecutors headed by Asst Chief State Prosecutor Jovencio R.
evidence on hand as to enable His Honor to make his personal and separate judicial finding on whether Zuno to conduct the preliminary investigation.
to issue a warrant of arrest.[25]
fault the DOJ Panel for its finding of probable cause. They assail the credibility of Jessica Alfaro as
Lastly, it is not required that the complete or entire records of the case during the preliminary investigation inherently weak and uncorroborated due to her inconsistencies between her April 28, 1995 and May 22,
be submitted to and examined by the judge.[26] We do not intend to unduly burden trial courts by obliging 1995 sown statements. They criticize the procedure followed by the DOJ Panel when it did not examine
them to examine the complete records of every case all the time simply for the purpose of ordering the witnesses to clarify the alleged inconsistencies. Petitioners:
arrest of an accused. What is required, rather, is that the judge must have sufficient supporting
documents (such as the complaint, affidavits, counter-affidavits, sworn statements of witnesses or charge that respondent Judge Raul de Leon and respondent Judge Amelita Tolentino issued warrants of
transcripts of stenographic notes, if any) upon which to make his independent judgment or, at the very arrest against them without conducting the required preliminary examination. 
least, upon which to verify the findings of the prosecutor as to the existence of probable cause. The point
Complain about the denial of their constitutional right to due process and violation of their right to an On 12 April 1993, the information was filed with the trial court without anything accompanying it. A copy of
impartial investigation. They also assail the prejudicial publicity that attended their preliminary the investigating prosecutor’s Joint Resolution was forwarded to and received by the trial court only on 22
investigation.  April 1993. However, no affidavits of the witnesses, transcripts of stenographic notes of the proceedings
during the preliminary investigation, or other documents submitted in the course thereof were found in the
ISSUES: records of the case as of 19 May 1993.

(1) Did the DOJ Panel gravely abuse its discretion in holding that there is probable cause to charge On 15 April 1993, petitioners Roberts, et al. filed a petition for review to the Department of Justice seeking
accused with crime of rape and homicide? the reversal of the finding of probable cause by the investigating prosecutor. They also moved for the
suspension of the proceedings and the holding in abeyance of the issuance of warrants of arrest against
(2) Did respondent judges de Leon and Tolentino gravely abuse their discretion when they failed to them. Meanwhile, the public prosecutor also moved to defer the arraignment of the accused-appellants
conduct a preliminary examination before issuing warrants of arrest against the accused? pending the final disposition of the appeal to the Department of Justice.

(3) Did the DOJ Panel deny them their constitutional right to due process during their preliminary On 17 May 1993, respondent Judge Asuncion issued the challenged order (1) denying, on the basis of
investigation? Crespovs. Mogul, the foregoing motions respectively filed by the petitioners and the public prosecutor,
and directing the issuance of the warrants of arrest “after June 1993” and setting the arraignment on 28
(4) Did the DOJ Panel unlawfully intrude into judicial prerogative when it failed to charge Jessica Alfaro in June 1993. In part, respondent judge stated in his order that since the case is already pending in this
the information as an accused? Court for trial, following whatever opinion the Secretary of Justice may have on the matter would
undermine the independence and integrity his court. To justify his order, he quoted the ruling of the
HELD: Supreme Court in Crespo, which stated:

(1) NO. Valid determination -- A probable cause needs only to rest on evidence showing that more likely In order therefor to avoid such a situation whereby the opinion of the Secretary of Justice who reviewed
than not, a crime has been committed and was committed by the suspects. Probable cause need not be the action of the fiscal may be disregarded by the trial court, the Secretary of Justice should, as far as
based on clear and convincing evidence of guilt, neither on evidence establishing guilt beyond reasonable practicable, refrain from entertaining a petition for review or appeal from the action of the fiscal, when the
doubt and definitely, not on evidence establishing absolute certainty of guilt. complaint or information has already been filed in Court. The matter should be left entirely for the
determination of the Court.
(2) NO. Valid arrest -- In arrest cases, there must be a probable cause that a crime has been committed
and that the person arrested committed it. Petitioners went to the Court of Appeals (CA), arguing that the respondent judge had not the slightest
basis at all for determining probable cause when he ordered the issuance of warrants of arrest. After
Section 6 of Rule 112 provides that – “upon filing of an information, the RTC may issue a warrant for the finding that a copy of the public prosecutor’s Joint Resolution had in fact been forwarded to, and received
accused.” by, the trial court on 22 April 1993, the CA denied petitioners’ application for writ of preliminary injunction.
The CA ruled that the Joint Resolution “was sufficient in itself to have been relied upon by respondent
Clearly then, our laws repudiate the submission that respondent judges should have conducted Judge in convincing himself that probable cause indeed exists for the purpose of issuing the
“searching examination of witnesses” before issuing warrants of arrest against them. corresponding warrants of arrest” and that the “mere silence of the records or the absence of any express
declaration” in the questioned order as to the basis of such finding does not give rise to an adverse
(3) NO. There is no merit in this contention because petitioners were given all the opportunities to be inference, for the respondent Judge enjoys in his favor the presumption of regularity in the performance of
heard. his official duty. Roberts, et al. sought reconsideration, but meanwhile, the DOJ affirmed the finding of
probable cause by the investigating prosecutor. The CA therefore dismissed the petition for mootness.
The DOJ Panel precisely requested the parties to adduce more evidence in their behalf and for the panel
to study the evidence submitted more fully. II. THE ISSUES

(4) NO. 1. Did Judge Asuncion commit grave abuse of discretion in denying, on the basis of Crespo vs. Mogul,
the motions to suspend proceedings and hold in abeyance the issuance of warrants of arrest and to defer
Petitioner's argument lacks appeal for it lies on the faulty assumption that the decision whom to prosecute arraignment until after the petition for review filed with the DOJ shall have been resolved?
is a judicial function, the sole prerogative of courts and beyond executive and legislative interference.
2. Did Judge Asuncion commit grave abuse of discretion in ordering the issuance of warrants of arrest
In truth, the prosecution of crimes appertains to the executive department whose principal power and without examining the records of the preliminary investigation?
responsibility is to see that our laws are faithfully executed. A necessary component of this right is to
prosecute their violators. 3. May the Supreme Court determine in this [sic] proceedings the existence of probable cause either for
the issuance of warrants of arrest against the petitioners or for their prosecution for the crime of estafa?
ROBERTS VS CA
III. THE RULING
Paul G. Roberts, et al. v. Court of Appeals, et al., G.R. No. 113930, March 5, 1996
[The Court, in a 7-5-2 vote, GRANTED the petition. It SET ASIDE the decision and resolution of the CA,
DECISION the resolutions of the DOJ 349 Committee, and the order of respondent judge.]

DAVIDE, JR., J.: 1. YES, Judge Asuncion committed grave abuse of discretion in denying, on the basis of Crespo vs.
Mogul, the motions to suspend proceedings and hold in abeyance the issuance of warrants of arrest and
I. THE FACTS to defer arraignment until after the petition for review filed with the DOJ shall have been resolved.

Petitioners, who are corporate officers and members of the Board of Pepsi Cola Products Phils., Inc. were There is nothing in Crespo vs. Mogul which bars the DOJ from taking cognizance of an appeal, by way of
prosecuted in connection with the Pepsi “Number Fever” promotion by handlers of the supposedly a petition for review, by an accused in a criminal case from an unfavorable ruling of the investigating
winning “349” Pepsi crowns. Of the four cases filed against the petitioners, probable cause was found by prosecutor. It merely advised the DOJ to, “as far as practicable, refrain from entertaining a petition for
the investigating prosecutor only for the crime of estafa, but not for the other alleged offenses. review or appeal from the action of the fiscal, when the complaint or information has already been filed in
Court.”
extensions to prolong stay in Philippines, their visas expired on Sept. 10, 1962 and were ordered by
Whether the DOJ would affirm or reverse the challenged Joint Resolution is still a matter of guesswork. Commissioner of Immigration (COI) on Aug. 31, 1962 thru a letter, to leave the country on or before Sept.
Accordingly, it was premature for respondent Judge Asuncion to deny the motions to suspend 10, 1962 with warning of issuance of warrant of arrest for failure to leave and confiscation of bond.
proceedings and to defer arraignment on the following grounds: Petitioners then filed with the CFI of Manila for Mandamus to compel COI to cancel their ACR, to stop
issuing arrest warrant, and preliminary injunction from confiscating their bond. They argue that Chan Sau
This case is already pending in this Court for trial. To follow whatever opinion the Secretary of Justice Wah became a Filipino Citizen upon marriage to Esteban Morano by virtue of Section 15 of
may have on the matter would undermine the independence and integrity of this Court. This Court is still Commonwealth Act No. 473 (Revised Naturalization Act). Likewise, it argues that Section 37 of the
capable of administering justice. Naturalization Law is unconstitutional for allowing the COI to issue warrant of arrest and effecting
deportation without judicial intervention enshrined in the Constitution. CFI decided partly against
The real and ultimate test of the independence and integrity of this court is not the filing of the petitioners, thus, COI and petitioners both appealed to SC.
aforementioned motions [to suspend proceedings and issuance of warrants of arrest and to defer Issues:
arraignment] at that stage but the filing of a motion to dismiss or to withdraw the information on the basis Whether or not the marriage of Chan Sau Wah to Esteban Morano makes her a Filipino citizen.
of a resolution of the petition for review reversing the Joint Resolution of the investigating prosecutor. Whether or not Section 37 of the Naturalization Law empowering the COI to issue a warrant of arrest, and
However, once a motion to dismiss or withdraw the information is filed the trial judge may grant or deny it, deport upon a warrant on deportation cases is unconstitutional for are covered by the Constitutional
not out of subservience to the Secretary of Justice, but in faithful exercise of judicial prerogative. mandate on searches and seizures without judicial intervention required under the Constitution on
searches and seizures.
2. YES, Judge Asuncion committed grave abuse of discretion in ordering the issuance of warrants of Ruling:
arrest without examining the records of the preliminary investigation. Citizenship. Marriage to a Filipino citizen does not ipso facto make her a Filipino citizen. She must show
that she possess all the qualifications, and none of the disqualifications required by the Naturalization
The teachings then of Soliven, Inting, Lim, Allado, and Webb reject the proposition that the investigating Law requiring as follows:
prosecutor’s certification in an information or his resolution which is made the basis for the filing of the Valid marriage; and
information, or both, would suffice in the judicial determination of probable cause for the issuance of a Alien woman herself might be lawfully naturalized
warrant of arrest. In Webb, this Court assumed that since the respondent Judges had before them not In the additional stipulation of facts of July 3, 1963, petitioners admit that Chan Sau Wah is not possessed
only the 26-page resolution of the investigating panel but also the affidavits of the prosecution witnesses of all the qualifications required by the Naturalization Law. Thus, she did not become a Filipino citizen.
and even the counter-affidavits of the respondents, they (judges) made personal evaluation of the Searches and seizures. Power to deport aliens is an attribute of sovereignty planted on the accepted
evidence attached to the records of the case. maxim of international law, that every sovereign nation has the power, as inherent in sovereignty, and
essential to self-preservation, to forbid the entrance of foreigners within its dominions. Section 1 (3),
In this case, nothing accompanied the information upon its filing on 12 April 1993 with the trial court. A Article III of the Constitution, does not require judicial intervention in the execution of a final order of
copy of the Joint Resolution was forwarded to, and received by, the trial court only on 22 April 1993. And deportation issued in accordance with law. The constitutional limitation contemplates an order of arrest in
as revealed by the certification of respondent judge’s clerk of court, no affidavits of the witnesses, the exercise of judicial power as a step preliminary or incidental to prosecution or proceeding for a given
transcripts of stenographic notes of the proceedings during the preliminary investigation, or other offense or administrative action, not as a measure indispensable to carry out a valid decision by a
documents submitted in the course thereof were found in the records of this case as of 19 May 1993. competent official, such as a legal order of deportation, issued by the Commissioner of Immigration, in
Clearly, when respondent Judge Asuncion issued the assailed order of 17 May 1993 directing, among pursuance of a valid legislation.
other things, the issuance of warrants of arrest, he had only the information, amended information, and Petition for mandamus and prohibition with respect to petitioners Chan Sau Wah is hereby denied; and
Joint Resolution as bases thereof. He did not have the records or evidence supporting the prosecutor's judgment declaring her a citizen of the Philippines, directing COI to cancel her Alien Certificate of
finding of probable cause. And strangely enough, he made no specific finding of probable cause; he Registration and other immigration papers, and declaring the preliminary injunction with respect to her
merely directed the issuance of warrants of arrest “after June 21, 1993.” It may, however, be argued that permanent, are all hereby set aside; and in all other respects, the decision appealed is hereby affirmed.
the directive presupposes a finding of probable cause. But then compliance with a constitutional
requirement for the protection of individual liberty cannot be left to presupposition, conjecture, or even HARVEY V. DEFENSOR-SANTIAGO [162 SCRA 840; G.R. NO. 82544; 28 JUN 1988]
convincing logic. Facts: This is a petition for Habeas Corpus. Petitioners are the following: American nationals Andrew
Harvey, 52 and Jonh Sherman 72. Dutch Citizen Adriaan Van Den Elshout, 58. All reside at Pagsanjan
3. NO, the Supreme Court MAY NOT determine in this [sic] proceedings the existence of probable Laguna respondent Commissioner Miriam Defensor Santiago issued Mission Orders to the Commission
cause either for the issuance of warrants of arrest against the petitioners or for their prosecution for the of Immigration and Deportation (CID) to apprehended petitioners at their residences. The “Operation
crime of estafa. Report” read that Andrew Harvey was found together with two young boys. Richard Sherman was found
with two naked boys inside his room. While Van Den Elshout in the “after Mission Report” read that two
Ordinarily, the determination of probable cause is not lodged with this Court. Its duty in an appropriate children of ages 14 and 16 has been under his care and subjects confirmed being live-in for sometime
case is confined to the issue of whether the executive or judicial determination, as the case may be, of now.
probable cause was done without or in excess of jurisdiction or with grave abuse of discretion amounting
to want of jurisdiction. This is consistent with the general rule that criminal prosecutions may not be Seized during the petitioner’s apprehension were rolls of photo negatives and photos of
restrained or stayed by injunction, preliminary or final. suspected child prostitutes shown in scandalous poses as well as boys and girls engaged in sex. Posters
and other literature advertisingthe child prostitutes were also found.
There are, however, exceptions to the foregoing rule. But the Court refused to reevaluate the evidence to
determine if indeed there is probable cause for the issuance of warrants of arrest in this case. For the Petitioners were among the 22 suspected alien pedophiles. They were apprehended 17 February1988
respondent judge did not, in fact, find that probable cause exists, and if he did he did not have the basis after close surveillance for 3 month of the CID in Pagsanjan, Laguna. 17 of the arrested aliens opted for
therefor. Moreover, the records of the preliminary investigation in this case are not with the Court. They self-deportation. One released for lack of evidence, another charged not for pedophile but working with
were forwarded by the Office of the City Prosecutor of Quezon City to the DOJ in compliance with the NO VISA, the 3 petitioners chose to facedeportation proceedings. On 4
latter's 1st Indorsement of 21 April 1993. The trial court and the DOJ must be required to perform their March1988, deportation proceedings were instituted against aliens for being undesirable aliens under
duty. Sec.69 of Revised Administrative Code.

Morano v. Vivo Warrants of Arrest were issued 7March1988 against petitioners for violation of Sec37, 45 and 46 of
Facts: Petitioners are Chinese nationals (Chan Sau Wah from Fukein, with a minor child from prior Immigration Act and sec69 of Revised Administrative Code. Trial by the Board of Special Inquiry III
marriage, Fu Yan Fun) who were granted a temporary visitor’s visa as mom-immigrant for 2 months upon commenced the same date. Petition for bail was filed 11March 1988 but was not granted by the
posting P4k cash bond to visit a cousin in the Philippines. She soon married to Esteban Morano, a Commissioner of Immigration. 4 April1988 Petitioners filed a petitionfor Writ of Habeas Corpus. The court
Filipino Citizen, on January 24, 1962 and gave birth to a child, Esteban Morano, Jr. After several heard the case on oral argument on 20 April 1988.
Issue: Whether or Not the Philippine Overseas EmploymentAdministration (or the Secretary of Labor) can
validly issue warrants of search and seizure (or arrest) under Article 38 of the Labor Code
Issues:

(1) Whether or Not the Commissioner has the power to arrest and detain petitioners pending Held: Under the new Constitution, “. . . no search warrant or warrant of arrest shall issue except upon
determination of existence of probable cause. probable cause to be determined personally by the judge after examination under oath or affirmation of
the complainant and the witnesses he may produce, and particularly describing the place to be searched
(2) Whether or Not there was unreasonable searches and seizures by CIDagents. and the persons or things to be seized”. Mayors and prosecuting officers cannot issue warrants of seizure
or arrest. The Closure and Seizure Order was based on Article 38 of the Labor Code. The Supreme Court
(3) Whether or Not the writ of Habeas Corpus may be granted to petitioners. held, “We reiterate that the Secretary of Labor, not being a judge, may no longer issue search or arrest
warrants. Hence, the authorities must go through the judicial process. To that extent, we declare Article
38, paragraph (c), of the Labor Code, unconstitutional and of no force and effect… The power of the
Held: While pedophilia is not a crime under the Revised Penal Code, it violates the declared policy of the President to order the arrest of aliens fordeportation is, obviously, exceptional. It (the power to order
state to promote and protect the physical, moral, spiritual and social well being of the youth. The arrest of arrests) cannot be made to extend to other cases, like the one at bar. Under the Constitution, it is the sole
petitioners was based on the probable cause determined after close surveillance of 3 months. The domain of the courts.” Furthermore, the search and seizure order was in the nature of a general warrant.
existence of probable cause justified the arrest and seizure of articles linked to the offense. The articles The court held that the warrant is null and void, because it must identify specifically the things to be
were seized as an incident to a lawful arrest; therefore the articles are admissible evidences (Rule 126, seized.
Section12 of Rules on Criminal Procedure).
WHEREFORE, the petition is GRANTED. Article 38, paragraph (c) of the Labor Code is declared
The rule that search and seizures must be supported by a valid warrant of arrest is not an absolute rule. UNCONSTITUTIONAL and null and void. The respondents are ORDERED to return all materials seized
There are at least three exceptions to this rule. 1.) Search is incidental to the arrest. 2.) Search in a as a result of the implementation of Search and Seizure Order No. 1205.
moving vehicle. 3.) Seizure of evidence in plain view. In view of the foregoing, the search done was
incidental to the arrest. ALVAREZ vs CFI

The filing of the petitioners for bail is considered as a waiver of any irregularity attending their arrest and Facts: The Anti-Usury Board of the Department of Justice presented to Judge David a sworn affidavit that
estops them from questioning its validity. Furthermore, the deportation charges and the hearing presently a certain Narciso Alvarez is in possession of books, receipts, chits, lists used by him as money
conducted by the Board of Special Inquiry made their detention legal. It is a fundamental rule that habeas lender/usurer charging usurious rates in violation of law. Affiant Almeda, chief of the task force, didn’t say
corpus will not be granted when confinement is or has become legal, although such confinement was that the information was based on his personal knowledge but was only received by him from a reliable
illegal at the beginning. source. Subsequently, the judge issued the warrant ordering the search of Alvarez’ house. On June 4,
1936, the agents raided the subject place and seized different documents namely, banknotes, bankbooks,
The deportation charges instituted by the Commissioner of Immigration are in accordance with Sec37 (a) stubs, cashbooks, bills of lading, credit receipts, etc. Thereafter, the articles seized was not brought
of the Philippine Immigration Act of 1940 in relation to sec69 of the Revised Administrative code. Section immediately to the custody of the judge who issued the SW. Alvarez moved that the agents of the Board
37 (a) provides that aliens shall be arrested and deported upon warrant of the Commissioner of be declared guilty of contempt and prays that all articles in question be returned to him because the SW
Immigration and Deportation after a determination by the Board of Commissioners of the existence of a issued was illegal. On the other hand, the Anti-Usury Board pleaded that they be allowed to retain
ground for deportation against them. Deportation proceedings are administrative in character and never custody of the articles seized for further investigation. When the judge sustained the latter’s motion.
construed as a punishment but a preventive measure. Therefore, it need not be conducted strictly in Alvarez elevated the matter to the SC and prayed that the search warrant as well as the order of the
accordance with ordinary Court proceedings. What is essential is that there should be a specific charge judge authorizing the Anti-Usury Board to retain custody be declared null and void.
against the alien intended to be arrested and deported. A fair hearing must also be conducted with Issue: Whether the SW issued by the judge is illegal for having solely as basis the affidavit of Agent
assistance of a counsel if desired. Almeda in whose oath the latter declared that he had no personal knowledge of the facts which were to
serve as basis for the issuance of the warrant but he had knowledge thereof only through information
Lastly, the power to deport aliens is an act of the State and done under the authority of the sovereign secured from a person whom he considered reliable.
power. It a police measure against the undesirable aliens whose continued presence in the country is Ruling: Section 1, paragraph 3, of Article III of the Constitution and Section 97 of General Orders 58
found to be injurious to the public good and tranquility of the people. require that there be not only probable cause before the issuance of a search warrant but that the search
warrant must be based upon an application supported by oath of the applicant and the witnesses he may
produce. In its broadest sense, an oath includes any form of attestation by which a party signifies that he
SALAZAR VS. ACHACOSO [183 SCRA 145; G.R. NO. 81510; 14 MAR 1990] is bound in conscience to perform an act faithfully and truthfully; and it is sometimes defined as an
outward pledge given by the person taking it that his attestation or promise is made under an immediate
Facts: Rosalie Tesoro of Pasay City in a sworn statement filed with the POEA, charged petitioner with sense of his responsibility to God. The oath required must refer to the truth of the facts within the
illegal recruitment. Public respondent Atty. Ferdinand Marquez sent petitioner a telegram directing him to personal knowledge of the petitioner or his witnesses, because the purpose thereof is to convince
appear to the POEA regarding the complaint against him. On the same day, after knowing that petitioner the committing magistrate, not the individual making the affidavit and seeking the issuance of the
had no license to operate a recruitment agency, publicrespondent Administrator Tomas Achacoso issued warrant, of the existence of probable cause. The true test of sufficiency of an affidavit to warrant
a Closure and Seizure Order No. 1205 to petitioner. It stated that there will a seizure of the documents issuance of a search warrant is whether it has been drawn in such a manner that perjury could be
and paraphernalia being used or intended to be used as the means of committing illegal recruitment, it charged thereon and affiant be held liable for damages caused . The affidavit, which served as the
having verified that petitioner has— (1) No valid license or authority from the Department of exclusive basis of the search warrant, is insufficient and fatally defective by reason of the manner in
Labor andEmployment to recruit and deploy workers for overseas employment; (2) Committed/are which the oath was made, and therefore, the search warrant and the subsequent seizure of the books,
committing acts prohibited under Article 34 of the New Labor Code in relation to Article 38 of the same documents and other papers are illegal. Further, it is the practice in this jurisdiction to attach the affidavit
code. A team was then tasked to implement the said Order. The group, accompanied by mediamen and of at least the applicant or complainant to the application. It is admitted that the judge who issued the
Mandaluyong policemen, went to petitioner’s residence. They served the order to a certain Mrs. For a search warrant in this case, relied exclusively upon the affidavit made by agent Almeda and that he did
Salazar, who let them in. The team confiscated assorted costumes. Petitioner filed with POEA a letter not require nor take the deposition of any other witness. The Constitution does not provide that it is of an
requesting for the return of the seized properties, because she was not given prior notice and hearing. imperative necessity to take the depositions of the witnesses to be presented by the applicant or
The said Order violated due process. She also alleged that it violated sec 2 of the Bill of Rights, and the complainant in addition to the affidavit of the latter. The purpose of both in requiring the presentation of
properties were confiscated against her will and were done with unreasonable force and intimidation. depositions is nothing more than to satisfy the committing magistrate of the existence of probable cause.
Therefore, if the affidavit of the applicant or complainant is sufficient, the judge may dispense with that of
other witnesses. Inasmuch as the affidavit of the agent was insufficient because his knowledge of the
facts was not personal but merely hearsay, it is the duty of the judge to require the affidavit of one or more the use first, of water spewed from fire hoses, and later, tear gas. Eventually there was gunfire. Within
witnesses for the purpose of determining the existence of probable cause to warrant the issuance of the moments, rallyists lay dead on and by the National Road. The fatalities numbered fifteen (15), according
search warrant. When the affidavit of the applicant or complainant contains sufficient facts within his to the military officers; twenty-nine (29), according to the demonstrators.
personal and direct knowledge, it is sufficient if the judge is satisfied that there exists probable cause;
when the applicant’s knowledge of the facts is mere hearsay, the affidavit of one or more witnesses In the afternoon of that day, Escalante Town Mayor Braulio Lumayno, in view of the absence of the
having a personal knowledge of the facts is necessary. Thus the warrant issued is likewise illegal because Municipal Circuit Court Judge (Emilio Ignalaga), took cognizance of a complaint filed by the Military
it was based only on the affidavit of the agent who had no personal knowledge of the facts Station Commander charging some of the rallyists with the felony of inciting to sedition, and after
avowedly conducting an investigation of the witnesses presented by the complainant, issued an order for
Mata v. Bayona the arrest of certain of the demonstrators. His order reads as follows:
FACTS: Soriano Mata was accused under Presidential Decree (PD) 810, as amended by PD 1306, the
information against him alleging that Soriano Mata offered, took and arranged bets on the Jai Alai game It appearing that the Presiding Municipal Circuit Court Judge, Hon. Emilio M. Ignalaga, is on official leave
by “selling illegal tickets known as ‘Masiao tickets’ without any authority from the Philippine Jai Alai & of absence and, in the interest of justice, the undersigned has to urgently act on the complaint filed by the
Amusement Corporation or from the government authorities concerned.” Mata claimed that during the Station Commander, against the above-named accused for "INCITING TO SEDITION" and, on the basis
hearing of the case, he discovered that nowhere from the records of the said case could be found the of the evidence submitted after a searching question and answer were conducted and, being satisfied
search warrant and other pertinent papers connected to the issuance of the same, so that he had to that said crime has been committed, in order not to frustrate the ends of justice, it is necessary that the
inquire from the City Fiscal its whereabouts, and to which inquiry Judge Josephine K. Bayona, presiding above-named accused be placed under custody. Let therefore, a warrant of arrest be issued for said
Judge of the City Court of Ormoc replied, “it is with the court”. The Judge then handed the records to the above-named accused.
Fiscal who attached them to the records. This led Mata to file a motion to quash and annul the search
warrant and for the return of the articles seized, citing and invoking, among others, Section 4 of Rule 126 Bail recommended: P12,000.00. 2
of the Revised Rules of Court. The motion was denied by the Judge on 1 March 1979, stating that the
court has made a thorough investigation and examination under oath of Bernardo U. Goles and Reynaldo In the record of the Court a quo appear the following inter alia:
T. Mayote, members of the Intelligence Section of 352nd PC Co./Police District II INP; that in fact the
court made a certification to that effect; and that the fact that documents relating to the search warrant 1) Complaint for "Inciting to Sedition" (RPC 142, as and by PD 183 and PD 1974), signed by a Capt.
were not attached immediately to the record of the criminal case is of no moment, considering that the Jugan, and sworn to before Mayor Lumayno on Sept. 20, 1985, bearing the stamped notation of filing with
rule does not specify when these documents are to be attached to the records. Mata’s motion for the MTC: "9/24/85, 4:00 PM:" 3
reconsideration of the aforesaid order having been denied, he came to the Supreme Court, with the
petition for certiorari, praying, among others, that the Court declare the search warrant to be invalid for its 2) 2-page Affidavit of Capt. Sanson, dated, and sworn to before Actg.. City Fiscal Abros (Cadiz City) on
alleged failure to comply with the requisites of the Constitution and the Rules of Court, and that all the Sept. 20, 1985; 4
articles confiscated under such warrant as inadmissible as evidence in the case, or in any proceedings on
the matter. 3) 3-page document, "Searching Questions and Answers" signed by Capt. Sanson, dated. and sworn to
ISSUE: WON the judge must before issuing the warrant personally examine on oath or affirmation the before Mayor Lumayno on, September 20, 1985; 5
complainant and any witnesses he may produce and take their depositions in writing, and attach them to
the record, in addition to any affidavits presented to him? 4) 3-page sworn statement of Godofredo Hoyo-A y Jayme, General Manager of the Balintawak-Escalante
HELD:YES. Under the Constitution “no search warrant shall issue but upon probable cause to be Water District; 6
determined by the Judge or such other responsible officer as may be authorized by law after examination
under oath or affirmation of the complainant and the witnesses he may produce”. More emphatic and 5) Affidavit of Leopoldo Villalon; 7
detailed is the implementing rule of the constitutional injunction, The Rules provide that the judge must
before issuing the warrant personally examine on oath or affirmation the complainant and any 6) Affidavit of Elpidio Carbajosa; 8 and
witnesses he may produce and take their depositions in writing, and attach them to the record, in
addition to any affidavits presented to him. Mere affidavits of the complainant and his witnesses 7) Affidavit of Eduardo Flores. 9
are thus not sufficient. The examining Judge has to take depositions in writing of the complainant and
the witnesses he may produce and to attach them to the record. Such written deposition is necessary in The gist of the testimony of Capt. Sanson and the other affiants is that on September 20, 1985, the
order that the Judge may be able to properly determine the existence or nonexistence of the probable demonstrators, numbering "about 1,000, " had blockaded the main highway in front of the Escalante
cause, to hold liable for perjury the person giving it if it will be found later that his declarations are false. Municipal Building, by massing themselves on the road as well as by piling stones, coconut trunks and
We, therefore, hold that the search warrant is tainted with illegality by the failure of the Judge to conform pieces of wood in the middle of the highway. They were also "shouting invectives, seditious and scurrilous
with the essential requisites of taking the depositions in writing and attaching them to the record, words against the government." Negotiations with Ponsica, Chairman of the Escalante Chapter of
rendering the search warrant invalid. "BAYAN" (Bagong Alyansang Makabayan), to have the road cleared having been unavailing, firemen on
firetrucks began to train a "torrent of water" from their fire hoses on the demonstrators. The rallyists
PONSICA VS IGNALAGA (full text) retaliated by hurling stones at the firemen. One of them "approached the security of the firetruck and
stabbed him." Others climbed aboard the trucks and tried to grab the firehoses and firearms of the
NARVASA, J.: officers. At this point, on Capt. Sanson's orders, his "back-up teams" of soldiers commenced to throw tear
gas at the crowd. One of the demonstrators picked up a tear gas canister and hurled it back at the
The chief issue raised by the petitioners in this case is whether or not Section 143 of the Local soldiers. At the same time gunfire from "different assorted firearms" emanated from the rallyists; and
Government Code1 granting power to the municipal mayor to conduct preliminary investigations and some of the shots hit the blinker of a firetruck and the headlight of another. The soldiers shot back. This
order the arrest of the accused, was repealed by the 1985 Rules on Criminal Procedure promulgated by exchange of gunfire resulted in "fourteen (14) demonstrators killed on the spot. " Recovered at the scene
this Court; and is, in addition, unconstitutional as vesting the power to conduct preliminary investigations were a rifle; a U.S. 45 cal. pistol; 2 "homemade" pistols; 14 steel arrows and 4 assorted slings; 30
in an official who cannot be deemed a "neutral and detached magistrate" within the contemplation of assorted knives; a "pogakhang" with 2 live cartridges; 2 grenades; and several empty shells of different
Section 3, Article IV of the 1973 Constitution. The issue is hereby resolved adversely to the petitioners, caliber. Hours later, another corpse, Identified as that of a demonstrator, too, was brought to the PC
with the stressed qualification that the mayor's power to order arrest ceased to exist as of February 2, Headquarters.
1987 when the new Constitution was ratified by the Filipino people, and that, in any event, the
investigation actually conducted by respondent mayor in the case at bar was fatally defective. The petitioners however give a different version of the facts. What happened, according to them, was that
at 9 o'clock in the morning on that day, a group of demonstrators, "composed mostly of laid-off sugar field
Shortly after noon on September 20, 1985, an attempt was made by firemen and soldiers to disperse a workers," gathered in front of Escalante Municipal Hall "in the exercise of their constitutionally guaranteed
crowd of demonstrators massed in front of the Municipal Building of Escalante, Negros Occidental, with right to freedom of expression and to assemble peacefully to petition the government for redress of
grievances."10 About an hour afterwards, fire trucks arrived one after another, as well as jeeploads of 3) the Mayor's examination "falls short of the requirements of "searching questions and answers;" the
soldiers and CHDF members, in full combat gear, Shortly after noon, after "going thru the motions of statements of the witnesses supposedly interrogated by the mayor, are either merely conclusions of law
negotiating with the demonstrators," the military officers ordered the crowd to disperse; but without or sterile as regards seditious utterances, hence, probable cause was non-existent23 and
warning, fire hoses were trained on and sprayed water at the demonstrators. When the rallyists did not
budge, tear gas canisters were thrown at them. A demonstrator picked up a canister and threw it at an 4) Article 142 of the Revised Penal Code, as amended-under which the petitioners are charged is based
"empty space in the plaza" The soldier and CHDF members thereupon fired indiscriminately at the crowd, on the US Sedition Act of 1978, which has been declared by the US Supreme Court as "repugnant to the
killing 29 and injuring at least 30 persons.11 constitutional guarantee of freedom of speech and expression (New York Times Co. v. Sullivan, 376 U.S.
254);" hence, said Article 142 is also fatally flawed and therefore, the warrant. of arrest in question was in
After the Mayor had referred the case to Municipal Trial Court Judge Ignalaga on September 24, 1985,12 effect issued for a "crime which in the context of the constitutionally guaranteed freedom of speech and
an "URGENT MOTION TO QUASH WARRANT OF ARREST" was filed on September 26, 1985 by expression does not exist."24
petitioners' counsel on the ground that a mayor no longer has authority to conduct preliminary
investigations or issue warrants of arrest that authority having been "withheld in the 1985 New Rules on In his comment filed with this Court,25 Judge Ignalaga argues that —
Criminal Procedure."13 This was opposed by the Station Commander14 who invoked Section 143 of the
Local Government Code providing that "(i)n case of temporary absence of the Judge assigned to the 1) The validity of the preliminary examination on the basis of which the arrest warrants were issued, may
municipality, the mayor may conduct the preliminary examination in criminal cases when, in his opinion not be raised for the first time in the Supreme Court, but should first be ventilated before the RTC having
the investigation cannot be delayed without prejudice to the interest of justice." A reply was filed by the cognizance of the crime.26 In any case, upon the considerations set out in his resolution of October 11,
petitioners after their receipt of the opposition "only last October 8, 1985."15 They contended that the 1985, the preliminary examination in question is valid.27
"power of the municipal mayor to conduct preliminary investigation and issue a warrant of arrest under the
1964 Revised Rules of Court** (had been) impliedly repealed by the 1985 New Rules on Criminal 2) The petitioners raise factual issues which are best left for determination by the RTC, the Supreme
Procedure;" that "the 1985 New Rules on Criminal Procedure being a special law, controls over Court not being a trier of facts.28
provisions of the Local Government Code (BP 337, 1983), which is a general law;" and in any case,
"subject t warrants of arrest should be reviewed and revoked as done without observance of legal 3) The Mayor in fact followed the procedure in the constitution prior to issuing the arrest warrants.29
requisites."
4) The constitutionality of PD 1974 should be assailed by separate petition.30
By Resolution dated Oct. 11, 1985,16 the Judge confirmed the mayor's arrest order. He opined that in the
absence of the judge, the mayor still has authority to conduct preliminary investigations and issue arrest The Solicitor General at the time, Estelito Mendoza, also filed a comment on December 6, 1985.31 The
warrants, since Rule 112, Sec. 2 (d), of the 1985 Rules, includes as among those authorized to conduct comment addressed itself only to the issue of "the validity of the law authorizing municipal mayors to
preliminary investigations, "Such other officers as may be authorized by law;" and the Local G issue warrants of arrest and the law punishing inciting to sedition," in view of the fact that the respondents
government Code of 1983, Section 143, grants a town mayor authority to conduct preliminary had presented their own separate comments on the petition.32 The comment points out that:
examinations in case of the temporary absence of the judge when such investigation cannot be delayed
without prejudice to the interest of justice. The Judge declared that in the case at bar, the mayor had 1) Section 3, Article IV, of the 1973 Constitution, which mentions a "judge, or such other responsible
conducted the examination personally, and having in the exercise of his discretion found probable cause, officer as may be authorized by law" as vested with competence to conduct preliminary investigations, is
issued the warrants of arrest in question; and conceding arguendo irregularity in that the preliminary an innovation. In implementation thereof, BP Blg. 337 was enacted on February 10, 1983, empowering
examination was conducted without according the parties the assistance of lawyers (contrary to par. 2, mayors to conduct preliminary investigations (Sec. 143). This is a recognition of the truism that the
Sec. 143, Chap. 3, Title 2, Book 11, Local Government Code), this does not render the proceedings void determination of probable cause is but a quasi judicial function Ocampo v. US, 58 LED 1231).33
because at any rate, the mayor had duly observed the uniform procedure under PD 91 (citing: People v. Petitioners' American authorities are not applicable: the mayor is the highest official in the municipality; he
Paran, 52 Phil. 712; Hashim v. Boncan, 71 Phil. 216; Lino v. Fugoso, 77 Phil. 933). exercises only general supervision over the police but is not directly involved in police work; the old rules
precisely expressed a recognition of the capability of i mayors to determine probable cause, and the
It is Mayor Lumayno's order for the petitioners' arrest of September 20, 1985, and Jude Ignalaga's Order omission of mayors in the 1985 Rules simply means that the determination of officers who may be
of October 11, 1985 "validating that order of arrest" that the petitioners would have this Court nullify and authorized to conduct preliminary investigations was deemed best left to legislation.34
perpetually enjoin.17
2) The Philippine sedition law is not akin to the US Sedition Law; in any event our own sedition law has
In their petition for certiorari and prohibition filed on October 15, 1985, and their Memorandum of January passed the test of constitutionality (Espuelas v. People, 90 Phil. 524).35
21, 1986,18 they assert that:
Solicitor General Sedfrey A. Ordonez subsequently declared that he "stands by the constitutionality of the
1) while Section 3,Rule 112 of the l964 Rules of Court,empowers the municipal mayor, "in case of statutes the petitioners question and, therefore, sustains the position taken by his predecessor in
temporary absence of both the municipal and the auxiliary municipal judges from the municipality, town or office;"36 this, in response to this Court's resolution dated May 15, 1986, requiring the parties to state
place wherein they exercise their jurisdiction, to make the preliminary examination in criminal cases when whether supervening events had transpired materially affecting the case.37
such examination cannot be delayed without prejudice to the interest of justice," that power has been
removed from him by the 1985 New Rules on Criminal Procedure which "do not mention the Municipal It is clear from the outset that the issue before this Court does not involved the adjudgment of the guilt or
Mayor as among the officers authorized to conduct preliminary investigation (Section 2, Rule 112), much innocence of the soldiers in the tragic and regrettable killings in front of the Escalante Town Hall in the
less to issue an order of arrest (Section 6, Ibid.)19 early afternoon of that day, the twentieth of September, 1985, an event that caused a great outcry of
lamentation and condemnation throughout the land. This is a matter that should be and is in fact now
2) Section 143 of BP Blg. 337 (Local Government Code) — under which the order of arrest in question is subject of a separate criminal proceeding. Neither is the propriety of the victims' exercise of their
presumably made to rest-cannot withstand the "constitutional test" of Section 3, Article IV,20 which constitutional rights of free speech and free assembly for redress of grievances in the premises at issue
safeguards the right against unreasonable searches and seizures, and requires the interposition of a here. The basic question before the Court is divorced of the drama and the passion of those issues; it
"judge, or such other responsible officer as may be authorized by law," meaning "a neutral and detached deals mainly with the dry, unexciting, but nonetheless important matter of whether or not the municipal
magistrate competent to determine probable cause (Shadwick v. City of Tampa, 40 LW 4758; Castillo v. mayor has the power to conduct preliminary investigations in the light of the 1985 amendments of the
Jias, 62 SCRA 124; Ang Tibay v. CIR, 69 Phil. 635; Zambales Chromite, etc. v. C.A., 934 SCRA 2617);" rules governing criminal procedure in the Rules of Court. The answer to the question entails a re-
and since a mayor is obviously not such a magistrate (U.S. v. Chadwick, 433 U.S. 197), the orders of examination and analysis of the relevant legal provisions.
arrest at bar are constitutionally infirm;21 moreover, said Section 143 of the Local Government Code is
"merely a rule of procedure** (and is thus) deemed to have been superseded by the New Rules of The 1964 Rules of Court explicitly gave the mayor authority to conduct preliminary investigations.
Criminal Procedure;22
SEC. 3. Preliminary examination by the municipal mayor. — In case of temporary absence of both the
municipal and the auxiliary municipal judges from the municipality, town, or place wherein they exercise The 1973 Constitution plainly and unmistakably grants to the legislature the power to determine which "
their jurisdiction, the municipal mayor shall make the preliminary examination in criminal cases when such responsible officers, " aside from judges, may issue warrants of arrest after examination under oath or
examination cannot be delayed without prejudice to the interest of justice. He shall make a report of any affirmation of the complainant and the witnesses he may produce. And as plainly and unmistakably, the
preliminary examination so made to the municipal or to the auxiliary municipal judge immediately upon legislature, the Batas Pambansa, has in the Local Government Code made the determination that the
the return of one or the other. He shall have authority in such cases to order the arrest of the defendant mayor is such a "responsible officer "and has in consequence authorized him to conduct preliminary
and to grant him bail in the manner and cases provided for in Rule 114. 38 investigations in criminal cases and order the arrest of the accused upon a finding of probable cause.

The Rules of 1940 contained an Identical provision, in Section 3, Rule 108. The first issue raised by the petitioners — that in September ,1985 the mayor no longer had power to
conduct preliminary investigations and issue arrest warrants43 — must therefore be resolved against
The 1985 Rules on Criminal Procedure did not reproduce this provision, and did not include the mayor in them. The argument that Section 143 of the Local Government Code is just "a rule of procedure merely
the enumeration of the officers authorized to conduct preliminary investigations, those listed being judges having incorporated Rule 112, Section 2 of the old rules of Criminal Procedure," and should therefore be
of municipal trial courts and municipal circuit trial courts; city or provincial fiscals and their assistants; deemed "superseded by the New Rules of Criminal Procedure pursuant to the power of this ** Court to
national and regional state prosecutors; and "such other officers as may be authorized by law." 39 promulgate rules of procedure (Article X, Section 5 151, Constitution **)"44 cannot be sustained. However
superior the Supreme Court may be to the other branches of government in the realm of adjudication, its
SEC. 6. When warrant of arrest may issue. — power to ordain rules of court was at the time inferior to the lawmaking power of the legislature.45 It is
true that the 1935 Constitution repealed all procedural laws then in force "as statutes" and declared them
(a) By the Regional Trial Court. — Upon the filing of an information, the Regional Trial Court may issue a to be "rules of court;" but this was only so that they could be subject to repeal or modification by the
warrant for the arrest of the accused Supreme Court, which was given the power to promulgate rules of procedure,46 and has since been
exercising such power by promulgating the Rules of Court of 1940 and of 1964, and the 1985 Rules on
(b) By the Municipal Trial Court. — If the municipal trial judge conducting the preliminary investigation is Criminal Procedure, etc. But the 1935 and 1973 Constitutions explicitly conferred on the legislature the
satisfied after an examination in writing and under oath of the complainant and his witnesses in the form power to repeal, alter or supplement those rules of court, although it would appear that that power is no
of searching questions and answers, that a probable cause exists and that there is a necessity of placing longer granted to it by the 1987 Charter.47
the respondent under immediate custody in order not to frustrate the ends of justice, he shall issue a
warrant of arrest. (n). 40 Now, the conditions under which the mayor could conduct preliminary investigations are also clearly
indicated by the law, to wit:
It is of course the deletion in the 1985 rules of reference to the municipal mayor as among those
authorized to conduct preliminary investigations and order arrests upon which the petitioners chiefly rely 1) in case of the temporary absence of the judge assigned to the municipality; and
as basis for their impugnation of Mayor Lumayno's preliminary investigation and order of arrest based
thereon. 2) in his (the mayor's) opinion, the investigation cannot be delayed without prejudice to the interests of
justice.
The matter is however treated of not solely by the Rules of Court but also by the 1973 Constitution,
Section 3, Article IV whereof reads: On these occasions, the mayor may order the arrest of the accused upon a finding of probable cause
after searching examination of the complainant and the witnesses the latter may present.
SEC. 3. The right of the people to be secure in their persons, houses, papers, and effects against
unreasonable searches and seizures of whatever nature and for any purpose shall not be violated, and In the case at bar, there is no showing that these conditions have not been met. Indeed, quite apart from
no search warrant or warrant of arrest shall issue except upon probable cause to be determined by the the presumption that official duty has been regularly performed.48 and the affirmative statements of
judge, or such other responsible officer as may be authorized by law, after examination under oath or respondent official that on that fateful 20th day of September, 1985, Judge Ignalaga was in Manila"49 on
affirmation of the complainant and the witnesses he may produce, and particularly describing the place to official leave or absence,"50 the petitioners have not put the fulfillment of those conditions at issue at all,
be searched, and the persons or things to be seized. 41 or the matter of whether or not the accused were "assisted by lawyers" in the course of the
investigation.51
Parenthetically, it may be noted that Section 3, Rule III of the 1935 Constitution mentioned only "the
judge" as having power to determine probable cause and issue search and arrest warrants. The petitioners also challenge the constitutionality of Section 143 of the Local Government Code, it being
claimed that the mayor has been thereby authorized to conduct pre investigations and issue warrants of
The matter is further dealt with in Section 143 of Batas Pambansa Bilang 337, otherwise known as the arrest, although he can not be deemed a "neutral and detached magistrate" within the contemplation of
Local Government Code, which took effect on February 10, 1983. This section evidently deems the mayor Section 3, Article IV of the 1973 Constitution.52
a "responsible officer" in contemplation of the cited constitutional provision, and explicitly authorizes him
to conduct preliminary examination in criminal cases and order the arrest of the accused upon probable The competence of the Batasang Pambansa to decide and declare by statute which "responsible
cause. officers," aside from judges, should be entrusted with the authority to conduct preliminary investigations
and issue arrest warrants cannot be doubted in view of the clear language of the Constitution.53 And the
SEC. 143. Authority of the Mayor to Conduct Preliminary Examination. determination by the Batasan that a municipal mayor is one such "responsible officer" who may properly
be entrusted with the function of conducting preliminary investigations and ordering arrests of suspects
(1) In case of temporary absence of the judge assigned to the municipalities the mayor may conduct the upon probable cause, can not be subject of judicial review, absent any indication that the legislative
preliminary examination in criminal cases when, in his opinion the investigation cannot be delayed without proceedings leading to that statutory determination are void on account of some grave cause. Certainly,
prejudice to the interest of justice. the wisdom of the statute, or the validity of the reasons underlying it, or the adequacy of the statistics,
facts and circumstances considered by the legislature in its enactment, are beyond the sphere of inquiry
(2) No examination shall be conducted unless the parties are assisted by lawyers. of the courts54 The Batasan was apparently of the belief that a municipal mayor could be sufficiently
objective and impartial as to be relied upon to conduct preliminary investigations and issue orders of
(3) In cases where he may conduct preliminary examination, the mayor shall upon probable cause after arrest in the exceptional situation when the judge assigned in the municipality was absent. The petitioners
examination of witnesses, have authority order the arrest of the accused and to grant him bail in the disagree. They contend that the mayor cannot in the very nature of things be "neutral and detached." The
manner and cases provided in the Revised Rules of Court and order his provisional release. disagreement, and the fact that plausible reasons may be adduced by one side or the other on the
proposition does not make the question a justiciable one. The theory advocated by the petitioners that the
(4) The mayor shall make a report of any preliminary examination so made immediately after the return of mayor's "deep involvement in law enforcement functions is likely to color his judgment as a trier of
the judge assigned in the area, or upon the designation of his replacement. 42 probable cause,"55 does not induce persuasion. In the first place the premise cannot be conceded. While
it is true that the mayors do "exercise general supervision over units and elements of the INP stationed or were issued by Mayor Lumayno. The facts recited by Capt. Sanson may perhaps warrant a charge of
assigned in their respective jurisdictions," they are not themselves directly involved in police work and traffic obstruction, or grave coercion, or malicious mischief, or illegal possession of firearms or deadly
cannot in any sense be described, as the petitioners do, as being deeply involved in law enforcement weapons, or maybe, attempted murder or homicide. But it is barren of facts to support an accusation of
functions. And even if that "deep involvement" be conceded, it does not follow that this would necessarily inciting to sedition.
preclude their assuming "the cold neutrality of an impartial judge" in conducting preliminary investigations
of persons suspected of crimes. The other evidence on record exhibits the same barreness.

But it must be emphasized here and now that what has just been described is the state of the law as it Two witnesses, Godofredo Hoyo-A and Elpidio Carbajosa, advert in their affidavits63 to the shouting of
was in September, 1985. The law has since been alter. No longer does the mayor have at this time the "seditious words," etc. Hoyo-A is quoted as deposing that "the demonstrators were already unruly ** (and
power to conduct preliminary investigations, much less issue orders of arrest. Section 143 of the Local kept) on shouting anti-government slogans, ** invectiues and seditious words against the government,"64
Government Code, conferring this power on the mayor has been abrogated, rendered functus officio by that certain of the petitioners were Identified as being "against the government, ** and using the (Mt.
the 1987 Constitution which took effect on February 2, 1987, the date of its ratification by the Filipino Carmel) School in doctrining (sic) the students against the government,"65 that petitioner Rogelio Arnaiz
people. Section 2, Article III of the 1987 Constitution pertinently provides that "no search warrant or had delivered a speech in which he had said, "Rumpagon ang dictatorial na regimen ni Pres. Marcos,
warrant of arrest shall issue except upon probable cause to be determined personally by the judge after pamatukan ang NSL, ibagsak ang military kagwalaon ang CHDF, suklan nato ang gobyerno," a woman
examination under oath or affirmation of the complainant and the witnesses he may produce, and speaker had said the same thing, and a third speaker, petitioner Daniel Gempesala, also stated:
particularly describing the place to be searched and the person or things to be seized. " The constitutional Lumpagon and dictatorial na regimen ni Marcos, kag ibagsak ang Gobyerno;"66 and they had elicited a
proscription has thereby been manifested that thenceforth, the function of determining probable cause response from the people there who had raised their right hands and repeated the shouted "slogan."67
and issuing, on the basis thereof, warrants of arrest or search warrants, may be validly exercised only by Carbajosa in his turn declared that the demonstrators ** were shouting for trouble and ready to make
judges, this being evidenced by the elimination in the present Constitution of the phrase, "such other revolution with the use of arms."68
responsible officer as may be authorized by law" found in the counterpart provision of said 1973
Constitution,56 — who, aside from judges, might conduct preliminary investigations and issue warrants of Another affiant, Eduardo Flores, stated that the demonstrators were unruly, "tumultuously shouting
arrest or search warrants. seditious words against the government and shouting for revolution."69

As the law now stands, the mayor may no longer conduct preliminary investigation, the authority to do so The sworn statement of the only other witness, Leopoldo Villalon,70 is totally innocuous as far as proving
being limited under Section 2, Rule 1 1 2 of the Rules of Court to (1) provincial or city fiscals and their the elements of inciting to sedition is concerned.
assistants; (2) judges of the Municipal Trial Courts and Municipal Circuit Trial Courts; (3) national and
regional state prosecutors; and (d) such other officers as may be authorized by law.57 But only "the The evidence can not justify the action taken by the respondent Mayor and Judge. The Court thus
judge" may issue search and arrest warrants after due determination of probable cause.58 declares as sorely inadequate and mortally defective the avowed evidentiary foundation for Mayor
Lumayno's finding of probable cause respecting the commission by the petitioners of the crime of inciting
The petitioners' assault, on the other hand, upon the constitutionality of Article 141 of the Revised Penal to sedition. The affiants declarations in their sworn statements which might otherwise be pertinent to the
Code, defining and penalizing the felony of inciting to sedition, upon the claim that it was "borrowed" from offense, are generalities, mere conclusions of their, not positive averments of particular facts within their
the U.S. Sedition Act of 1798 which in turn has been struck down as inconsistent with the First personal knowledge. 'They do not Identify the specific persons supposed to have perpetrated the crime
Amendment of the American Constitution,59 is repelled by the Solicitor General's arguments.60 charged, except two. But even the Identification of these two is of no moment. For except as regards
Capt. Sanson, whose testimony, to repeat, is in any case ineffectual to prove the precise offense ascribed
In the first place, ** our law on inciting to sedition is not akin to the US Sedition Act of 1798 which was to the petitioners, there had been no searching interrogation by Mayor Lumayno of the witnesses as
imposed on the American colonies by their British ruler. With the success of the American revolution, the required by the Constitution. Hence, whatever credit could possibly have been accorded to the affidavit of
1798 Sedition Act naturally ceased to have effect as it would be utterly incongrous to punish those who Hoyo-A — which ventures to quote the exact words allegedly shouted by petitioners Arnaiz and
sought the overthrow of the British government in America. Gempesala, and an unidentified woman — was thereby effectively foreclosed.

xxx xxx xxx WHEREFORE, the writs of certiorari and prohibition are granted. The order of respondent Mayor
Lumayno issued on September 20, 1985 and the resolution promulgated by respondent Judge Ignalaga
xxx xxx xxx on October 11, 1985 are annulled and set aside, and the respondents are perpetually forbidden to
enforce or in any way implement the orders for the arrest of any of the petitioners. No costs.
To annul our law on sedition is to give license to those who seek the application of lawless methods in the
advancement of their political views. Our constitution surely does not contemplate this. Cynthia P. Nolasco vs Hon. Ernani Cruz Pano

Finally, the petitioners postulate that in the determination of the existence of probable cause from the FACTS: Aquilar-Roque and Nolasco were arrested by a Constabulary Security Group (CSG) at the
constitutional aspect, it is required that: "(1) The judge (or) officer must examine the ** witnesses intersection of Mayon Street, Quezon City On the same day, a searched was conducted. Ct. Col. Virgilio
personally; (2) The examination must be under oath; and (3) The examination must be reduced to writing Saldajeno; applied for search warrant from the respondent Hon. Ernani Cruz Pano, after a month of
in the form of searching questions and answers. (Marinas v. Siochio, 104 SCRA 403)." “round the clock” surveillance of the premises as a “suspected underground house of the CPP/NPA”,
particularly connected to MV Karagatan / Pena Andrea cases. The searching party seized 428 documents
In the light of these principles they contend: firstly, that the mayor's questioning of the witnesses was not and written materials, and additionally a portable typewriter and 2 wooden boxes. The City Fiscal
"searching" enough; and secondly, that the witnesses' testimony does not establish prima facie the information for violation of PD No. 33, Illegal Possession of Subversive Documents
commission of the felony of inciting to sedition. Petitioners contend that the Search Warrant is void because it is a general warrant since it does not
sufficiently describe with particularly the things subject of the search and seizure and that probable cause
The fact is that is shown by the record, questions about the material events were in truth propounded by had not been properly established for lack of searching questions propounded to the applicant’s witness.
Mayor Lumayno to the chief witness. Capt. Sanson;61 and no proof to the contrary has been submitted. Disputed Search Warrant: Documents, papers and other records of the communist party of the
The circumstance that the answers given by Capt. Sanson to the mayor's questions are closely reflective Philippines / New People’s Army and or the National Democratic Front, such as Minutes of the Party
of the contends of his affidavit should not come as a surprise and cannot, without more, be taken as Philippines..
debilitating or nullifying the interrogation.
ISSUE: Whether or not the search warrant issued was of general warrant and illegal?
The petitioner's second point is much more substantial and is decisive of the controversy. The petitioners
are correct in their claim that Capt. Sanson's testimony62 does not in truth contain any facts HELD: The search warrant is of General, thus, it was hereby annulled by set aside.
demonstrating the actuality of the crime of inciting to sedition, which is the crime for which arrest warrants
RATIONALE: The Search Warrant does not specify what the subversive books and instructions are; what II. THE ISSUE
are the manuals not otherwise available to the public certain to make them subversive or to enable them
to be used for the crime of rebellion. There is absent a definite guideline to the searching team as to what Did the judge properly lift the search warrants he issued earlier?
items might be lawfully seized thus giving the officers of the law discretion regarding what articles they
should seize as, in fact, taken also were a portable typewriter. III. THE RULING
Mere generalization will not suffice and odes not satisfy the requirements of probable cause upon which a
warrant may issue. [The Court DISMISSED the petition and AFFIRMED the questioned decision and resolution of the CA.]

Nolasco, et al vs. Paño YES, the judge properly lifted the search warrants he issued earlier.
G.R. No. L-69803
October 8, 1985 The lower court lifted the three (3) questioned search warrants in the absence of probable cause that the
Facts: private respondents violated P.D. 49. NBI agents who acted as witnesses during the application for
The present case was subject for resolution. search warrant did not have personal knowledge of the subject matter of their testimony, which was the
alleged commission of the offense of piracy by the private respondents. Only the petitioner’s counsel who
Supreme Court held in a criminal case that the arrest of the petitioners was illegal, annulling the decision was also a witness during the application stated that he had personal knowledge that the confiscated
of respondent Judge Paño, and that the seizure of the items by virtue of the warrant by the same tapes owned by the private respondents were pirated tapes taken from master tapes belonging to the
respondent judge are inadmissible as evidence in the Subversive Documents case. However the Court petitioner. The lower court lifted the warrants, declaring that the testimony of petitioner’s counsel did not
held that the items were to be retained in case it would be used as evidence in a separate criminal case have much credence because the master tapes of the allegedly pirated tapes were not shown to the court
pending before the Special Military Commission No.1, returning the rest which are determined irrelevant during the application.
by petitioner.
The presentation of the master tapes of the copyrighted films, from which the pirated films were allegedly
Petitioners questioned the portion of the decision regarding the retention of the properties seized. One of copied, was necessary for the validity of search warrants against those who have in their possession the
the petitioners also assailed the respondent’s claim that the search was incidental to her arrest for the pirated films. The petitioner's argument to the effect that the presentation of the master tapes at the time
crime of rebellion. of application may not be necessary as these would be merely evidentiary in nature and not determinative
of whether or not a probable cause exists to justify the issuance of the search warrants is not
Issue: meritorious. The court cannot presume that duplicate or copied tapes were necessarily reproduced from
Whether or not some of the properties seized may be introduced as evidence in a separate criminal case. master tapes that it owns.

Held: The essence of a copyright infringement is the similarity or at least substantial similarity of the purported
pirated works to the copyrighted work. Hence, the applicant must present to the court the copyrighted
The Court ruled the propriety of the declaration of the arrest and search as null and void. It was held that films to compare them with the purchased evidence of the video tapes allegedly pirated to determine
the warrant was one of a general warrant issued in gross violation of the constitutional mandate against whether the latter is an unauthorized reproduction of the former. This linkage of the copyrighted films to
unreasonable searches and seizures. The Bill of rights also orders the absolute exclusion of all illegally the pirated films must be established to satisfy the requirements of probable cause. Mere allegations as
obtained evidence: "Any evidence obtained in violation of this . . . section shall be inadmissible for any to the existence of the copyrighted films cannot serve as basis for the issuance of a search warrant.
purpose in any proceeding" (Sec. 4[2]).
All the articles thus seized fag under the exclusionary rule totally and unqualifiedly and cannot be used Columbia Pictures v CA 261 SCRA 144 (1996)
against any of the three petitioners, as held by the majority in the recent case of Galman vs.
Pamaran (G.R. Nos. 71208-09, August 30, 1985). Nature
Since the search was not an incident of an arrest as it was in fact made under a void general warrant, the : Petitions for review on certiorari of the decision of the CA.
seizure of documents could not be justified as an incident of an arrest. Ponente
: J. Vitug
20th Century Fox Film v. Court of Appeals, G.R. Nos. 76649-51, August 19, 1988 Facts:

DECISION 07 April 1998: NBI filed with the RTC of Pasig 3 applications for SW
(3rd Division) againstprivate respondent (Tube Video Enterprises – Edward C. Cham; BloomingRose Tape Center –
Ma. Jajorie T. Uy; Video Channel – Lydia Nabong)
GUTIERREZ, JR., J.: chargingthem with violations of Sec. 56 of PD 49 (Decree on the Protection of Intellectual Property) as
amended by PD 1988.
I. THE FACTS •
RTC Judge Austria consolidated the 3 applications and conducted a jointhearing where she made a
Petitioner 20th Century Fox Film Corporation sought the assistance of the NBI in conducting searches and personal examination of the applicant (NBI AgentReyes) and his witnesses.
seizures in connection with the NBI’s anti-film piracy campaign. Petitioner alleged that certain videotape •
outlets all over Metro Manila are engaged in the unauthorized sale and renting out of copyrighted films in Finding just and probable cause, Judge Austria issued the search warrants.
violation of PD No. 49 (the old Intellectual Property Law). •
Private Respondents filed their “Motion to Quash” the SW citing as groundsthat there was no probable
The NBI conducted surveillance and investigation of the outlets pinpointed by the petitioner and cause; the films in question are not protected byPD 1988 in that they were never registered in the
subsequently filed three (3) applications for search warrants against the video outlets owned by the National Library as acondition precedent to the availment of the protection; the Motion PictureAssociation
private respondents. The lower court issued the desired search warrants. The NBI, accompanied by the of America have not proven nor established their ownership overthe films; etc.
petitioner's agents, raided the video outlets and seized the items described in the three warrants. •
Judge Austria reversed her former stand initially finding probable cause forthe issuance of the search
Private respondents later filed a motion to lift the search warrants and release the seized properties, warrants and ordered their quashal:
which was granted by the lower court. Petitioner’s motion for reconsideration was denied by the lower o
court. The CA affirmed the trial court. Private complainants uncertain of their ownership over the titles;
o
Complainants did not comply with the requirement that master tapesshould be presented during The fact that the receipt issued was not in Samiano’s name nor was it in Sacriz’ name does not render the
the application for search warrants; issuance of the warrant void. No law or rule states that probable cause requires a specific kind of
o evidence. No formula or fixed rule for its determination exists. Probable cause is determined in the light of
Complainants failed to comply with the deposit and registrationrequirements of PD 49 as amended by PD conditions obtaining in a given situation.Thus, it was improper for the Court of Appeals to reverse the
1988. RTC’s findings simply because the sales receipt evidencing NBI Agent Samiano’s purchase of counterfeit
• goods is not in his name.
Judge Austria also ordered the return of the items seized by virtue of thewarrants.
• Prudente vs Dayrit Case
CA affirmed the quashal of the SWs. FACTS:
Issue The Chief of the Intelligence Special Action Division (ISAD) filed with the Regional Trial Court (RTC)
: WON the SWs were issued with probable cause. Manila, Judge Abelardo Dayrit, for the issuance of Search Warrant for violation of PD No. 1866 (Illegal
NORatio Possession of Firearm, etc). In the deposition of witness (P/Lt. Florencio C. Angeles), it was made
:BASIC REQUIREMENT for the validity of search warrants (in cases of this nature) is mentioned of “result of our continuous surveillance conducted for several days. We gathered information
the presentation of the master tapes of the copyrighted films from whichthe pirated films are supposed to from verified sources that the holders of said firearms and explosives as well as ammunitions aren’t
have been copied licensed to possess said firearms and ammunition. Further, the premises is a school and the holders of
(20 these firearms are not student who were not supposed to possess firearms, explosives and ammunitions.
th
Century Fox FilmCorp. vs. CA, 164 SCRA 655). The essence of a copyright infringement is the similarity Person to be searched in Nemesio Prudente at the Polytechnic University of the Philippines, Sta. Mesa,
or at least Sampaloc, Manila, has in his control or possession firearms, explosives hand grenades and ammunitions
substantialsimilarity of the purported pirated works to the copyrighted work. Hence, theapplicant must which are illegally possesses at the office of Department of Military Science and Tactics and at the office
present to the court the copyrighted films to compare them with thepurchased evidence of the video tapes of the President.
allegedly pirated to determine whether thelatter is an unauthorized reproduction of the former. This
linkage of the Petitioner moved to quash the Search Warrant. He claimed that:
copyrightedfilms to the pirated films must be established to satisfy the requirements of probable cause. Petitioners, had no personal knowledge of the facts
Mere allegations as to the existence of the copyrighted films cannotserve as basis for the issuance of a The examination of the said witness was not in form of searching questions and answers
search warrant.According to the CA, in which the SC concurs: Search warrant was a general warrant
It is not correct to say that "the basic fact" to be proven to establishprobable cause in the instant cases is Violation of Circular No. 19 of the Supreme Court in that the complainant failed to allege under oath that
not the "unauthorized transfer"of a motion picture that has been recorded but the "sale, lease, the issuance of the search warrant on a Saturday, urgent.
ordistribution of pirated video tapes of copyrighted films."In applying for the search warrants the NBI
charged violation of theentire provisions of Section 56 of P.D. No. 49 as amended by P.D.No.1988. This ISSUE:
included not only the sale, lease or distribution of piratedtapes but also the transfer or causing to be Whether or not the search and seizure was valid?
transferred of any soundrecording or motion picture or other audio visual work.But even assuming, as
appellants argue, that only the sale, lease, ordistribution of pirated video tapes is involved, the fact HELD:
remains thatthere is need to establish probable cause that the tapes being sold,leased or distributed are Search Warrant annulled and set aside.
pirated tapes, hence the issue reverts back tothe question of whether there was unauthorized transfer,
directly orindirectly, of a sound recording or motion picture or other audio visualwork that has been RATIONALE:
recorded. Valid search warrant to issue, there must be probable cause, which is to be determined personally by the
Petitions denied. Judge, after examination under oath and affirmation of the complainant, and that witnesses he may
produce and particularly describing the place to be searched and the persons and things to be seized.
MICROSOFT VS MAXICORP The probable cause must be in connection with one specific offense and the Judge must, before issuing
Search Warrant, personally examine in the form of searching questions and answers, In writing and under
438 SCRA 224 – Mercantile Law – Intellectual Property – Law on Copyright – Probable Cause in Issuing oath, the complainant and any witnesses he may produce, on facts personally known to them and attach
Search Warrant to the record their sworn statements together with any affidavits submitted.
In 1996, Dominador Samiano, Jr., an agent of the National Bureau of Investigation (NBI) conducted a
surveillance against Maxicorp, Inc. He observed that Microsoft Softwares (Windows Operating Systems) “Probable Cause” for a valid search warrant, has been defined as such facts and circumstances which
were being produced and packaged within the premises of Maxicorp. Samiano, together with a civilian would lead a reasonably discreet and prudent man to believe that an offense has been committed, and
witness (John Benedict Sacriz) then bought a computer unit from Maxicorp. The unit was pre-installed that objects sought in connection which the offense are in the place sought to be searched.
with a pirated copy of Windows. For their purchase, they were issued a receipt, however, the receipt was - This probable case must be shown to be personal knowledge and of the complainant and
in the name of a certain “Joel Diaz”. Subsequently, Samiano applied for a search warrant before the RTC. witnesses he may produce and not based on mere hearsay.
He brought with him Sacriz as witness. He also brought the computer unit they bought as evidence as
well as the receipt. He even added an additional witness (Felixberto Pante), a computer technician, who PARTICULARITY
showed the judge that the software in the computer unit bought by Samiano from Maxicorp was pirated. For violation of PD 1866 (Illegal Possession of Firearms, etc.) while the said decree punishes several
The RTC judge, convinced that there is a probable cause for a case of copyright infringement and unfair offenses, the alleged violation in this case was, qualified by the phrase illegal possession of firearms etc.
competition committed by Maxicorp, issued the corresponding warrant. Maxicorp assailed the legality of – – Reformed to ammunitions and explosives. In other words, the search warrant was issued for the
the warrant before the Court of Appeals. The Court of Appeals ruled in favor of Maxicorp and in its specific offense of illegal possession of firearms and explosives. Hence, the failure of the Search Warrant
decision it highlighted the fact that the receipt issued was not in Samiano’s or Sacriz’ name hence the to mention the particular provision of PD1-866 that was violated is not of such gravity as to call for the
proceeding in the trial court was infirm from the onset. invalidation of this case.
ISSUE: Whether or not the Court of Appeals is correct.
HELD: No. The testimonies of the two witnesses, coupled with the object and documentary evidence they Social Justice Society vs Dangerous Drugs Board
presented, are sufficient to establish the existence of probable cause. From what they have witnessed,
there is reason to believe that Maxicorp engaged in copyright infringement and unfair competition to the 570 SCRA 410 – Political Law – Qualifications of a Senator or a Congress Representative
prejudice of Microsoft. Both NBI Agent Samiano and Sacriz were clear and insistent that the counterfeit NOTE: This is consolidated with Laserna vs Dangerous Drugs Board (G.R. No. 158633) and Pimentel vs
software were not only displayed and sold within Maxicorp’s premises, they were also produced, COMELEC (G.R. No. 161658)
packaged and in some cases, installed there.
In 2002, Republic Act No. 9165 or the Comprehensive Dangerous Drugs Act of 2002 was implemented. SEQUESTRATION ORDER THROUGH A MOTION TO QUASH FILED WITH THE PCGG. BEING VOID,
Section 36 thereof requires mandatory drug testing of candidates for public office, students of secondary THE SANDIGANBAYAN HAS THE POWER TO STRIKE IT DOWN ON SIGHT.
and tertiary schools, officers and employees of public and private offices, and persons charged before the
prosecutor’s office with certain offenses. XXXXXXXXXXXXXXXXXXXXXXXXXXXXXXXXXXXXXXXX
In December 2003, COMELEC issued Resolution No. 6486, prescribing the rules and regulations on
the mandatory drug testing of candidates for public office in connection with the May 10, 2004 RULING OF THE COURT:
synchronized national and local elections. Aquilino Pimentel, Jr., a senator and a candidate for re-election
in the May elections, filed a Petition for Certiorari and Prohibition under Rule 65. In it, he seeks (1) to The Court’s Ruling
nullify Sec. 36(g) of RA 9165 and COMELEC Resolution No. 6486 dated December 23, 2003 for being Under Section 26, Article XVIII of the Constitution, an order of sequestration may only issue upon a
unconstitutional in that they impose a qualification for candidates for senators in addition to those already showing “of a prima facie case” that the properties are ill-gotten wealth under Executive Orders 1 and 2.
provided for in the 1987 Constitution; and (2) to enjoin the COMELEC from implementing Resolution No. [2][11] When a court nullifies an order of sequestration for having been issued without a prima facie case,
6486. the Court does not substitute its judgment for that of the PCGG but simply applies the law.[3][12]
According to Pimentel, the Constitution only prescribes a maximum of five (5)qualifications for one to be a In Bataan Shipyard & Engineering Co, Inc. v. PCGG,[4][13] the Court held that a prima facie factual
candidate for, elected to, and be a member of the Senate. He says that both the Congress and foundation that the properties sequestered are “ill-gotten wealth” is required. The power to determine the
COMELEC, by requiring, via RA 9165 and Resolution No. 6486, a senatorial aspirant, among other existence of a prima facie case has been vested in the PCGG as an incident to its investigatory powers.
candidates, to undergo a mandatory drug test, create an additional qualification that all candidates for The two-commissioner rule is obviously intended to assure a collegial determination of such fact.[5][14]
senator must first be certified as drug free. He adds that there is no provision in the Constitution Here, it is clear that the PCGG did not make a prior determination of the existence of a prima facie case
authorizing the Congress or COMELEC to expand the qualification requirements of candidates for that would warrant the sequestration of the Olot Resthouse. The Republic presented no evidence before
senator. the Sandiganbayan that shows differently. Nor did the Republic demonstrate that the two PCGG
ISSUE: Whether or not Sec 36 of RA 9165 and Resolution 6486 are constitutional. representatives were given the quasi-judicial authority to receive and consider evidence that would
HELD: No. Pimentel’s contention is valid. Accordingly, Sec. 36 of RA 9165 is unconstitutional. It is basic warrant such a prima facie finding.
that if a law or an administrative rule violates any norm of the Constitution, that issuance is null and void Parenthetically, the Republic’s supposed evidence does not show how the Marcoses acquired the
and has no effect. The Constitution is the basic law to which all laws must conform; no act shall be valid if sequestered property, what makes it “ill-gotten wealth,” and how former President Marcos intervened in
it conflicts with the Constitution. In the discharge of their defined functions, the three departments of its acquisition. Taking the foregoing view, the resolution of the issue surrounding the character of the
government have no choice but to yield obedience to the commands of the Constitution. Whatever limits property sequestered – whether or not it could prima facie be considered ill-gotten – should be necessary.
it imposes must be observed. The issue in this case is not new. The facts are substantially identical to those in the case of
The provision “[n]o person elected to any public office shall enter upon the duties of his office until he has Republic v. Sandiganbayan (Dio Island Resort, Inc.).[6][15] There, the same Atty. Ramirez issued a
undergonemandatory drug test” is not tenable as it enlarges the qualifications. COMELEC cannot, in the sequestration order on April 14, 1986 against Dio Island Resort, Inc. and all its assets and properties
guise of enforcing and administering election laws or promulgating rules and regulations to implement which were thought to be part of the Marcoses’ ill-gotten wealth. Alerted by a challenge to his action, the
Sec. 36, validly imposequalifications on candidates for senator in addition to what the Constitution PCGG passed a resolution “to confirm, ratify and adopt as its own all the Writs of Sequestration” that
prescribes. If Congress cannot require a candidate for senator to meet such additional qualification, the Attys. Ramirez and Abella issued “to remove any doubt as to the validity and enforceability” of their writs.
COMELEC, to be sure, is also without such power. The right of a citizen in the democratic process of Still, the Court struck them down as void:
election should not be defeated by unwarranted impositions of requirement not otherwise specified in the x x x It is indubitable that under no circumstances can a sequestration or freeze order be validly issued
Constitution. by one not a Commissioner of the PCGG.

PP VS MARCOS The invalidity of the sequestration order was made more apparent by the fact that Atty. Ramirez did not
even have any specific authority to act on behalf of the Commission at the time he issued the said
FACTS: sequestration order. x x x

ON 18 MARCH 1986, ATTY. RAMIREZ AND ATTY. ABELLA, PCGG AGENTS, ISSUED A xxxx
SEQUESTRATION ORDER AGAINST THE RESTHOUS THE SOLE ISSUE PRESENTED IS WHETHER
OR NOT THE MARCH 18, 1986 SEQUESTRATION ORDER AGAINST PROPERTIES OF IMELDA IN Even assuming arguendo that Atty. Ramirez had been given prior authority by the PCGG to place Dio
LEYTE INCLUDING THE RESTHOUSE AT OLOT. THEIR ORDER WAS NOT SIGNED BY ANY PCGG Island Resort under sequestration, nevertheless, the sequestration order he issued is still void since
COMMISSIONERS. PCGG may not delegate its authority to sequester to its representatives and subordinates, and any such
delegation is invalid and ineffective.
ISSUE:
Under Executive Order Nos. 1 and 2, PCGG is the sole entity primarily charged with the responsibility of
IS THEIR ORDER VALID? recovering ill-gotten wealth. x x x The power to sequester, therefore, carries with it the corollary duty to
make a preliminary determination of whether there is a reasonable basis for sequestering a property
RULING: alleged to be ill-gotten. After a careful evaluation of the evidence adduced, the PCGG clearly has to use
its own judgment in determining the existence of a prima facie case.
NO. JUDICIAL OR QUASI-JUDICIAL POWERS MAY NOT BE DELEGATED. IN PCGG V. JUDGE PEÑA,
[1][17] THE COURT HELD THAT THE POWERS, FUNCTIONS AND DUTIES OF THE PCGG AMOUNT xxxx
TO THE EXERCISE OF QUASI-JUDICIAL FUNCTIONS, AND THE EXERCISE OF SUCH FUNCTIONS
CANNOT BE DELEGATED BY THE COMMISSION TO ITS REPRESENTATIVES OR SUBORDINATES The absence of a prior determination by the PCGG of a prima facie basis for the sequestration order is,
OR TASK FORCES BECAUSE OF THE WELL ESTABLISHED PRINCIPLE THAT JUDICIAL OR QUASI- unavoidably, a fatal defect which rendered the sequestration of respondent corporation and its properties
JUDICIAL POWERS MAY NOT BE DELEGATED. void ab initio. Being void ab initio, it is deemed non-existent, as though it had never been issued, x x x.[7]
[16]
PETITIONER REPUBLIC ARGUES THAT MRS. MARCOS SHOULD BE DEEMED ESTOPPED FROM
QUESTIONING THE SEQUESTRATION OF HER OLOT RESTHOUSE BY HER ACTIONS IN REGARD The Court is maintaining its above ruling in this case.
TO THE SAME. BUT A VOID ORDER PRODUCES NO EFFECT AND CANNOT BE VALIDATED Although the two PCGG lawyers issued the sequestration order in this case on March 18, 1986, before
UNDER THE DOCTRINE OF ESTOPPEL. FOR THE SAME REASON, THE COURT CANNOT ACCEPT the passage of Sec. 3 of the PCGG Rules, such consideration is immaterial following our above ruling.
PETITIONER’S VIEW THAT MRS. MARCOS SHOULD HAVE FIRST SOUGHT THE LIFTING OF THE In PCGG v. Judge Peña,[8][17] the Court held that the powers, functions and duties of the PCGG
amount to the exercise of quasi-judicial functions, and the exercise of such functions cannot be delegated
by the Commission to its representatives or subordinates or task forces because of the well established auxiliary stamps, and using crown caps not produced by the company, which is in violation of the
principle that judicial or quasi-judicial powers may not be delegated. provisions of Articles 188 and 189 of the Revised Penal Code.
It is the Republic’s theory of course that Commissioner Daza’s letter, directing Attys. Ramirez and Abella ... and therefore has reason enough to believe that a search warrant should be issued to enable
to search and sequester all properties, documents, money and other assets of respondents, should be the undersigned to take possession and control and bring it to this Court said stock, as evidence in
considered as the writ of sequestration while the order issued by Attys. Ramirez and Abella should be the above case under investigation.
treated merely as an implementing order. City Judge Patricio Perez of Baguio City, acting on the aforesaid application issued on June 15,
But the letter did not have the tenor of a sequestration order covering specific properties that the lawyers 1967, Search Warrant No. 459, under the following terms:
were ordered to seize and hold for the PCGG. Actually, that letter is of the same kind issued to Attys.
Whereas, after examination under oath of Supervising Agent, NBI Jose Vicente and Cesar de
Ramirez and Abella in Dio Island Resort. Consequently, there is no reason to depart from the Court’s
ruling in the latter case where it said:
Leon, both of the NBI Regional Office, at Dagupan City, this Court finds that there is probable
The invalidity of the sequestration order was made more apparent by the fact that Atty. Ramirez did not cause to believe that the owner and/or manager of the Suntory Grocery, located at No. 36 Rajah
even have any specific authority to act on behalf of the Commission at the time he issued the said Soliman St., Baguio City, is in possession such stock of La Tondena product, San Miguel Gin,
sequestration order. Thus, the respondent Court noted: which is adulterated, bearing auxiliary stamps which is tampered and possession of falsified or fake
crown caps, which is now under investigation by the National Bureau of Investigation, in this City.
“Contrary to plaintiff’s representation, nothing exists to support its contention that the Task Force had Therefore, you are hereby commanded during the day only to make an immediate search on the
been given prior authority to place DIO under PCGG control. On the contrary, as the text of the above premises of the store and/or grocery owned by the Manager and/or proprietor of the Suntory
letters clearly show, Attys. Jose Tan Ramirez and Ben Abella, had acted on broad and non-specific Grocery which is located in No. 36 Rajah Soliman, of this City and if you should find the same to
powers: ‘By authority of the commission and the powers vested in it. x x x.’”[9][18] bring it forthwith before me in the City Court of Baguio to be dealt with as the law directs.
Armed with said search warrant, the NBI agents, on June 19, 1967, searched the premises of the
Petitioner Republic argues that Mrs. Marcos should be deemed estopped from questioning the Suntory Grocery located at 36 Rajah Soliman St., Baguio City, owned and managed by private
sequestration of her Olot Resthouse by her actions in regard to the same. But a void order produces no respondent Yu Cua Sio. During the search conducted in the presence of private respondent and
effect and cannot be validated under the doctrine of estoppel. For the same reason, the Court cannot his wife, the NBI agents seized the following articles which were properly inventoried and
accept petitioner’s view that Mrs. Marcos should have first sought the lifting of the sequestration order receipted:
through a motion to quash filed with the PCGG. Being void, the Sandiganbayan has the power to strike it
(1) One (1) bundle consisting of rubber siphon, trainer and funnel;
down on sight.
(2) One (1) galvanized tank, 15" in diameter, 2 feet in height;
Besides, the lifting of the sequestration order will not necessarily be fatal to the main case since it does
not follow from such lifting that the sequestered properties are not ill-gotten wealth. Such lifting simply (3) One (1) mounted crown cap sealer with accessories;
means that the government may not act as conservator or may not exercise administrative or (4) Six (6) cases of San Miguel Gin, round bottles, fined with suspected adulterated Gin and
housekeeping powers over the property.[10][19] Indeed, the Republic can be protected by a notice of lis bearing BIR stamps dated 6 April 1967;
pendens. (5) Two(2)cases of San Miguel Gin empty bottles;
WHEREFORE, the Court DISMISSES the petition for lack of merit and AFFIRMS the challenged (6) One (1) case containing ten (10) San Miguel Gin (adulterated) and 14 empty bottles, without
resolutions of the Fourth Division of the Sandiganbayan dated February 28, 2002 and August 28, 2002 in stamps;
Civil Case 0002, which granted respondent Imelda R. Marcos’ Motion to Quash the March 18, 1986 (7) One (1) bundle consisting of two (2) bottles believed to be containing genuine San Miguel Gin
Sequestration Order covering the Olot Resthouse. which was marked as standard; and
Further, the Court DIRECTS the Register of Deeds of Leyte to immediately annotate a notice of lis (8) One (1) paper bag containing 122 auxiliary stamps dated 6 April 1967, without perforation and
pendens on the certificate of title of the Olot Resthouse with respect to the Republic of the Philippines’ bearing successive serial numbers starting from 43,000,000.
claim over the same in Civil Case 0002 of the Sandiganbayan. Private respondent Yu Cua Sio filed a motion to quash Search Warrant No. 459 with the City Court
No pronouncement as to costs. of Baguio. In an Order dated February 3, 1969, the inferior court ordered the NBI agents who
SO ORDERED. seized and confiscated the various articles from the store of Yu Cua Sio to return the same
immediately to him.
On appeal, the Court of First Instance of Baguio City, reversed the decision of the City Court and
G.R. No. , 117 SCRA 999
sustained the validity of the questioned search warrant. However, upon motion for reconsideration
Republic of the Philippines
filed by private respondent Yu Cua Sio, respondent Judge, on October 13, 1969, reconsidered his
SUPREME COURT
decision and declared the issuance of Search Warrant No. 459 as contrary to law and, forthwith,
Manila
ordered the NBI agents to deliver and return the articles seized by virtue of the search warrant to
EN BANC
private respondent Yu Cua Sio.
DECISION
The only legal issue posed for resolution in this case is the validity of Search Warrant No. 459
September 30, 1982
issued by City Judge Patricio Perez of Baguio City which respondent Judge declared as null and
G.R. No. , ,
void in his order dated October 13, 1969 on the grounds that (1) the same was issued "for more
vs.
than one specific offense in violation of Section 3, Rule 126 of the New Rules of Court which states
,.
'no search warrant shall issue for more than one specific offense' "; and, (2) that the search warrant
, J.:
was issued to fish for evidence.
Petitioner, through the Solicitor General, filed this petition to review on certiorari the order dated
Petitioner claims that the lower court erred (1) when it held that the questioned search warrant
October 13, 1969 of respondent Judge of the Court of First Instance of Baguio and Benguet
violates the provisions of Section 3, Rule 126 of the New Rules of Court; (2) in holding that the
Province, declaring the issuance by the City Court of Search Warrant No. 459, as contrary to law
search warrant in question was issued to fish for evidence; and (3) in declaring Search Warrant No.
and ordering the National Bureau of Investigation (NBI) agents and any person in possession of
459 as contrary to law and in ordering the return of the articles seized by virtue of said search
the articles seized by virtue of the search warrant to deliver and return the same to Yu Cua Sio,
warrant to respondent Yu Cua Sio.
owner and/or manager of the Suntory Grocery.
We find merit in the petition. The search warrant issued by the City Court did not mention any
On June 15, 1969, NBI Supervising Agent Jose Vicente filed an application for the issuance of a
specific offense deemed to have been violated by respondent Yu Cua Sio. It is in the application
search warrant with the City Court of Baguio City which provides, among others, the following:
filed by the NBI agents which states that the owner and/or manager of the Suntory Grocery has in
That he has been reliably informed and verily believes that the owner and/or manager of the
his possession and control stocks of San Miguel Gin which are adulterated and therefore, violative
Suntory Grocery, of 36 Rajah Soliman St., Baguio City has in his possession and control stocks of
of the provisions of Articles 188 and 189 of the Revised Penal Code. These articles of the Revised
San Miguel Gin, product of the La Tonde?a, Inc., Manila, which is adulterated, bearing fake
Penal Code are entitled: "Substituting and Altering Trademarks, Tradenames, or Service marks" be the technical common-law meaning of the word "felony", which isused in paragraph 2 of sec. 96 above
and "Unfair Competition and Fraudulent Registration of Trademark and Tradename," respectively. quoted, the Court believes it would be the height of absurdity tohold, upon technical grounds, that a
As aptly stated by the Solicitor General, "the specific acts defining said offenses and mentioned in search warrant is illegal which is issued to search for and seizeproperty the very possession of which is
said articles are closely allied to each other that in a sense, the punishable acts defined in one of forbidden by law and constitutes a crime. Opium is such property.- Search-warrants have heretofore
them can be considered as including, or necessarily included in the other." been allowed to search for… material so kept as to endanger the publicsafety.- A search warrant may be
There is no merit also in the pronouncement by respondent Judge that the search warrant was likened to a warrant of arrest. The issuance of both is restricted by the sameprovision of the Jones Law
issued to fish for evidence just because the application for search warrant states that its purpose is (sec. 3) which is as follows:
“That no warrant shall issue but upon probablecause, supported by oath or affirmation, and particularly
"to take possession and control of the articles to be used as evidence in the above case under
describing the place to be searched and the person or thing to be seized.”
investigation." - In the present case there was an irregularity in the issuance of the search warrant in question in
The search warrant as issued mentions that respondent has in his possession and control stocks that the judge did not first examine the complainant or any witnesses under oath. But the property sought
of La Tonde?a product which are adulterated, with fake auxiliary stamps, and are using crown caps to besearched for and seized having been actually found in the place described by the complainant,
which are not produced by the company. The articles seized in the premises of respondent Yu Cua reasoningby analogy from the case of an improper arrest, we are of the opinion that that irregularity is
Sio show that he was in possession of these articles mentioned in the application for search not sufficientcause for ordering the return of the opium found and seized under said warrant, to the
warrant and in the search warrant itself. Possession of said fake stamps is illegal and the same petitioners, andexonerating the latter.
should not be returned to respondent Yu Cua Sio.
ACCORDINGLY, the order of respondent Judge, dated October 13, 1969, is SET ASIDE, and While it is true that a warrant is good for 10 days after the date of issuance, this cannot be interpreted
private respondent Yu Cua Sio is hereby ordered to return the articles seized if they had been tomean that a search warrant can be used every day for 10 days, and for a different purpose each day.
delivered to him by the NBI agents. Thiswould be absurd.-It appears from the oral evidence adduced during the hearing of the petitioners'
SO ORDERED. motion in the court belowthat the search for opium, the property mentioned in the warrant, was not
completed on April 30th; it wasinterrupted by the necessity to ascertain who the owner of the bodega on
UY KHEYTIN VS VILLAREAL the ground-floor was, becausethe petitioner Uy Kheytin falsely disclaimed ownership thereof. In other
Facts: words, the search of May 1st wasnot made "for a different purpose," nor could it be considered "another
-On April 30, 1919, one Ramon Gayanilo, corporal of the Philippine Constabulary, presented to the judge search," but was really acontinuation of the search begun on April 30th. This is shown by the fact that
of the Court of First Instance of Iloilo an application for search warrant, the said Ramon Gayanilo stating during the interval betweenthe two searches the premises in question were guarded by Constabulary
in hisapplication; "That in the house of Chino Uy Kheytin, Sto. Niño St., No. 20, Iloilo, under the writing soldiers, and the petitionerswere made to understand on April 30th that the authorities were not yet
desk inhis store, there is kept a certain amount of opium."- Armed with that search warrant, the through with the search andwould continue the same as soon as they found out that the bodega was also
respondent M. S. Torralba, on the same day (April 30th) searchedthe house of the petitioner Uy Kheytin occupied by the petitionerUy Kheytin.(Contention # 3)- In order to comply with the constitutional
and found therein 60 small cans of opium. They wanted to searchalso the bodega on the ground-floor of provisions regulating the issuance of search warrants, theproperty to be seized under a warrant must be
the house, but Uy Kheytin positively denied that it was his or thathe rented it. Lt. Torralba wanted to particularly described therein and no other property can betaken thereunder.- That the officers of the law
be sure, and for this reason, he placed a guard in the premises to seethat nothing was removed believed that the books, papers, etc., which they seized might be used asevidence against the petitioners
therefrom, and then went away to find out who the owner of the bodega was. The next morning he herein a criminal action against them for a violation of the Opium Law, isno reason or justification under
learned from the owner of the house, one Segovia, of the town of Molo, that theChinaman Uy Kheytin the law for the seizure: First, because they were not "particularlydescribed" or even mentioned in the
was the one who was renting the bodega. Thereupon Lt. Torralba and hissubordinates resumed search warrant; second, because, even if they had been mentioned inthe search warrant, they could not
the search and then and there found and seized articles which were all withconnection to the using be legally seized, for a search warrant cannot be used for the purposeof obtaining evidence; and third,
of opium.- A criminal complaint was filed in the court of the justice of the peace of Iloilo against all the because to compel a person to produce his private papers to be used inevidence against him would be
petitionersherein, charging them with a violation of the Opium Law. They were duly arrested. equivalent to compelling him to be a witness against himself.

-Defendants urged: (1) that the search warrant of April 30 was illegal because the requisites prescribed PEOPLE OF THE PHILIPPINES vs. EXALA
bythe General Orders No. 58 had not been complied with in its issuance BELLOSILLO, J.:
(specifically (a) because it was not issued upon either of the grounds mentioned in section 96 of General
Orders No. 58, and (b) because the judge who issued it did not determine the probable cause by I. THE FACTS
examining witnesses under oath)
; A private jeep driven by accused-appellant Bocalan was stopped at a police checkpoint in Cavite City for
routine inspection. With Bocalan were his co-accused Fernandez and Exala. Pfc. Galang, a member of
(2) that thesearches and seizures made on May 1st had been made without any semblance of authority the inspection team, went near the jeep and asked the occupants if there were firearms inside. They
and henceillegal; and (3) that the seizure of the defendants' books and letters was a violation of the answered in the negative. Pfc. Galang proceeded to inspect the vehicle by beaming a flashlight inside. He
provisions of the Jones Law providing that no person shall be compelled to testify against himself, and then noticed a black leather bag measuring about 1 foot wide and 2 feet long with its sides bulging. When
protecting himagainst unreasonable searches and seizures. he asked what it contained, there was deadening silence from the 3 accused. Nobody answered. Instead,
Issue: they suddenly became fidgety. Suspicious, Pfc. Galang ordered the bag opened, which was found out to
WON the defendants’ positions are with merit. contain marijuana. The 3 accused were thereafter prosecuted and convicted of illegal transportation of
Ruling (s): marijuana. Accused Bocalan appealed and questioned the legality of the admission of the marijuana as
1. That although in the issuance of the search warrant in question the judge did not comply with evidence against him since it was seized without a valid search warrant.
therequirements of section 98 of General Orders No. 58, the petitioners are not entitled to the return of
theopium and its paraphernalia which were found and seized under said warrant, and much less are II. THE ISSUE
theyentitled to be exonerated because of such omission of the judge.2. That the search made on May 1st
was a continuation of the search begun on the previous day, and,therefore, did not require another search Was the marijuana seized without warrant during the checkpoint admissible in evidence against the
warrant.3. That the seizure of the petitioner's books, letters, telegrams, and other articles which have no accused?
inherentrelation with opium and the possession of which is not forbidden by law, was illegal and in
violation of thepetitioners' constitutional rights. III. THE RULING
RD:
(Contention # 1)-SEC. 96. of General Orders No. 58 provide: “ It (a search warrant) may be issued upon [The 1st Division voted 3-1 to AFFIRM the conviction of the accused. JusticesGriño-Aquino and
either of thefollowing grounds: 1. When the property was stolen or embezzled. ; 2. When it was used or Quiason concurred with Justice Bellosillo’s ponencia. Justice Cruz, by his lonesome, dissented from the
when the intentexists to use it as the means of committing a felony.”-Suffice it to say that, whatever may majority.]
The Court held that Bocalan is deemed to have waived his objection to the admission of the seized Pp vs. ZaspaFacts:
marijuana because he neither raised this issue before the trial court nor objected to the admissibility of On April 29, 1994, Zaspa and a companion were caught by the policemen carrying dried marijuana
the marijuana when it was offered in evidence. leavesinside a bag. They were arrested thereafter without a warrant and brought to the police station for
investigation.
And even assuming that there was no such waiver, the Court held that still Bocalan’s contention deserves Issue:
scant consideration because there are instances where search and seizure can be made without W/N the warrantless arrest is valid?
necessarily being preceded by an arrest. An illustration would be the “stop-and-search” without a warrant Held:
at military or police checkpoints, the constitutionality of which has already been upheld by this Yes. Sec. 5, Rule 113 of the Rules of Court authorizes an arrest without a warrant when the person to be
Court [inValmonte vs. De Villa]. Vehicles are generally allowed to pass through these checkpoints after arrestedhas committed a crime, is actually committing or about to commit a crime in the presence of the
a routine inspection and answering a few questions. If vehicles are stopped and extensively searched it is police officers. Asthe appellants were found to be in possession of the prohibited drug at the time of their
because of some probable cause which justifies a reasonable belief of those manning the checkpoints arrest, the same isadmissible in evidence.
that either the motorist is a law-offender or thecontents of the vehicle are or have been instruments in the
commission of an offense. PEOPLE OF THE PHILIPPINES, plaintiff-appellee,
vs.
According to the Court, lest it be misunderstood, the foregoing doctrine is not intended to do away with ROLANDO ZASPA and JULIUS GALVAN, accused-appellants.
the general rule that no person shall be subjected to search of his person, personal effects and DECISION
belongings, or his residence except of virtue of a search warrant or on the occasion of a lawful VITUG, J.:
arrest. This case, however, is an incident to or an offshoot of a lawful “stop-and-search” at a military or Elevated and certified by the Court of Appeals to this Court for review is the decision, dated 08 January
police checkpoint. 1996, of the Regional Trial Court of Mati, Davao Oriental, Branch 5, penned by Judge Ricardo M. Berba.
The case, docketed Criminal Case No. 2621 before the trial court, has charged Rolando Zaspa, a.k.a.
The checkpoint in the instant case was established in line with “Operational Bakal,” the main object of "Tata," and Julius Galvan with violation of Section 8 of Republic Act No. 6425, otherwise also known as
which was to search for unlicensed firearms and other prohibited items in the possession of unauthorized the Dangerous Drugs Act of 1972, as amended.
persons passing through it. When the jeep carrying the contraband passed through the checkpoint, it was The case originated from an Information, dated 13 October 1994, which read:
flagged down and the occupants were asked routine questions. In the course thereof, Pfc. Galang noticed "That on or about April 29, 1994, in the Municipality of Tarragona, Province of Davao Oriental, Philippines
a black leather bag the sides of which were bulging. He asked what the contents of the bag were. None of and within the jurisdiction of this Honorable Court, the abovenamed accused, with intent to use did then
the accused answered. At that moment, the demeanor of the accused changed; they became and there wilfully, unlawfully, feloniously own and possess five point six (5.6) kilos of marijuana dried
suspiciously quiet and nervous as if they were concealing something from Pfc. Galang. The accused leaves with stalks, a prohibited dangerous drugs, without proper license or permit from the authorities." 1
clearly appeared to be in abject fear of being discovered. Such peculiar apprehensiveness if not Upon arraignment, both accused pled "not guilty" to the charge.
restrained reaction of the accused, which did not appear normal, provided the probable cause justifying a Culled from the findings of the trial judge, as well as the decision of the Court of Appeals, 2 promulgated on
more extensive search that led to the opening of the bag and the discovery of the prohibited stuff. 09 November 1998 and penned by Associate Justice Delilah Vidallon-Magtolis, the facts could be
gathered, thusly:
[NOTE: Incidentally, one of the co-counsels for accused-appellant Bocalan in his appeal to the Supreme At about two o'clock in the morning of 29 April 1994, Chief of Police Rosauro Francisco of Tarragona,
Court was then-Atty. and now Supreme Court Senior Associate Justice Presbitero Velasco Jr.] Davao Oriental, received a tip from a police informer that Rolando Zaspa and a companion were bringing
dried marijuana leaves bound for Mati, somewhere at Crossing Banhawan, Tarragona, Davao Oriental.
The police chief promptly organized and dispatched to the area a team composed of SPO2 Honorio
CRUZ, J., dissenting: Carasca (the team leader), PO1 Letecio Rafael and SPO1 Cesar Travelegio. The group immediately
proceeded to Crossing Banhawan, arriving thereat at about five o'clock in the morning. There, the team
Justice Cruz maintained the proposition in his dissent in Valmonte vs. De Villathat checkpoints and the saw Zaspa and his companion standing by the side of the road with a big black "loalde" bag in front of
searches and seizures incident thereto are unconstitutional. InPeople vs. Exala, he expounded on this them. Just as SPO2 Carasca and PO1 Rafael, who were both in uniform, proceeded to approach the two
thesis: men, Zaspa tried to flee. He was intercepted by the policemen. Zaspa claimed that the contents of the
bag did not belong to them. When the bag was opened, Zaspa told the policemen that the dried
I am opposed to checkpoints as regular police measures aimed at reducing criminality in general. I do not marijuana leaves were owned by one Bito Mangandan. Zaspa and his companion, who turned out to be
agree that in the interest of peace and order, any or every vehicle may be stopped at any time by the Julius Galvan, were arrested and brought to the Tarragona police station for investigation. Samples of the
authorities and searched without warrant on the chance that it may be carrying prohibited articles. That leaves taken from the bag were sent to the PNP Crime Laboratory in Ecoland, Davao City, where the
possibility is not the probable cause envisioned in the Bill of Rights. specimen were tested and confirmed to be marijuana leaves. Chemistry Report No. 035-94, submitted by
Police Senior Inspector Noem; Austero, a forensic expert, contained the following findings:
In the case of the ordinary checkpoint, there is not even any suspicion to justify the search. The search is "Qualitative examination conducted on the above-mentioned specimen gave positive result to the tests for
made as a matter of course, either of all vehicles or at random. There is no showing that a crime is about marijuana, a prohibited drug."3
to be committed, is actually being committed, or has just been committed and the searching officer has Zaspa, taking the witness stand in his defense, testified that sometime in January 1994 he was hired by
personal knowledge that the person being searched or arrested is the culprit. Maturino Masanguid to cut an Antipolo round timber for the amount of P5,000.00. He was paid P3,500.00
and was about to get the balance of P1,500.00 on 29 April 1994 when the incident transpired. He stated
I will concede that checkpoints may be established at borders of states or at ‘constructive borders’ near that he was walking towards Barrio Sambarangay when an armed man in civilian outfit pointed a gun at
the boundary for the purpose of preventing violations of immigration and customs laws. But in the interior him and proceeded to examine the brown bag he was holding. He was thereafter dragged to the side of
of the territory, the requirements of a valid search and seizure must be strictly observed. The only the road and questioned whether a black bag also belonged to him. He denied either ownership or
permissible exemption is where a crime like a bank robbery has just been committed or a jailbreak has possession of the bag. He was brought to the police station in Tarragona with another man whom he later
just occurred, and the authorities have to seal off all possible avenues of escape in the area. In all other learned to be Julius Galvan. At the police station, a certain Francisco slapped him and made him and
cases, I submit that the checkpoint should not be allowed. Galvan crawl on the floor. The man also tried to smash the face of Galvan. He was detained in Tarragona
Municipal jail from 29 April until he was brought, on 02 May 1994, to the PC Barracks at Menzi, Mati,
xxx. [W]e cannot retroactively validate an illegal search on the justification that, after all, the Davao Oriental.
articles seized are illegal. That is putting the cart before the horse. I would rather see some Galvan corroborated the testimony of Zaspa. He asserted that on 12 April 1994, he was contracted by
criminals go unpunished now and then than agree to the Bill of Rights being systematically Algin Divinagracia to spray their mango trees in Banhawan. The first spray was made on 15 April; he
ignored in the oppressive checkpoint. Respect for the Constitution is more important than securing a returned to Mati on the same day. The second spray was made on 28 April but, unable to finish the work
conviction based on a violation of the rights of the accused. (Emphasis supplied.) that day, he stayed overnight with Divinagracia. At about four o'clock in the morning, he was on his way to
Banhawan crossing when he was stopped by a man holding a gun. He was brought to a place where noted that the appellant Zaspa failed to present any evidence to show that he collected the sum of
some police officers were questioning a man about the ownership of a black bag. The two were brought P3,500.00, or that he was to collect the balance of P1,500.00 from the witness, Mr. Maturino Masanguid.
to the police station where they were investigated and detained until they were brought the following day In the same manner, the appellant Galvan failed to present the receipts for the medicine he purportedly
to the P.C. Barracks at Mati. bought for the spraying of the mango trees (TSN, 7/19/95, p. 22). Third, if the appellants were really
After trial, the court a quo found the two accused guilty of the crime with which they were charged; the maltreated in order to confess to the crime charged, then why is it that they failed to submit to a medical
trial court held: examination or treatment? (TSN, 6/8/95, p. 48 and TSN, 7/19/95, p. 36). Why is it that the appellants filed
"WHEREFORE, the Court finds the accused Rolando Zaspa alias `Tata' and Julius Galvan both GUILTY a case before the Ombudsman and the Commission on Human Rights only after almost a year from the
beyond reasonable doubt of violation of Section 8 in relation to Section 20 of the Republic Act 6425, as time of the incident (Exhibit `1' and `2')? In any case, there is nothing on record which indicates that the
amended by R.A. 7659, and hereby imposes upon each of them the penalty of RECLUSION PERPETUA police operatives were actuated by improper motive against the appellants. Credence can be given to the
and to pay a fine of P500,000.00, with the accessory penalties provided by law, and to pay the costs of narration of the incident by the prosecution witnesses, who as police officers are presumed to have
the proceedings. performed their duties in a regular manner in the absence of proof to the contrary ( People vs. Bautista,
"The marijuana leaves subject hereof (Exhs. `D,' `D-1' and `D-2') are hereby ordered turned over to the 227 SCRA 152). Lastly, as the seized items were positive for marijuana, then, the corpus delicti of the
Dangerous Drugs Board, thru the National Bureau of Investigation (NBI), for disposition in accordance crime had been fully proved with certainty and conclusiveness (People vs. Simon, 234 SCRA 555).
with law. "In sum, the guilt of the accused-appellants has been established by proof beyond reasonable doubt;
"SO ORDERED."4 hence, the affirmance of the appealed judgment is in order. However, considering that the applicable
Zaspa and Galvan appealed their conviction, albeit the penalty imposed, to the Court of Appeals for penalty is reclusion perpetua, the second paragraph of Section 13, Rule 124 of the Rules of Court shall
review. The appellate court upheld the conviction; it said: apply.
"Regarding the first issue, the appellee asseverates that there is entirely no merit in appellants' claim that "WHEREFORE, the judgment appealed from is AFFIRMED in toto. Let this case
they were not in possession of marijuana when the police authorities apprehended them at Banhawan be CERTIFIED and ELEVATEDto the Honorable Supreme Court for review.
5
Crossing. The testimonial evidence submitted by the prosecution clearly pointed out that the two "SO ORDERED."
appellants were standing by the side of the road, with the black bag containing the prohibited marijuana Although the appeal from the decision of the trial court imposing reclusion perpetua should have been
just 1/2 foot away. The absence of any other person within the vicinity indicates that the contraband directly appealed to this Court, considering the penalty involved, the Court has decided to ignore this
belonged to the appellants and to no one else. Now, the appellants could not sufficiently explain the breach of technicality and to nevertheless consider the appeal and evaluate the case.
presence of the bag in their possession. Neither could they explain why the police would `plant' the same The testimony given by the witnesses for the prosecution and that of the defense are diametrically
as evidence. Likewise, they did not substantiate the alleged `maltreatment' suffered in the hands of the opposed to each other on almost every point. In resolving such a conflict, so dealing as it does on the
law enforcers. Moreover, no improper motive was attributed to the police as to why they would testify credibility of the witnesses, the Court relies heavily on findings of the trial court being in the best position,
falsely against the appellants -- if such was the fact. certainly more than the appellate court, in making that judgment. Thus, often repeated, is the standing
"With respect to the alleged unlawful and warrantless arrest, the People manifests that Section 5, Rule rule that absent any showing that the trial judge has overlooked, misunderstood or misappreciated any
113 of the Revised Rules of Court authorizes an arrest without a warrant when the person to be arrested evidence that could otherwise alter the result of the case, the Court would adhere to the assessment
has committed a crime, is actually committing or about to commit a crime in the presence of the police made by the trial court on the question. Parenthetically, the Court of Appeals has basically done likewise
officers. As the appellants were found to be in possession of the prohibited drug at the time of their arrest, in finding no reason to alter the conclusion of the trial judge.
the same is admissible as evidence. In almost every case involving a buy-bust operation, the accused would put up the defense of frame-
"We agree with the appellee. Well-settled is the rule that peace officers may pursue and arrest without a up.1âwphi1 The Court views such a claim with disfavor for, like alibi, the frame-up theory can easily be
warrant any person under circumstances reasonably tending to show that such person has committed or concocted.6 In the instant case, the police informer has particularly mentioned the name of Rolando
is about to commit any crime or breach of the peace ( People vs. Bautista, 227 SCRA 152). In the case at Zaspa as being one of those who would be bringing the bag containing the marijuana, 7 thus paving the
bench, the facts and circumstances leading to the arrest of the accused at dawn of April 29, 1994 would way for the authorities to conduct their operation. When Zaspa, indeed, has made an attempt to run away
show that the arresting officers have proper and justifiable reasons to arrest the two (2) suspects. First, upon seeing the police officers, he inadvertently has also confirmed the information given to the police. It
they received a confidential information from a police informer that a certain Rolando Zaspa with a bears to repeat that absent any convincing proof of an intent on the part of police authorities to falsely
companion were bringing dried marijuana leaves bound for Mati. Second, when the police arrived at the impute a serious crime against an accused, the presumption of regularity in the performance of official
crime scene, the two (2) suspects were suspiciously at the side of the road with a big black bag in front of duty will ordinarily have to prevail.8
them. Third, there were no other people in sight and it is therefore safe to conclude that the bag On the validity of the warrantless arrest, along with the corresponding search and seizure, suffice it to say
containing the contraband belonged to no one else but the suspects.Lastly, when the police officers were that any objection regarding the regularity of an arrest must be made before the accused enters his
approaching, the appellant Zaspa attempted to escape. plea;9 otherwise, the defect shall be deemed cured by the voluntary submission by the accused to the
"A warrantless arrest and seizure was valid where it was done by the police team dispatched to look for jurisdiction of the trial court.10
persons responsible for the crime (People vs. Acol, 232 SCRA 406). In any case, in accordance with No significant value could be given to the allegations of accused-appellants that they were maltreated.
settled jurisprudence, any objection, defect or irregularity attending an arrest must be made before the Zaspa only decided to file charges against the apprehending police officers after almost a year following
accused enters his plea (Padilla vs. Court of Appeals, 269 SCRA 402). Thus, any irregularity attendant to the incident. For his part, Galvan chose to remain silent. Neither one of the two accused-appellants
the arrest of the accused was cured when they voluntarily submitted to the jurisdiction of the trial court by submitted himself to medical examination. The attendant circumstances scarcely augur well to support
entering a plea of not guilty and by participating in the trial (People vs. De Guzman, 224 SCRA 93). the asseveration of maltreatment allegedly suffered by accused-appellants from the police authorities.
"As to the seized marijuana, the same is admissible in evidence, for trite is the jurisprudence that the WHEREFORE, the decision of the trial court is AFFIRMED in toto. Costs against accused-appellants.
search of the appellant's person and the seizure of the marijuana in his possession were valid because SO ORDERED.
they were incident to a lawful warrantless arrest (People vs. Gerente, 219 SCRA 756). As the appellant
Zaspa opened the black bag containing the prohibited drug, then it is a consented search. Settled is the Malacat v CA 283 SCRA 159 (December 12, 1997)
rule that drugs discovered as a result of a consented search is admissible in evidence ( People vs.
Cuizon, 256 SCRA 325). Facts:
"In the case at bench, it is clear that the appellants were really in possession of the seized marijuana. On 27 August 1990, at about 6:30 p.m., allegedly in response to bomb threats reported seven days
Prosecution witness SPO2 Honorio Carasca's testimony that the black bag containing the contraband earlier, Rodolfo Yu ofthe Western Police District, Metropolitan Police Force of the Integrated National
was just 1/2 foot from infront of the appellants (TSN, 4/25/95, p. 10), apart from the fact that there were Police, Police Station No. 3, Quiapo,Manila, was on foot patrol with three other police officers (all of them
no other people in the vicinity, would clearly show that the two (2) appellants were the possessors of the in uniform) along Quezon Boulevard, Quiapo,Manila, near the Mercury Drug store at Plaza Miranda. They
prohibited item. The appellants' respective defenses denying the ownership and possession of the chanced upon two groups of Muslim-looking men, with eachgroup, comprised of three to four men,
marijuana cannot topple the evidence proffered by the prosecution. In the first place, denial, like an alibi, posted at opposite sides of the corner of
is a weak defense which becomes even weaker in the face of positive identification of the accused by “stop and frisk,” where a “warrant andseizure can be effected without necessarily being preceded by an
prosecution witnesses (People vs. Ompad, 233 SCRA 62). Second, the appellants failed to present any arrest” and “whose object is either to maintain the
receipts proving that on different occasions, they were at the crime scene merely by chance. It must be status quo momentarily while the police officer seeks to obtain more information
”; and that the seizure of the grenade from frisk” serves a two
Malacat was incidental to a lawful arrest. The trial court thus found Malacat guilty of the crime of illegal -fold interest: (1) the general interest of effective crime prevention and detection, which underliesthe
possession ofexplosives under Section 3 of PD 1866, and sentenced him to suffer the penalty of not less recognition that a police officer may, under appropriate circumstances and in an appropriate manner,
than 17 years, 4 months and 1day of Reclusion Temporal, as minimum, and not more than 30 years of approach aperson for purposes of investigating possible criminal behavior even without probable cause;
Reclusion Perpetua, as maximum. On 18February 1994, Malacat filed a notice of appeal indicating that and (2) the more pressinginterest of safety and self-preservation which permit the police officer to take
he was appealing to the Supreme Court. However, therecord of the case was forwarded to the Court of steps to assure himself that the person withwhom he deals is not armed with a deadly weapon that could
Appeals (CA-GR CR 15988). In its decision of 24 January 1996, theCourt of Appeals affirmed the trial unexpectedly and fatally be used against the police officer.
court. Manalili filed a petition for review with the Supreme Court. Here, there are at least three (3) reasons why the “stop
Quezon Boulevard - -and-
near the Mercury Drug Store. These men were acting suspiciously with “their eyes moving very fast.” Yu frisk” was invalid: First, there is grave doubts as to Yu’s claim
and his that Malacat was a member of the group which attempted to bomb Plaza Miranda 2 days earlier. This
companions positioned themselves at strategic points and observed both groups for about 30 minutes. claim is neithersupported by any police report or record nor corroborated by any other police officer who
The police officers thenapproached one group of men, who then fled in different directions. As the allegedly chased that group.Second, there was nothing in Malaca
policemen gave chase, Yu caught up with and apprehendedSammy Malacat y Mandar (who Yu t’s behavior or conduct which could have reasonably elicited even mere suspicionother than that his eyes
recognized, inasmuch as allegedly the previous Saturday, 25 August 1990, likewise at PlazaMiranda, Yu were “moving very fast” —
saw Malacat and 2 others attempt to detonate a grenade). Upon searching Malacat, Yu found a an observation which leaves us incredulous since Yu and histeammates were nowhere near Malacat and
fragmentation grenade it was already 6:30 p.m., thus presumably dusk. Malacat and his companionswere merely standing at the
tucked inside the latter’s “front waist line.” Yu’s companion, police officer Rogelio Malibiran, apprehended corner and were not creating any commotion or trouble. Third, there was at all no ground,probable or
Abdul Casan from otherwise, to believe that Malacat was armed with a deadly weapon. None was visible to Yu, for as he
whom a. admitted, the alleged grenade was “discovered” “inside the front waistline” of Malacat, and from all
38 caliber revolver was recovered. Malacat and Casan were then brought to Police Station 3 where Yu indications as to the
placed an “X” mark at the distance between Yu and Malacat, any telltale bulge, assuming that Malacat was indeed hiding a
bottom of the grenade and thereafter gave it to his commander. Yu did not issue any receipt for the grenade, could not have
grenade he allegedly recoveredfrom Malacat. On 30 August 1990, Malacat was charged with violating been visible to Yu. What is unequivocal then are blatant violations of Malacat’s rights solemnly
Section 3 of Presidential Decree 1866. At arraignment on 9October 1990, petitioner, assisted by counsel guaranteed in Sections 2
de officio, entered a plea of not guilty. Malacat denied the charges and explained that heonly recently and 12(1) of Article III of the Constitution.
arrived in Manila. However, several other police officers mauled him, hitting him with benches and guns.
Petitioner wasonce again searched, but nothing was found on him. He saw the grenade only in court Pp v. Figueroa
when it was presented. In its decision dated 10February 1994 but promulgated on 15 February 1994, the
trial court ruled that the warrantless search and seizure of Malacat was akinto a The PEOPLE, Plaintiff and Respondent, v. Bernabe Silva FIGUEROA, Defendant and Appellant.
Issue: E041876. No.
Decided: April 22, 2008
Whether the search made on Malacat is valid, pursuant to the exception of “stop and frisk.” Mark A. Hart, Northridge, under appointment by the Court of Appeal, for Defendant and Appellant.
Edmund G. Brown, Jr., Attorney General, Dane R. Gillette, Chief Assistant Attorney General, Gary W.
Held: Schons, Senior Assistant Attorney General, Rhonda Cartwright-Ladendorf, Supervising Deputy Attorney
The general rule as regards arrests, searches and seizures is that a warrant is needed in order to validly General, and Kristen Kinnaird Chenelia, Deputy Attorney General, for Plaintiff and Respondent.
effect the same.The Constitutional prohibition against unreasonable arrests, searches and seizures refers OPINION
to those effected without avalidly issued warrant, subject to certain exceptions. As regards 269, subd. (a)(1)), A jury convicted defendant of two counts each of aggravated sexual assault of a minor
valid warrantless arrests, these are found in Section5, Rule 113 of the Rules of Court. A (Pen.Code, §1 We disagree and affirm the judgment, while directing the trial court to correct errors in the
warrantless arrest under the circumstances contemplated under Section 5(a)has been deno abstract of judgment.  Defendant appeals, contending that the sentencing court erred in concluding that
minated as one “in flagrante delicto,” while that under Section 5(b) has been described as a his terms for the aggravated sexual assaults must be run consecutively to each other under the
“hotpursuit” arrest. Turning to valid warrantless searches, they are limited to the following: (1) provisions of section 667.6, subdivision (d).   He was sentenced to prison for two consecutive terms of 15
customs searches; (2) years to life plus 15 years four months.  286, subd. (c)(2)).  288, subd. (c)) and one count of forcible
search of moving vehicles; (3) seizure of evidence in plain view; (4) consent searches; (5) a sodomy (§ 288, subd. (a)), committing lewd and lascivious acts on a person under the age of 16 (§ 288,
search incidental to a subd.(b)(1)), committing lewd and lascivious acts on a minor (§ forcibly committing lewd and lascivious
lawful arrest; and (6) a “stop and frisk.” acts on a minor (§
We need not recite the facts surrounding this case, as they are unnecessary to our discussion of the
The concepts of a “stop issue raised by the defendant.
-and- Applicability of Section 667.6, Subdivision (d) to the Terms for Counts 1 and 3 1.
frisk” and of a search incidental to a lawful arrest At the beginning of the sentencing hearing, the court announced that its tentative sentence would include
must not be confused. These two types of warrantless searches differ in terms of the requisite quantum of a 15-year-to-life term each for counts 1 and 3, to run concurrently with each other and full consecutive
proof beforethey may be validly effected and in their allowable scope. In a search incidental to a lawful terms for counts 2 and 4. However, after the prosecutor argued that the terms for counts 2 and 4 would
arrest, as the precedent arrestdetermines the validity of the incidental search. Here, there could have have to be stayed pursuant to section 654, because they were the same acts as were involved in counts
been no valid in flagrante delicto or hot pursuitarrest preceding the search in light of the lack of personal 1 and 3, and the terms for counts 1 and 3 would have to be run consecutively under the provisions of
knowledge on the part of Yu, the arresting officer, or an overtphysical act, on the part of Malacat, section 667.6, subdivision (d), the court imposed consecutive 15-year-to-life terms for counts 1 and 3.
indicating that a crime had just been committed, was being committed or was going tobe committed. The jury was instructed that it had to find, beyond a reasonable doubt, that defendant raped the victim,
Plainly, the search conducted on Malacat could not have been one incidental to a lawful arrest. On the “in violation of Penal Code section 261 subdivision (a), subsection (2)” in order to return verdicts of guilt
other hand, while probable cause is not required to conduct a “stop and frisk,” it neverthe as to counts 1 and 3. In convicting defendant of these counts, the jury necessarily found that he
less holds that mere suspicion or committed violations of section 261, subdivision (a)(2), for which section 667.6, subdivision (d) imposes
a hunch will not validate a “stop and frisk.” A genuine reason must exist, in light of the police officer’s mandatory consecutive sentences.  At the time defendant committed them, section 667.6, subdivision (d)
experience and provided, in pertinent part, “A full, separate and consecutive term shall be served for each violation of ․
surrounding conditions, to warrant the belief that the person detained has weapons concealed about paragraph (2) ․ of subdivision (a) of Section 261․” As to counts 1 and 3, the Information charged
him. Finally, a “stop defendant with “a violation of Penal Code section 269, subdivision (a), subsection (1) ․ in that ․ he did ․
-and- commit a violation of Penal Code section 261, subdivision (a), subsection (2), ․ upon [the victim], a child
who was under 14 years of age and 10 or more years younger than the defendant.”  Defendant first interpretation of inaction on the part of the Legislature.  Additionally, section 667.6, subdivision (d) has
contends that at the time he committed the aggravated sexual assaults of the minor victim, section 667.6, not, since 2006, been amended to add section 269.  Not to be ignored is the analysis of the Assembly
subdivision (d) did not provide for mandatory consecutive sentences for those offenses.   Committee on Public Safety in 1994 that multiple convictions under section 269 were to be punished
Defendant contends that because section 269 was not amended until after he committed his crimes to consecutively under section 667.6, subdivision (d).   However, the digest did not state that that
also include the mandatory consecutive term provision of section 667.6, subdivision (d),  2 that provision requirement did not already exist under section 667.6, subdivision (d).   No doubt, the amendment does,
does not apply to him. indeed, require a consecutive sentence for each offense.  each offense that results in a conviction under
He fails to account for the fact that characterization of section 269 as such would work to the advantage this provision.’ ” 3 Therefore, consecutive sentencing was mandatory under that subdivision.  4
of pedophiles by exempting them from the additional penalties that would ordinarily result when they Disposition
commit multiple offenses․” (Id. at pp. 291-292, 94 Cal.Rptr.2d 884.)  Defendant does not proffer any In all other respects, the judgment is affirmed. The trial court is directed to amend the abstract of
decisional or historical support for his assertion that by enacting section 269 the Legislature created a judgment to show that the sentences imposed for counts 6, 7 and 8 are consecutive terms of one-third of
separate sentencing scheme for violent sexual offenders who prey on a particular class of victims.  [¶] It the midterm, and not full consecutive terms as the abstract currently states.
would be irrational to suppose the Legislature intended that criminals who commit multiple violent sexual FOOTNOTES
offenses would be exempt from the aggravated punishment prescribed by section 667.6 merely because 1All further statutory references are to the Penal Code unless otherwise indicated. .
their victims happened to be children under age 14 who were 10 or more years younger than they.   2Section 269 states, in pertinent part, “(c) The court shall impose a consecutive sentence for each
When the jury found defendant had violated section 269 under the circumstances presented here, it offense that results in a conviction under this section if the crimes involve ․ the same victim on separate
necessarily found he had violated [the] section [prohibiting forcible sodomy] ․ Thus, the factual predicate occasions, as defined in subdivision (d) of section 667.6.” .
necessary to apply section 667.6, subdivision (d) was proved beyond a reasonable doubt.   However, he 3 Therefore, there was no ex post facto application of the 2006 amendment of section 269 to defendant. 
makes too much of this omission, ignoring the fact that violation of [the] section [prohibiting forcible Contrary to defendant's claim, the trial court did not rely on, and did not have to rely on, the post-offense
sodomy] is one of the predicate offenses of section 269;  one committing a forcible sodomy offense with amendment of section 269, which added its reiteration of section 667.6, subdivision (d)'s mandatory
the prescribed age disparity violates section 269.  [¶] Defendant correctly points out that section 667.6, consecutive sentencing provision.  .
subdivision (d) does not explicitly provide that it applies to violations of section 269.  ․ [T]he Legislature 4Because we address defendant's arguments on the merits, and do not conclude that his trial counsel
intended [in Section 269] to aggravate punishment for forcible sexual offenses where the defendant's waived the matter below by failing to object to the imposition of consecutive terms for counts 1 and 3, we
culpability is increased by a substantial age disparity.   [Citation.]  Subdivision (d) of section 667.6 ․ was need not address his contention that his trial counsel was incompetent for failing to make such an
intended by the Legislature to provide increased punishment for cases where defendant's culpability is objection. .
increased by the ‘number and violence of his crimes.’  [¶] Section 667.6 and section 269 serve two THE PEOPLE OF THE PHILIPPINES, plaintiff-appellee,
different objectives.   [Citation.]   In enacting subsequent statutes, the Legislature is presumed to be vs.
aware of existing statutes and judicial decisions.  He notes that before section 269 was amended in DOMINGO SALAZAR alias DARQUEZ, defendant-appellant.
2006, in People v. Jimenez (2000) 80 Cal.App.4th 286, 94 Cal.Rptr.2d 884 (Jimenez ), the Fifth District Office of the Solicitor General Ambrosio Padilla and Solicitor Sumilang V. Bermudo for appellee.
disposed of an argument similar to his as follows, “Section 667.6 was enacted in 1979 [citation];  section Juan T. David for appellant.
269 followed in 1994 [citation]. PER CURIAM:
(Id. at p. 1037, fn. 10, 8 Cal.Rptr.3d 417.)  [Citation.]”  ․ Under [section 269], as it might interact with A complaint for multiple murder, frustrated murder, and attempted murder was filed against the accused
Penal Code section 667.6, a person convicted of six counts of child molestation, could receive ․ six before the Justice of the Peace court of Roxas, Palawan. Having waived his right to preliminary
consecutive life sentences.'  (Penal Code section 667.6, subdivision (d).)   The court is required to investigation, the case was forwarded to the Court of First Instance of the province where the Fiscal filed
sentence such a defendant to full-term consecutive sentences.   They further observed in a footnote, an information for the same crime against him of the following tenor:
“Under current law, a person who is convicted of forcible sex offenses against ․ the same victim on That on or about the 11th day of October, 1956, in the municipality of Roxas, province of Palawan,
different occasions, is subject to special rules of sentencing.  (Id. at p. 1037, 8 Cal.Rptr.3d 417.)   Philippines and within the jurisdiction of this Honorable Court, the said accused, with deliberate intent,
[Citation.]”  Four years after Jimenez was decided, in People v. Glass (2004) 114 Cal.App.4th 1032, 8 evident premeditation, treachery and with the use of deadly weapon, to wit: a spear and big bolo, did then
Cal.Rptr.3d 417 (Glass ), the Fifth District observed, “[T]he bill analysis for section 269 from the Assembly and there wilfully, unlawfully and feloniously attack, assault and wound one after another, in one
Committee on Public Safety specifically refers to the applicability of section 667.6 to section 269 crimes. continuous act and in and around their dwelling houses, the following persons, namely: Maxima Pacho,
However, in so doing, he ignores the language quoted in Glass.  Thus, he suggests, the Legislature has 37 years old and Romana Pacho, 34 years old, both pregnant; Fortunato Nares Jr., 5 years old; Aurelia
already imposed increased punishment for sex crimes against minors, and further punishment under Paz, 7 years old; Lilia Paz, 5 years old; Herminia Paz, 6 months old; Nenita Sausa, 5 years old; and
section 667.6, subdivision (d) is neither appropriate nor intended by the Legislature.   He points out that Henry Pacaldo, 5 years old, all minors; and Felomina Baaco, 48 years old; Salome Baaco, 23 years old;
the terms addressed in section 667.6, subdivision (d) are determinate, while the ones provided in section Baudelio Pacho, 18 years old; Leonila Llavan, 25 years old; Urbana Abique, 50 years old; Felisa Adion,
269, for the same crimes against children, are 15 years to life.  Defendant contends that Jimenez was 37 years old; Lolita Yayen, 17 years old and Manuela Llavan, 39 years old and as a result thereof, the
wrongly decided. said victims died instantly; and in furtherance, of his criminal and heinous act, did then and there wilfully,
It certainly does not apply to sentences under section 667.6, subdivision (d).  However, section 669 unlawfully and feloniously assault, attack and wound one Manuel Adion with the same spear, who, as a
makes no reference whatsoever to section 269.   He appears to suggest that section 669 precludes result thereof, suffered a spear wound at the back, hitting the left lower lobe of the lung, which ordinarily
mandatory consecutive sentences for violations of section 269.   Life sentences ․ may be imposed to run would cause the death of said Manuel Adion, thus performing all the acts of execution which should have
consecutively with one another, with any term imposed for applicable enhancements, or with any other produced the crime of Murder as a consequence, but nevertheless, did not produce it by reason of
term of imprisonment for a felony conviction.”   That section states, “When any person is convicted of two causes of independent of his will, that is, by the timely and able medical attendance rendered to said
or more crimes, whether in the same proceeding ․ or in different proceedings ․ the second or other Manuel Adion which prevented his death; and, finally, not satisfied with his heinous acts and to show
subsequent judgment upon which sentence is ordered to be executed shall direct whether the terms of further his criminal propensity, did then and there wilfully, unlawfully and feloniously, with deliberate to
imprisonment ․ shall run concurrently or consecutively.  He also asserts that section 669 provides the intent to kill Pablo Paz and Severino Adion, by overt acts, but fortunately missed the said Pablo Paz and
sentencing court with discretion to run terms for violations of section 269 either consecutively or Severino Adion.
concurrently.   With the aggravating circumstances of evident premeditation, taking advantage of superior
He also cites the Legislative Counsel's Digest for the 2006 amendment to section 269 which states, strength,alevosia, that the crime was committed in the dwelling of the victims and that the wrong done in
“ ‘The [amendment] would require the court to impose a consecutive sentence for Finally, defendant the commission of the crime was deliberately augmented by causing other wrong not necessary for its
asserts that the amendment of section 269 in September 2006, to expressly apply the mandatory commission. (pp. 2-3, appellee's brief.)
consecutive provision of section 667.6, subdivision (d) signals that the applicability of section 667.6, On October 24, 1956, a physical and mental examination of the accused was conducted by the Chief of
subdivision (d) to section 269 was ambiguous before that date. Section 667.6, subdivision (d) was crystal the Puerto Princesa Hospital and he was found to be normal and sane. He even narrated how he killed
clear, at the time defendant committed his crimes, in its application to the rapes that the jury in this case his common-law wife and his other victims. Two local lawyers were appointed counsel de oficio to defend
found beyond a reasonable doubt to have been committed.  Rather than attempt to sort out these him. Upon arraignment, interpreted in the local dialect, he pleaded guilty. This notwithstanding,
legislative tea leaves, we prefer to adopt the reasoned analysis of our colleagues at District Five in considering the gravity of the offense charged, the Court asked him to take the witness stand and narrate
Jimenez.   What we are left then, at best for defendant, are dueling legislative analyses and an the circumstances surrounding their commission, but he refused stating that he had already made a
confession. Thereupon, the Court asked the prosecution to present its evidence, and the Fiscal presented murdered under our laws, counsel believes that the accused should only be considered as having
among his exhibits the murder weapon, the confession of the accused, the sketches of the scene of the committed the crimes of homicide, or something lesser than murder.
crime, the sixteen certificates of death, the affidavits of some witnesses, and the ante mortem declaration There is no merit in the first argument. It should be noted that, in spite of the plea of guilty entered by the
of Manuel Adion. accused, the Court asked him to take the witness stand and narrate the circumstances surrounding the
The Provincial Commander of Puerto Princesa who investigated the accused testified that he propounded commission of the acts imputed to him, but he refused stating that he had already made a confession.
the questions to him in Tagalog, which is the dialect known to him, while Justice of the Peace Oscar Siat, When he was arraigned, he was assisted by two lawyers who were appointed by the Court to defend him.
before whom the confession was subscribed and sworn to, testified that he read the document and And when the information was read to him, its contents were interpreted into a dialect known to him, and
translated it into Tagalog for the benefit of the accused, and when he asked him if he was coerced into when asked about his answer, his plea was of guilty. If he did not understand the clear import of what is
making the statement, he acknowledged that it was voluntarily given by him. His statement is embodied in embodied in the information, or of what was stated in his written confession, he could have so explained
Exhibit "C". when called by the trial court to take the witness stand. His refusal to do so gives rise to no other
After hearing, the lower court rendered decision the dispositive part of which reads: implication than that he was well aware of the seriousness of the occasion and of the import of the
IN VIEW OF THE FOREGOING, the Court renders judgment finding DOMINGO proceedings against him. He cannot, therefore, complain of the consequences.
SALAZAR, aliasDARQUEZ, guilty of the crime of Multiple Murder with Frustrated Murder as charged, and A plea of guilty is an admission of all the material facts alleged in the information (U.S. vs. Burlado, 42
sentences him to suffer the penalty, as provided for in the Revised Penal Code, of sixteen (16) death Phil., 72; People vs. Acosta, 98 Phil., 642; 52 Off. Gaz., 1930-1933, March 23, 1956). A plea of guilty
penalties corresponding to the murder of each of the sixteen (16) victims, to indemnify the heirs of the when formally entered on arraignment is sufficient to sustain conviction of the offense charged without
deceased in the amount of P3,000.00 each; also to suffer the penalty for the crime of Frustrated Murder introduction of further evidence, upon the theory that the defendant himself has supplied the necessary
of imprisonment the minimum of which shall not be less than EIGHT (8) years and ONE (1) DAY proof by his plea of guilty (U.S. vs. Jawad, 37 Phil., 305). But in this case, despite the accused's plea of
of prision mayor and the maximum of which shall not be less than SEVENTEEN (17) YEARS, FOUR (4) guilty, the prosecution offered evidence considering the peculiar circumstances surrounding the
MONTHS and ONE (1) DAY of reclusion temporal for the two crimes of Attempted Murder to suffer commission of the acts charged. And the evidence presented substantially supports the material
imprisonment for each crime, the minimum of which shall not be less than TWO (2) YEARS, Four (4) allegations of the information . In other words, by his plea, the accused is deemed to have admitted not
MONTHS and ONE (1) DAY of prision correccionaland the maximum of which shall not be more than only the commission of the offense charged, but the circumstances surrounding their commission, such
EIGHT (8) YEARS and ONE (1) DAY of prision mayorand to pay the costs. (pp. 10-11, appellee's brief) as evident premeditation, taking advantage of superior strength, alevosia,and dwelling.
The case was elevated to this Court for review. The mitigating circumstance of obfuscation arising from jealousy cannot be invoked in favor of the
The accused is a moro native of Zamboanga. For many years before the occurrence of the tragedy, he accused considering that his relationship with his common-law wife was illegitimate. (U.S. vs. Hicks, 14
went to barrio San Nicolas, Roxas, Palawan, where he established his home with his common-law wife Phil., 217; People vs. Olgado, et al., G.R. No. L-4406, March 31, 1952). In addition, many days had
Maxima Pacho. In the morning of October 11, 1956, he invited his wife to go with him to gather nipa for already passed from the discovery of the alleged infidelity of his common-law wife before he committed
the repair of their house. Romana then arrived and invited Maxima to accompany her to her house to get the crime allegedly in vindication of his honor. As a matter of fact he admitted having planned his
palay. Because of the invitation of Romana, Maxima refused to go with her husband, which aroused his vengeance long before the opportune moment came to carry it out.
anger. At that time, the accused already entertained the suspicion that his wife was having illicit relation Regarding the contention that running amuck is a cult among the Moros that is age-old and deeply rooted
with Fortunato Nares, husband of Romana, to the extent that he believed that the child his wife was and should be distinguished from murders where the murderer is not resigned to expiate his offense by
bearing was the result of such illicit relation. This incident started the accused on a killing rampage being killed unlike the amuck, the claim is likewise unmeritorious. Our penal laws enumerate the
leaving in its wake sixteen dead and some wounded. The following is what happened as related by the circumstances which mitigate criminal liability, and the condition of running amuck is not one of them. In
trial court: "He started by killing his own sister-in-law. Then he turned to kill his own wife and his nephew. so far as they are applicable they must be applied alike to all criminals be they Christians, Moros or
He then walked for about 800 meters to another house where he killed and massacred all the inmates he Pagans. More so in the case of the accused who though Moro by blood, has however settled for many
found therein. He went to three other houses and repeated the same performance leaving behind him years before the occurrence of the tragedy in christian community and lived there with a Christian
several household filled with tragedy and bloody deaths, the like of which have never been seen or known common-law wife and relatives. Under such atmosphere, he must have been indoctrinated into the
in this province. Then he went into the school premises of that sitio and attacked one man from behind Christian way of life to such extent that he should have known that running amuck is abhorred in our
who was able to run away and is the lone survivor of this bloody incident. After that he chased and society and punished by our law. He cannot even invoke in his favor what Section 106 of the
actually did throw his spear to two other men but whom he did not hit. And then he went to the school Administrative Code of Mindanao and Sulu accords to a Moro who commits a crime and is convicted, for
house and tried to force open the door, also with dark criminal intention. Fortunately due to the presence even then said section gives to the Court ample discretion to determine the penalty to be imposed
of mind of the lady teacher the door and windows were locked and barricaded while the accused went considering the circumstances of the case, the degree of his instruction, and nature of the crime
around trying to force upon the entrance and the windows of the school house. The imagination of this committed, the Court being justified in imposing the penalty which would best serve the interest of justice.
Court shudders of the thought of what might have happened if this accused succeeded in going into the This is a case where the degree of perversity of the criminal warrants not mercy but the enforcement of
school room filled with young school children. And the accused went up to the bell tower of the Chapel, the law to its full extent.
sounded the alarm calling for everybody to come. He asked that he be killed but no one dared to come We are, therefore, constrained to hold that the accused is guilty beyond reasonable doubt of the crimes of
and obey his request. Finally, when two armed guards arrived accompanied by a barrio officer, the multiple murder, frustrated murder, and attempted murder, all qualified by evident premeditation as
accused was persuaded to surrender but on condition that he be shot and killed immediately. The guards charged in the information.
and barrio officer cleverly made a ruse by asking the accused to go with them to the wharf where he will The murder of Filomena Baaco, Baudilla, Pacho, Salome Baaco, Leonila Llavan, Urbana Abique, Lilia
be shot, and by asking him to sign a piece of paper to serve as their protection for the killing the accused Paz, Aurelia Paz, Herminia Paz, Lolita Yayen, Felisa Adion, Nenita Sausa, Henry Pacaldo and Manuela
which would be presented to the authorities later on. The accused in the meantime had put down his Llavan, was attended by the aggravating circumstances of treachery and dwelling. The murder of Maxima
arms and as he was about to affix his thumbmark on the piece of paper he was suddenly grabbed and Pacho, Romana Pacho, and Fortunata Nares, the frustrated murder of Manuel Adion and the attempted
was subdued and put under custody." murder of Pablo Paz and Severino Adion were committed with the aggravating circumstance of treachery.
Let us now come to the arguments advanced by counsel for the accused in an effort to mitigate his The aggravating circumstance of treachery in the sixteen murders, one frustrated murder and two
liability. These arguments in a nutshell may be boiled down into three, to wit: attempted murders, is however offset by the mitigating circumstance of plea guilty.
1. That the accused's plea of guilty did not extend to the admission of the correctness of the qualification With respect to the murder of the accused common-law wife, Maxima Pacho, his sister-in-law, Romana
of his acts as expressed by the prosecution in its information, particularly the allegation that the acts of Pacho, and the latter's nephew, Fortunato Nares, the frustrated murder of Manuel Adion and attempted
the accused were committed with evident premeditation and with treachery, murder of Pablo Paz and Severino Adion, the penalty should be imposed in its medium period. In the
2. That running amuck, or becoming a "juramentado" is a cult among the Moros that forms part of their murder of Maxima Pacho, Romana Pacho and Fortunato Nares, the penalty should be reclusion
religion. It is age-old and deeply rooted in their psychology. The Moros do not discourage its observance. perpetua, in the frustrated murder of Manuel Adion, the defendant should be sentenced to suffer an
They do not view the observance of this cult as a heinous crime. The calculated risk they take when a indeterminate penalty of 4 years, 2 months and 1 day of prision correccional as minimum to 12 years and
Moro becomes "juramentado" is taken graciously. They are always alert to kill a "juramentado" before his 1 day of reclusion temporal as maximum.
victims multiply. And For each of the two crimes of attempted murder, the defendant should be sentenced to suffer an
3. Consequently, because the acts committed by the accused were parts and parcel of his observance of indeterminate penalty of 4 months and 1 day of arresto mayor as minimum to 6 years and 1 day of prision
the Moro cult of being a "juramentado", which should be distinguished from the acts of a common mayor as maximum.
The indemnity to the heirs of each of the deceased should be P6,000. facts or circumstances of weight and substance which could have altered the conviction of the
Wherefore, modified as above indicated, the decision appealed from is hereby affirmed in all other appellants. 6
respects, with costs. In this case, the findings of the trial court that the prosecution witnesses were more credible than those of
the defense must stand. Petitioner failed to show that Pat. Pagilagan, in testifying against him, was
RODOLFO ESPANO, accused-petitioner, motivated by reasons other than his duty to curb drug abuse and had any intent to falsely impute to him
vs. such a serious crime as possession of prohibited drugs. In the absence of such ill motive, the
COURT OF APPEALS and PEOPLE OF THE PHILIPPINES, respondents. presumption of regularity in the performance of his official duty must prevail.
In People v. Velasco, 7 this Court reiterated the doctrine of presumption of regularity in the performance of
ROMERO, J.: official duty which provides:
This is a petition for review of the decision of the Court of Appeals in CA-G.R. CR No. 13976 dated . . . Appellant failed to establish that Pat. Godoy and the other members of the buy-bust team are
January 16, 1995, 1 which affirmed in toto the judgment of the Regional Trial Court of Manila, Branch 1, policemen engaged in mulcting or other unscrupulous activities who were motivated either by the desire
convincing petitioner Rodolfo Espano for violation of Article II, Section 8 of Republic Act No. 6425, as to extort money or exact personal vengeance, or by sheer whim and caprice, when they entrapped her.
amended, otherwise known as the Dangerous Drugs Act. And in the absence of proof of any intent on the part of the police authorities to falsely impute such a
Petitioner was charged under the following information: serious crime against appellant, as in this case, the presumption of regularity in the performance of official
That on or about July 14, 1991, in the City of Manila, Philippines, the said accused not being authorized duty, . . . , must prevail over the self-serving and uncorroborated claim of appellant that she had been
by law to possess or use any prohibited drug, did then and there willfully, unlawfully and knowingly have framed. 8
in his possession and under his custody and control twelve (12) plastic cellophane (bags) containing Furthermore, the defense set up by petitioner does not deserve any consideration. He simply contended
crushed flowering tops, marijuana weighing 5.5 grams which is a prohibited drug. that he was in his house sleeping at the time of the incident. This Court has consistently held that alibi is
Contrary to law. 2 the weakest of all defenses; and for it to prosper, the accused has the burden of proving that he was not
The evidence for the prosecution, based on the testimony of Pat. Romeo Pagilagan, shows that on July at the scene of the crime at the time of its commission and that it was physically impossible for him to be
14, 1991, at about 12:30 a.m., he and other police officers, namely, Pat. Wilfredo Aquino, Simplicio there. Moreover, the "claim of a 'frame-up', like alibi, is a defense that has been invariably viewed by the
Rivera, and Erlindo Lumboy of the Western Police District (WPD), Narcotics Division went to Zamora and Court with disfavor for it can just as easily be concocted but difficult to prove, and is a common and
Pandacan Streets, Manila to confirm reports of drug pushing in the area. They saw petitioner selling standard line of defense in most prosecutions arising from violations of the Dangerous Drugs Act." 9 No
"something" to another person. After the alleged buyer left, they approached petitioner, identified clear and convincing evidence was presented by petitioner to prove his defense of alibi.
themselves as policemen, and frisked him. The search yielded two plastic cellophane tea bags of Second, petitioner contends that the prosecution's failure to present the alleged informant in court cast a
marijuana. When asked if he had more marijuana, he replied that there was more in his house. The reasonable doubt which warrants his acquittal. This is again without merit, since failure of the prosecution
policemen went to his residence where they found ten more cellophane tea bags of marijuana. Petitioner to produce the informant in court is of no moment especially when he is not even the best witness to
was brought to the police headquarters where he was charged with possession of prohibited drugs. On establish the fact that a buy-bust operation had indeed been conducted. In this case, Pat. Pagilagan, one
July 24, 1991, petitioner posted bail 3 and the trial court issued his order of release on July 29, 1991. 4 of the policemen who apprehended petitioner, testified on the actual incident of July 14, 1991, and
Annabelle Alip, forensic chemist of the WPD Criminal Investigation Laboratory Section, testified that the identified him as the one they caught in possession of prohibited drugs. Thus,
articles sent to her by Pat. Wilfredo Aquino regarding the apprehension of a certain Rodolfo Espano for We find that the prosecution had satisfactorily proved its case against appellants. There is no compelling
examination tested positive for marijuana, with a total weight of 5.5 grams. reason for us to overturn the finding of the trial court that the testimony of Sgt. Gamboa, the lone witness
By way of defense, petitioner testified that on said evening, he was sleeping in his house and was for the prosecution, was straightforward spontaneous and convincing. The testimony of a sole witness, if
awakened only when the policemen handcuffed him. He alleged that the policemen were looking for his credible and positive and satisfies the court beyond reasonable doubt, is sufficient to convict. 10
brother-in-law Lauro, and when they could not find the latter, he was instead brought to the police station Thus on the basis of Pat. Pagilagan's testimony, the prosecution was able to prove that petitioner indeed
for investigation and later indicted for possession of prohibited drugs. His wife Myrna corroborated his committed the crime charged; consequently, the finding of conviction was proper.
story. Lastly, the issue on the admissibility of the marijuana seized should likewise be ruled upon. Rule 113
The trial court rejected petitioner's, defense as a "mere afterthought" and found the version of the Section 5(a) of the Rules of Court provides:
prosecution "more credible and trustworthy." A peace officer or a private person may, without a warrant, arrest a person:
Thus, on August 14, 1992, the trial court rendered a decision, convicting petitioner of the crime charged, a. when, in his presence, the person to be arrested has committed, is actually committing, or is attempting
the dispositive portion of which reads: to commit an offense;
WHEREFORE there being proof beyond reasonable doubt, the court finds the accused Rodolfo Espano y xxx xxx xxx
Valeria guilty of the crime of violation of Section 8, Article II, in relation to Section 2 (e-L) (I) of Republic Petitioner's arrest falls squarely under the aforecited rule. He was caught in flagranti as a result of a buy-
Act No. 6425 as amended by Batas Pambansa Blg. 179, and pursuant to law hereby sentences him to bust operation conducted by police officers on the basis of information received regarding the illegal trade
suffer imprisonment of six (6) years and one (1) day to twelve (12) years and to pay a fine of P6,000.00 of drugs within the area of Zamora and Pandacan Streets, Manila. The police officer saw petitioner
with subsidiary imprisonment in case of default plus costs. handing over something to an alleged buyer. After the buyer left, they searched him and discovered two
The marijuana is declared forfeited in favor of government and shall be turned over to the Dangerous cellophanes of marijuana. His arrest was, therefore, lawful and the two cellophane bags of marijuana
Drugs Board without delay. seized were admissible in evidence, being the fruits of the crime.
SO ORDERED. 5 As for the ten cellophane bags of marijuana found at petitioner's residence, however, the same are
Petitioner appealed the decision to the Court of Appeals. The appellate court, however, affirmed the inadmissible in evidence.
decision of the trial court in toto. The 1987 Constitution guarantees freedom against unreasonable searches and seizures under Article III,
Hence, this petition. Section 2 which provides:
Petitioner contends that the trial and appellate courts erred in convicting him on the basis of the following: The right of the people to be secure in their persons, houses, papers and effects against unreasonable
(a) the pieces of evidence seized were inadmissible; (b) the superiority of his constitutional right to be searches and seizures of whatever nature and for any purpose shall be inviolable, and no search warrant
presumed innocent over the doctrine of presumption of regularity, (c) he was denied the constitutional or warrant of arrest shall issue except upon probable cause to be determined personally by the judge
right of confrontation and to compulsory process; and (d) his conviction was based on evidence which after examination under oath or affirmation of the complainant and the witnesses he may produce, and
was irrelevant and not properly identified. particularly describing the place to be searched and the persons or things to be seized.
After a careful examination of the records of the case, this Court finds no compelling reason sufficient to An exception to the said rule is a warrantless search incidental to a lawful arrest for dangerous weapons
reverse the decisions of the trial and appellate courts. or anything which may be used as proof of the commission of an offense. 11 It may extend beyond the
First, it is a well settled doctrine that findings of trial courts on the credibility of witnesses deserve a high person of the one arrested to include the premises or surroundings under his immediate control. In this
degree of respect. Having observed the deportment of witnesses during the trial, the trial judge is in a case, the ten cellophane bags of marijuana seized at petitioner's house after his arrest at Pandacan and
better position to determine the issue of credibility and, thus, his findings will not be disturbed during Zamora Streets do not fall under the said exceptions.
appeal in the absence of any clear showing that he had overlooked, misunderstood or misapplied some In the case of People v. Lua, 12 this Court held:
As regards the brick of marijuana found inside the appellant's house, the trial court correctly ignored it (5)stop and frisk situations (Terry search), and(6)search incidental to a lawful arrest.It is required in cases
apparently in view of its inadmissibility. While initially the arrest as well as the body search was lawful, the of
warrantless search made inside the appellant's house became unlawful since the police operatives were in flagrante delicto
not armed with a search warrant. Such search cannot fall under "search made incidental to a lawful that the arresting officer
arrest," the same being limited to body search and to that point within reach or control of the person musthave personal knowledge of such facts or circumstances convincinglyindicative or constitutive of pro
arrested, or that which may furnish him with the means of committing violence or of escaping. In the case bable cause. Probable cause means areasonable ground of suspicion supported by circumstances
at bar, appellant was admittedly outside his house when he was arrested. Hence, it can hardly be said sufficientlystrong in themselves to warrant a cautious man's belief that the personaccused is guilty of the
that the inner portion of his house was within his reach or control. offense with which he is charged. In the case atbar, there are no facts on record reasonably suggestive or
The articles seized from petitioner during his arrest were valid under the doctrine of search made demonstrativeof CHUA's participation in on going criminal enterprise that could havespurred police
incidental to a lawful arrest. The warrantless search made in his house, however, which yielded ten officers from conducting the obtrusive search. CHUA wasnot identified as a drug courier by a police
cellophane bags of marijuana became unlawful since the police officers were not armed with a search informer or agent. The fact thatthe vessel that ferried him to shore bore no resemblance to the
warrant at the time. Moreover, it was beyond the reach and control of petitioner. fishingboats of the area did not automatically mark him as in the process of perpetrating an offense. With
In sum, this Court finds petitioner Rodolfo Espano guilty beyond reasonable doubt of violating Article II, these, the Court held that there was noprobable cause to justify a search incidental to a lawful arrest. The
Section 8, in relation to Section 2 (e-L) (I) of Republic Act No. 6425, as amended. Under the said Court likewise did not appreciate the contention of the
provision, the penalty imposed is six years and one day to twelve years and a fine ranging from six Prosecutionthat there was a waiver or consented search. If CHUA could notunderstand what was orally
thousand to twelve thousand pesos. With the passage of Republic Act No. 7659, which took effect on articulated to him, how could he
December 31, 1993, the imposable penalty shall now depend on the quantity of drugs recovered. Under understandthe police's "sign language?" More importantly, it cannot logically beinferred from his alleged c
the provisions of Republic Act No. 7629, Section 20, and as interpreted in People v. Simon 13 and People ognizance of the "sign language" that hedeliberately, intelligently, and consciously waived his right against
v. Lara, 14 if the quantity of marijuana involved is less than 750 grams, the imposable penalty ranges suchan intrusive search.Finally, being a forbidden fruit, the subject regulated substance was heldto be
from prision correccional to reclusion temporal. Taking into consideration that petitioner is not a habitual inadmissible in evidence.Hence, the accused was acquitted as the evidence was not sufficient toestablish
delinquent, the amendatory provision is favorable to him and the quantity of marijuana involved is less guilt beyond reasonable doubt.
than 750 grams, the penalty imposed under Republic Act No. 7659 should be applied. There being no
mitigating nor aggravating circumstances, the imposable penalty shall be prision correccional in its Terry v. Ohio
medium period. Applying the Indeterminate Sentence Law, the maximum penalty shall be taken from the Brief Fact Summary. The Petitioner, John W. Terry (the “Petitioner”), was stopped and searched by an
medium period of prision correccional, which is two (2) years, four (4) months and one (1) day to four (4) officer after the officer observed the Petitioner seemingly casing a store for a potential robbery. The officer
years and two (2) months, while the minimum shall be taken from the penalty next lower in degree, which approached the Petitioner for questioning and decided to search him first.
is one (1) month and one (1) day to six (6) months of arresto mayor.
WHEREFORE, the instant petition is hereby DENIED. The decision of the Court of Appeals in C.A.-G.R. Synopsis of Rule of Law. An officer may perform a search for weapons without a warrant, even without
CR No. 13976 dated January 16, 1995 is AFFIRMED with the MODIFICATION that petitioner Rodolfo probable cause, when the officer reasonably believes that the person may be armed and dangerous.
Espano is sentenced to suffer an indeterminate penalty of TWO (2) months and ONE (1) day of arresto
mayor, as minimum to TWO (2) years, FOUR (4) months and ONE (1) day of prision correccional, as Facts. The officer noticed the Petitioner talking with another individual on a street corner while repeatedly
maximum. walking up and down the same street. The men would periodically peer into a store window and then talk
SO ORDERED. some more. The men also spoke to a third man whom they eventually followed up the street. The officer
believed that the Petitioner and the other men were “casing” a store for a potential robbery. The officer
People vs Chua Ho San decided to approach the men for questioning, and given the nature of the behavior the officer decided to
perform a quick search of the men before questioning. A quick frisking of the Petitioner produced a
G.R. No. 128222 June 17, 1999PEOPLE OF THE PHILIPPINES, plaintiff-appellee,vs.CHUA HO SAN @ concealed weapon and the Petitioner was charged with carrying a concealed weapon.
TSAY HO SAN, accused-appellant.FACTS OF THE
CASE:In response to reports of rampant smuggling of firearms and othercontraband, Chief of Police Jim Issue. Whether a search for weapons without probable cause for arrest is an unreasonable search under
Lagasca Cid of Bacnotan Police Station, the Fourth Amendment to the United States Constitution (”Constitution”)?
LaUnion began patrolling the Bacnotan coastline with his officers. Whilemonitoring the coastal area of Held. The Supreme Court of the United States (”Supreme Court”) held that it is a reasonable search when
Barangay Bulala, he intercepted a radio call an officer performs a quick seizure and a limited search for weapons on a person that the officer
ataround 12:45 p.m. from Barangay Captain Juan Almoite of Barangay Tammocalao requesting for police reasonably believes could be armed. A typical beat officer would be unduly burdened by being prohibited
assistance regarding an unfamiliarspeedboat the latter had spotted. According to Almoite, the vessel from searching individuals that the officer suspects to be armed.
lookeddifferent from the boats ordinarily used by fisherfolk of the area and waspoised to dock at
Tammocalao shores. Cid and six of his men led by Dissent. Justice William Douglas (”J. Douglas”) dissented, reasoning that the majority’s holding would
SPO1Reynoso Badua, proceeded immediately to Tammocalao beach and thereconferred with Almoite. grant powers to officers to authorize a search and seizure that even a magistrate would not possess.
Cid then observed that the speedboat ferried a lonemale passenger, who was later identified as Chua Ho
San. When the speedboat landed, the male passenger alighted, carrying a multicolored Concurrence.
strawbag,and walked towards the road. Upon seeing the police officers, the manchanged direction. Justice John Harlan (”J. Harlan”) agreed with the majority, but he emphasized an additional necessity of
Badua held Chua’s right arm to prevent him from fleeing. They then introduced themselves as police the reasonableness of the stop to investigate the crime.
officers; however, Chua did notunderstand what they’re saying. And by resorting of “sign language”, Justice Byron White (”J. White”) agreed with the majority, but he emphasized that the particular facts of
Cidmotioned with his hands for the man to open his bag. The man acceded tothe request. The said bag the case, that there was suspicion of a violent act, merit the forcible stop and frisk.
was found to contain several transparent plasticscontaining yellowish crystalline substances, which was
later identified to bemethamphetamine hydrochloride or shabu. Chua was then brought toBacnotan Police Discussion. The facts of the case are important to understand the Supreme Court’s willingness to allow
Station, where he was provided with an interpreter to informhim of his constitutional rights.ISSUE: the search. The suspicious activity was a violent crime, armed robbery, and if the officer’s suspicions were
Whether or not the warrantless arrest, search and seizure correct then he would be in a dangerous position to approach the men for questioning without searching
conductedby the Police Officers constitute a valid exemption from the warrantrequirement.RULING: The them. The officer also did not detain the men for a long period of time to constitute an arrest without
Court held in the negative. The Court explains that the Constitution bars State intrusions to a probable cause.
person'sbody, personal effects or residence except if conducted by virtue of a valid
of a valid search warrant issued in accordance with the Rules. However,warrantless searches may be PP VS TABAR
permitted in the following cases, to wit:(1)search of moving vehicles,(2)seizure in plain view, FACTS:
(3)customs searches,(4)waiver or consent searches, - Carmelina Tabar y Carmilotes and her nephew, Rommel Arriesgado y Tabar, were charged with
theviolation of Dangerous Drug Act, for selling and delivering, without authority of law, 3 sticks of
marijuanacigarettes.- Pfc. Josephus Trangia On February 8, 1989, Pfc. Josephus Trangia he was with evidence was inadvertently discovered by the police who had the right to be where they are; (c) the
Pfc. Romeo Cortes andGualberto Gabales on a buy-bust operation for marijuana after receipt of evidence must be immediately apparent, and (d) "plain view" justified mere seizure of evidence without
information about marijuanapushers in Punta Princesa, Cebu City and that they had their informant go further search;
ahead of them after giving theP5.00 bill for him to purchase marijuana. He continued saying that their
informant stood in front of ashanty while they posted themselves at a distance of about 50 meters from 3. Search of a moving vehicle. Highly regulated by the government, the vehicle's inherent mobility
the place where theirinformant was standing. And that they saw a young boy approached their informant reduces expectation of privacy especially when its transit in public thoroughfares furnishes a highly
and handedcigarettes to him who in turn handed the marked money to the young boy. Then, their reasonable suspicion amounting to probable cause that the occupant committed a criminal activity;
informant gavethem the pre-arranged signal of scratching his head with his right hand; that after the
signal, he and hiscompanions immediately approached the young boy and the informant introduced them 4. Consented warrantless search;
as policeofficers. This young boy was about 16-17 years old, by the name of Rommel Arriesgado y Tabar.
He hadearlier pleaded guilty to the lesser offense of mere possession of marijuana He explained the 5. Customs search;
initials GDGwhich stands for Gualberto G. Gabales, his team member. He further declared that after the
pre-arranged signal from their informant, they immediately proceeded to the scene and were given 6. Stop and Frisk;
threesticks of marijuana by their informant after buying the same from the boy, Rommel Arriesgado and
thatthey proceeded to confiscate the P5.00 bill from the boy. At this juncture, he claimed that he 7. Exigent and Emergency Circumstances.
observedthat after the transaction, the boy went inside the shanty and the moment he got out, he handed
thethree sticks of marijuana to the informant. In fact, he claimed that after the proceeding to the The essential requisite of probable cause must still be satisfied before a warrantless search and seizure
shanty,they met Carmelina Tabar, accused herein, and that Carmelina Tabar was holding white pants can be lawfully conducted.
fromwhere they found other marijuana sticks in cigarette packs which they confiscated; that they
broughtCarmelina Tabar to Fuente Police Station for investigation. He claimed that there were 75 sticks of The accused cannot be said to be committing a crime, she was merely crossing the street and was not
marijuana in the Hope Cigarette pack; 22 sticks of marijuana cigarettes in the Philip Morris pack and acting suspiciously for the Narcom agents to conclude that she was committing a crime. There was no
99sticks of marijuana in the Mark cigarette pack. legal basis to effect a warrantless arrest of the accused’s bag, there was no probable cause and the
ISSUE accused was not lawfully arrested.
: WON the marijuana cigarettes seized from her are inadmissible in evidence because they wereobtained
in violation of the constitutional guarantee against unreasonable search and seizure. The police had more than 24 hours to procure a search warrant and they did not do so. The seized
HELD: marijuana was illegal and inadmissible evidence.
The evidence for the prosecution discloses that the appellant placed the packs of marijuana sticksunder
the rolled pair of pants which she was then carrying at the time she hurriedly left her shanty afternoticing
the arrest of Rommel. When she was asked to spread it out, which she voluntary did, thepackage RULE 113, RULES OF COURT
containing the packs of marijuana sticks were thus exposed in plain view to the member of theteam. A
crime was thus committed in the presence of the policemen. Pursuant to Section 5, Rule 113 andSection Section 5. Arrest without warrant; when lawful. — A peace officer or a private person may, without a
12 Rule 126 of the Revised Rules of Court, warrant, arrest a person:
she could lawfully be arrested and searched foranything which may be used as proof of the commission
of an offense without thecorresponding arrest and search warrants. (a) When, in his presence, the person to be arrested has committed, is actually committing, or is
attempting to commit an offense;
PP VS ARUTA (b) When an offense has just been committed, and he has probable cause to believe based on personal
knowledge of facts or circumstances that the person to be arrested has committed it; and
Facts: On Dec. 13, 1988, P/Lt. Abello was tipped off by his informant that a certain “Aling Rosa” will be (c) When the person to be arrested is a prisoner who has escaped from a penal establishment or place
arriving from Baguio City with a large volume of marijuana and assembled a team. The next day, at the where he is serving final judgment or is temporarily confined while his case is pending, or has escaped
Victory Liner Bus terminal they waited for the bus coming from Baguio, when the informer pointed out while being transferred from one confinement to another.
who “Aling Rosa” was, the team approached her and introduced themselves as NARCOM agents. When
Abello asked “aling Rosa” about the contents of her bag, the latter handed it out to the police. They found In cases falling under paragraph (a) and (b) above, the person arrested without a warrant shall be
dried marijuana leaves packed in a plastic bag marked “cash katutak”. forthwith delivered to the nearest police station or jail and shall be proceeded against in accordance with
section 7 of Rule 112.
Instead of presenting its evidence, the defense filed a demurrer to evidence alleging the illegality of the
search and seizure of the items. In her testimony, the accused claimed that she had just come from
Choice theatre where she watched a movie “Balweg”. While about to cross the road an old woman asked RULE 126, RULES OF COURT
her for help in carrying a shoulder bag, when she was later on arrested by the police. She has no
knowledge of the identity of the old woman and the woman was nowhere to be found. Also, no search Section 2. Court where application for search warrant shall be filed. — An application for search warrant
warrant was presented. shall be filed with the following:

The trial court convicted the accused in violation of the dangerous drugs of 1972 a) Any court within whose territorial jurisdiction a crime was committed.
b) For compelling reasons stated in the application, any court within the judicial region where the crime
was committed if the place of the commission of the crime is known, or any court within the judicial region
Issue: Whether or Not the police correctly searched and seized the drugs from the accused. where the warrant shall be enforced.

However, if the criminal action has already been filed, the application shall only be made in the court
Held: The following cases are specifically provided or allowed by law: where the criminal action is pending.

1. Warrantless search incidental to a lawful arrest recognized under Section 12, Rule 126 of the Rules of Section 7. Right to break door or window to effect search. — The officer, if refused admittance to the
Court 8 and by prevailing jurisprudence place of directed search after giving notice of his purpose and authority, may break open any outer or
inner door or window of a house or any part of a house or anything therein to execute the warrant or
2. Seizure of evidence in "plain view," the elements of which are: (a) a prior valid intrusion based on the liberate himself or any person lawfully aiding him when unlawfully detained therein.
valid warrantless arrest in which the police are legally present in the pursuit of their official duties; (b) the
Section 12. Delivery of property and inventory thereof to court; return and proceedings thereon. Held:
Section 4 of Republic Act 3512 approved on 20 March 1963 empowers the Fisheries Commissioner to
(a) The officer must forthwith deliver the property seized to the judge who issued the warrant, together carry out the provisions of the Fisheries Act, as amended, and all rules and regulations promulgated
with a true inventory thereof duly verified under oath. thereunder, to make searches and seizures personally or through his duly authorized representatives in
(b) Ten (10) days after issuance of the search warrant, the issuing judge shall ascertain if the return has accordance with the Rules of Court, of "explosives such as dynamites and the like; including fishery
been made, and if none, shall summon the person to whom the warrant was issued and require him to products, fishing equipment, tackle and other things that are subject to seizure under existing fishery
explain why no return was made. If the return has been made, the judge shall ascertain whether section laws"; and "to effectively implement the enforcement of existing fishery laws on illegal fishing." Paragraph
11 of this Rule has been complained with and shall require that the property seized be delivered to him. 5 of Section 4 of the same Republic Act 3512 likewise transferred to and vested in the Philippine
The judge shall see to it that subsection (a) hereof has been complied with. Fisheries Commission "all the powers, functions and duties heretofore exercised by the Bureau of
(c) The return on the search warrant shall be filed and kept by the custodian of the log book on search Customs, Philippine Navy and Philippine Constabulary over fishing vessels and fishery matters." Section
warrants who shall enter therein the date of the return, the result, and other actions of the judge. 12 of the Fisheries Act, otherwise known as Republic Act 4003, as amended, prohibits fishing with
dynamites or other explosives which is penalized by Section 76 thereof "by a fine of not less than
A violation of this section shall constitute contempt of court. P1,500.00 nor more than P5,000.00, and by imprisonment for not less than one (1) year and six (6)
months nor more than five (5) years, aside from the confiscation and forfeiture of all explosives, boats,
Roldan vs. Arca [GR L-25434, 25 July 1975] tackles, apparel, furniture, and other apparatus used in fishing in violation of said Section 12 of this Act."
First Division, Makasiar (J): 4 concur, 1 took no part Constitutional Law II, 2005 ( 62 ) Narratives (Berne Section 78 of the same Fisheries Law provides that "in case of a second offense, the vessel, together
Guerrero) with its tackle, apparel, furniture and stores shall be forfeited to the Government." The second paragraph
Facts: of Section 12 also provides that "the possession and/or finding, of dynamite, blasting caps and other
On 3 April 1964, Morabe, De Guzman & Company filed with the Court of First Instance (CFI) of Manila a explosives in any fishing boat shall constitute a presumption that the said dynamite and/or blasting caps
civil case (56701) against Fisheries Commissioner Arsenio N. Roldan, Jr., for the recovery of fishing and explosives are being used for fishing purposes in violation of this Section, and that the possession or
vessel Tony Lex VI which had been seized and impounded by the Fisheries Commissioner through the discover in any fishing boat or fish caught or killed by the use of dynamite or other explosives, under
Philippine Navy. On 10 April 1964, the company prayed for a writ of preliminary mandatory injunction with expert testimony, shall constitute a presumption that the owner, if present in the fishing boat, or the fishing
the CFI, but said prayer was denied. On 28 April 1964, the CFI set aside its order of 10 April 1964 and crew have been fishing with dynamite or other explosives." Under Section 78 of the Fisheries Act, as
granted the company's motion for reconsideration praying for preliminary mandatory injunction. Thus, the amended, any person, association or corporation fishing in deep sea fishery without the corresponding
company took possession of the vessel Tony Lex VI from the Philippine Fisheries Commission and the license prescribed in Sections 17 to 22 Article V of the Fisheries Act or any other order or regulation
Philippine Navy by virtue of the said writ. On 10 December 1964, the CFI dismissed Civil Case 56701 for deriving force from its provisions, "shall be punished for each offense by a fine of not more than
failure of the company to prosecute as well as for failure of the Commission and the Navy to appear on P5,000.00, or imprisonment, for not more than one year, or both, in the discretion of the Court; Provided,
the scheduled date of hearing. The vessel, Tony Lex VI or Srta. Winnie however, remained in the That in case of an association or corporation, the President or manager shall be directly responsible for
possession of the company. On 20 July 1965, the Fisheries Commissioner requested the Philippine Navy the acts of his employees or laborers if it is proven that the latter acted with his knowledge; otherwise the
to apprehend vessels Tony Lex VI and Tony Lex III, also respectively called Srta. Winnie and Srta. Agnes, responsibility shall extend only as far as fine is concerned: Provided, further, That in the absence of a
for alleged violations of some provisions of the Fisheries Act and the rules and regulations promulgated known owner of the vessel, the master, patron or person in charge of such vessel shall be responsible for
thereunder. On August 5 or 6, 1965, the two fishing boats were actually seized for illegal fishing with any violation of this Act: and Provided, further, That in case of a second offense, the vessel together with
dynamite. Fish caught with dynamite and sticks of dynamite were then found aboard the two vessels. On its tackle, apparel, furniture and stores shall be forfeited to the Government." Under Section 13 of
18 August 1965, the Fisheries Commissioner requested the Palawan Provincial Fiscal to file criminal Executive Order 389 of 23 December 1950, reorganizing the Armed Forces of the Philippines, the
charges against the crew members of the fishing vessels. On 30 September 1965, there were filed in the Philippine Navy has the function, among others, "to assist the proper governmental agencies in the
CFI of Palawan a couple of informations, one against the crew members of Tony Lex III, and another enforcement of laws and regulations pertaining to Fishing. Section 2210 of the Tariff and Customs Code,
against the crew members of Tony Lex VI as amended by PD 34 of 27 October 1972, authorized any official or person exercising police authority
— under the provisions of the Code, to search and seize any vessel or air craft as well as any trunk,
both for violations of Act 4003, as amended by Commonwealth Acts 462, 659 and 1088, i.e., for illegal package, bag or envelope on board and to search any person on board for any breach or violation of the
fishing with the use of dynamite. On the same day, the Fiscal filed an ex parte motion to hold the boats in customs and tariff laws. Herein, when the Philippine Navy, upon request of the Fisheries Commissioner,
custody as instruments and therefore evidence of the crime, and cabled the Fisheries Commissioner to apprehended on August 5 or 6, 1965 the fishing boats Tony Lex III and Tony Lex VI, otherwise known
detain the vessels. On October 2 and 4, likewise, the CFI of Palawan ordered the Philippine Navy to take respectively as Srta. Agnes and Srta. Winnie, these vessels were found to be without the necessary
the boats in custody. On 2 October 1965, the company filed a complaint with application for preliminary license in violation of Section 903 of the Tariff and Customs Code and therefore subject to seizure under
mandatory injunction (Civil Case 62799) with the CFI of Manila against the Commission and the Navy. Section 2210 of the same Code, and illegally fishing with explosives and without fishing license required
Among others, it was alleged that at the time of the seizure of the fishing boats in issue, the same were by Sections 17 and 18 of the Fisheries Law. Search and seizure without search warrant of vessels and
engaged in legitimate fishing operations off the coast of Palawan; that by virtue of the offer of compromise aircrafts for violations of the customs laws have been the traditional exception to the constitutional
dated 13 September 1965 by the company to the Secretary of Agriculture and Natural Resources, the requirement of a search warrant, because the vessel can be quickly moved out of the locality or
numerous violations of the Fishery Laws, if any, by the crew members of the vessels were settled. On 18 jurisdiction in which the search warrant must be sought before such warrant could be secured; hence it is
October 1965, Judge Francisco Arca issued an order granting the issuance of the writ of preliminary not practicable to require a search warrant before such search or seizure can be constitutionally effected.
mandatory injunction and issued the preliminary writ upon the filing by the company of a bond of The same exception should apply to seizures of fishing vessels breaching our fishery laws: They are
P5,000.00 for the release of the two vessels. On 19 October 1965, the Commission and the Navy filed a usually equipped with powerful motors that enable them to elude pursuing ships of the Philippine Navy or
motion for reconsideration of the order issuing the preliminary writ on 18 October 1965 on the ground, Coast Guard.
among others, that on 18 October 1965 the Philippine Navy received from the Palawan CFI two orders
dated October 2 and 4, 1965 requiring the Philippine Navy to hold the fishing boats in custody and PAPA v. MAGO
directing that the said vessels should not be released until further orders from the Court, and that the G.R. No. L-27360/February 28, 1968/EN BANCOriginal action in the SC for prohibition and
bond of P5,000.00 is grossly insufficient to cover the Government's losses in case the two vessels, which certiorari
are worth P495,000.00, are placed beyond the reach of the Government, thus frustrating their forfeiture , praying for the annulment of the order issued by respondent judgeParties:Petitioners:Ricardo G. Papa
as instruments of the crime. On 23 November 1965, Judge Arca denied the said motion for (Chief of Police of Manila), Juan Ponce Enrile (Commissioner of Customs), Pedro Pacis(Collector of
reconsideration. The Commission and the Navy filed a petition for certiorari and prohibition with Customs of the Port of Manila), Martin Alagao (Patrolman, head of counter-intelligence of theManila
preliminary injunction to restrain Judge Arca from enforcing his order dated 18 October 1965, and the writ Police Department)Respondents:Remedios MagoHilarion Jarencio (Presiding Judge of Br. 23, CFI of
of preliminary mandatory injunction thereunder issued. Manila)J. ZaldivarNovember 4, 1966 – having received information the day before that a certain shipment
Issue: of misdeclared andundervalued personal effects would be released from the customs zone of the port of
Whether the Fisheries Commissioner and the Navy can validly direct and/or effect the seizure of the Manila, Alagao and aduly deputized agent of the Bureau of Customs conducted surveillance of two trucks
vessels of the company for illegal fishing by the use of dynamite and without the requisite licenses. allegedly carrying thegoods. When the trucks left the customs zone, elements of the counter-intelligence
unit intercepted them inErmita. The trucks and the nine bales of goods they carried were seized on under Section 2530, pars. e and m, (1), (3), (4), and (5) of the Tariff and Customs Code. The Court had
instructions of the Chief of Police. Upon investigation those claiming ownership showed the policemen a held before (and did again in this case) that
“Statement of Receipts of Duties Collected in Informal Entry No. 147-5501” issued by the Bureau of merchandise, the importation of which is effected contrary to law, is subject to forfeiture,
Customs in the name of oneBienvenido Naguit.Mago filed with the CFI of Manila a Petition for
Mandamus and that goods released contrary to law are subject to seizureand forfeiture.
with restraining order or preliminary injunction, alleging that she was the owner of the goods seized,
which were purchased from Sta. MonicaGrocery in San Fernando, Pampanga. She hired the trucks Even if it be granted,
owned by Lanopa (who filed with her) to bringthe goods to her residence in Sampaloc, Manila. She arguendo
complained that the goods were seized without a warrant, and that they were not subject to seizure under , that after the goods in question had been brought out of the customs areathe Bureau of Customs had
Section 2531 of the Tariff and Customs Code even if they were misdeclared and undervalued because lost jurisdiction over the same, nevertheless, when said goods were interceptedat the Agrifina Circle on
she had bought them without knowing they had beenimported illegally. They asked that the police not November 4, 1966 by members of the Manila Police Department, acting underdirections and orders of
open the bales, the goods be returned, and for moral andexemplary damages.November 10, 1966 – their Chief, Ricardo C. Papa, who had been formally deputized by theCommissioner of Customs,
Judge issued an order restraining the police from opening the nine bales in question, but by then some
had already been opened. Five days later Mago filed an amended petition including asparty defendants the Bureau of Customs had regained jurisdiction and custody of the goods.
Pedro Pacis and Martin Alagao.December 23, 1966 – Mago filed a motion to release the goods, alleging Section 1206 of the Tariff and Customs Code imposes upon the Collector of Customs the duty to hold
that since the inventory ordered by the court of the goods seized did not show any article of prohibited possession of all imported articles upon which duties, taxes, and other charges havenot been paid or
importation, the same should be releasedupon her posting of the appropriate bond. The petitioners in the secured to be paid, and to dispose of the same according to law. The goodsin question, therefore, were
instant case filed their opposition, allegingthat the court had no jurisdiction over the case and thus no under the custody and at the disposal of the Bureau of Customsat the time the petition for
jurisdiction to order the release (case under jurisdiction of CTA), and as the goods were not declared they mandamus
were subject to forfeiture. was filed in the Court of First Instance of Manila onNovember 9, 1966. The Court of First Instance of
March 7, 1967 – assailed Order issued by Jarencio, authorized release under bond of goods seized and Manila, therefore, could not exercise jurisdiction over said goods even if the warrant of seizure and
held by petitioners in connection with the enforcement of the Tariff and Customs Code. The bond of detention of the goods for thepurposes of the seizure and forfeiture proceedings had not yet been issued
P40,000.00 was filed five days later. On the same day, Papa filed on his own behalf a motion for by the Collectorof Customs.
reconsideration on theground that the Manila Police Department had been directed by the Collector of The Court reiterated its ruling in
Customs to hold the goodspending termination of the seizure proceedings. Without waiting for the court’s De Joya v. Lantin:
action on the MR, petitioners filed the present action. Arguments of Petitioners (that seem important)(1) The owner of seized goods may set up defenses beforethe Commissioner of Customs during the
CFI had no jurisdiction over the case(2) Mago had no cause of action in the civil case filed with the CFI proceedings following seizure. From his decision appeal may bemade to the Court of Tax Appeals. To
due to her failure to exhaust alladministrative remedies before invoking judicial intervention Arguments of permit recourse to the Court of First Instance in cases of seizure of imported goods would in effect render
Respondents(1) It was within the jurisdiction of the lower court presided by respondent Judge to hear and ineffective the power of the Customs authorities under the Tariff andCustoms Code and deprive the Court
decide CivilCase No. 67496 and to issue the questioned order of March 7, 1967, because said Civil Case of Tax Appeals of one of its exclusive appellate jurisdictions.
No. 67496 wasinstituted long before seizure, and identification proceedings against the nine bales of Republic Acts 1937 and 1125 vest jurisdiction over seizure and forfeiture proceedings exclusively uponthe
goods in question were instituted by the Collector of Customs(2) Petitioners could no longer go after the Bureau of Customs and the Court of Tax Appeals. Such law being special in nature, whilethe Judiciary Act
goods in question after the corresponding duties and taxes had been paid and said goods had left the defining the jurisdiction of Courts of First Instance is a general legislation,not to mention that the former
customs premises and were no longer within the control of the Bureauof Customs are later enactments, the Court of First Instance should yieldto the jurisdiction of the Customs
IMPORANT ISSUE authorities.The Bureau of Customs acquires exclusive jurisdiction over imported goods, for thepurposes
(there’s another involving illegal search and seizure) of enforcement of the customs laws, from the moment the goods are actually in itspossession or control,
: even if no warrant of seizure or detention had previously been issued by the Collector of Customs in
WON the judge acted with jurisdiction in issuing the Order releasing the goods in question connection with seizure and forfeiture proceedings. In thepresent case, the Bureau of Customs actually
HELD: NO. seized the goods in question on November 4,1966, and so from that date the Bureau of Customs
Petition granted, case filed by Mago dismissed.The Bureau of Customs has the duties, powers and acquired jurisdiction over the goods forthe purposes of the enforcement of the tariff and customs laws, to
jurisdiction, among others, to(1) assess and collect all lawful revenues from imported articles, and all the exclusion of theregular courts. Much less then would the Court of First Instance of Manila have
other dues, fees, charges, fines andpenalties, accruing under the tariff and customs laws(2) prevent and jurisdictionover the goods in question after the Collector of Customs had issued the warrant of seizureand
suppress smuggling and other frauds upon the customs; and(3) to enforce tariff and customs laws.The detention on January 12, 1967.
goods in question were imported from Hongkong, as shown in the "Statement and Receipts of
DutiesCollected on Informal Entry". Not having acquired jurisdiction over the goods, itfollows that the Court of First Instance of Manila had no
As long as the importation has not been terminated the importedgoods remain under the jurisdiction of jurisdiction to issue the questionedorder of March 7, 1967 releasing said goods.
the Bureau of customs. Importation is deemedterminated only upon the payment of the duties, taxes and
other charges upon the articles,or secured to be paid, at the port of entry and the legal permit for People vs Malmstedt
withdrawal shall have beengranted. The payment of the duties, taxes, fees and other charges must be in
full.The record shows, by comparing the articles and duties stated in the aforesaid "Statementand Facts:Mikael Malmstedt, a Swedish national, was found, via a routine NARCOMinspection at Kilometer
Receipts of Duties Collected on Informal Entry" with the manifestation of the Office of the Solicitor 14, Acop, Tublay Mountain Province, carrying Hashish, aderivative of Marijuana. RTC La Trinidad found
General wherein it is stated that the estimated duties, taxes and other chargeson the goods subject of this him guilty for violation of theDangerous Drugs Act. The accused filed a petition to the Supreme Court for
case amounted to P95,772.00 the reversalof the decision arguing that the search and the arrest made was illegal because there wasno
as evidenced by the report of theappraiser of the Bureau of Customs, search warrant.
that the duties, taxes and other charges had not been paid infull. Furthermore, a comparison of the goods
on which duties had been assessed, as shown in the "Statement and Receipts of Duties Collected on Issue:Whether or not the decision of the trial court should be reversed (or affirmed) because the accused
Informal Entry" and the "compliance" itemizing the articlesfound in the bales upon examination and argues that the search and arrest was made without a warrant
inventory, shows that
the quantity of the goods wasunderdeclared, presumably to avoid the payment of duties thereon. Held:The RTC decision is affirmed.
(e.g. 40 pieces of ladies’sweaters assessed in the Statement when there actually
42 dozen Ratio:The constitution states that a peace officer or a private person may arrest a personwithout a
; 100 watch bands were assessed but warrant when in his presence the person to be arrested has committed, isactually committing, or is
2,209dozen, etc.) attempting to commit an offense. The offense was recognizedwith the warrantless search conducted by
The articles contained in the nine bales in question, were, therefore, subject to forfeiture NARCOM prompted by probable cause: (1) thereceipt of information by NARCOM that a Caucasian
coming from Sagada had prohibited drugs in his possession and (2) failure of the accused to immediately
presenthis passport. Valmonte vs. De Villa
Facts:
PEOPLE VS ARUTA On 20 January 1987, the National Capital Region District Command (NCRDC) wasactivated pursuant to
Letter of Instruction 02/87 of the Philippine General Headquarters,AFP, with the mission of conducting
In the morning of 13 Dec 1988, the law enforcement officers received information from an informant security operations within its area of responsibility andperipheral areas, for the purpose of establishing an
named “Benjie” that a certain “Aling Rosa” would be leaving for Baguio City on 14 Dec 1988 and would be effective territorial defense, maintainingpeace and order, and providing an atmosphere conducive to the
back in the afternoon of the same day carrying with her a large volume of marijuana; At 6:30 in the social, economic andpolitical development of the National Capital Region. As part of its duty to maintain
evening of 14 Dec 1988, Aruta alighted from a Victory Liner Bus carrying a travelling bag even as the peaceand order, the NCRDC installed checkpoints in various parts of Valenzuela, Metro
informant pointed her out to the law enforcement officers; NARCOM officers approached her and Manila.Petitioners aver that, because of the installation of said checkpoints, the residents of Valenzuela
introduced themselves as NARCOM agents; When asked by Lt. Abello about the contents of her are worried of being harassed and of their safety being placed at the arbitrary,capricious and whimsical
travelling bag, she gave the same to him; When they opened the same, they found dried marijuana disposition of the military manning the checkpoints, consideringthat their cars and vehicles are being
leaves; Aruta was then brought to the NARCOM office for investigation. subjected to regular searches and check-ups, especiallyat night or at dawn, without the benefit of a
search warrant and/or court order. Their allegedfear for their safety increased when, at dawn of 9 July
ISSUE: Whether or not the conducted search and seizure is constitutional. 1988, Benjamin Parpon, a supplyofficer of the Municipality of Valenzuela, Bulacan, was gunned down
allegedly in cold bloodby the members of the NCRDC manning the checkpoint along McArthur Highway
HELD: The SC ruled in favor of Aruta and has noted that some drug traffickers are being freed due to at Malinta,Valenzuela, for ignoring and/or refusing to submit himself to the checkpoint and forcontinuing to
technicalities. Aruta cannot be said to be committing a crime. Neither was she about to commit one nor speed off inspire of warning shots fired in the air.
had she just committed a crime. Aruta was merely crossing the street and was not acting in any manner
that would engender a reasonable ground for the NARCOM agents to suspect and conclude that she was Issue:
committing a crime. It was only when the informant pointed to Aruta and identified her to the agents as the WON the installation of checkpoints violates the right of the people againstunreasonable searches and
carrier of the marijuana that she was singled out as the suspect. The NARCOM agents would not have seizures
apprehended Aruta were it not for the furtive finger of the informant because, as clearly illustrated by the
evidence on record, there was no reason whatsoever for them to suspect that accused-appellant was Held:
committing a crime, except for the pointing finger of the informant. The SC could neither sanction nor
tolerate as it is a clear violation of the constitutional guarantee against unreasonable search and seizure. Petitioner's concern for their safety and apprehension at being harassed by themilitary manning the
Neither was there any semblance of any compliance with the rigid requirements of probable cause and checkpoints are not sufficient grounds to declare the checkpoints per se
warrantless arrests. Consequently, there was no legal basis for the NARCOM agents to effect a , illegal. No proof has been presented before the Court to show that, in the course of theirroutine checks,
warrantless search of Aruta’s bag, there being no probable cause and the accused-appellant not having the military, indeed, committed specific violations of petitioners'' rightsagainst unlawful search and seizure
been lawfully arrested. Stated otherwise, the arrest being incipiently illegal, it logically follows that the of other rights. The constitutional right againstunreasonable searches and seizures is a personal right
subsequent search was similarly illegal, it being not incidental to a lawful arrest. The constitutional invocable only by those whose rightshave been infringed, or threatened to be infringed. Not all searches
guarantee against unreasonable search and seizure must perforce operate in favor of accused-appellant. and seizures areprohibited. Those which are reasonable are not forbidden. The setting up of the
As such, the articles seized could not be used as evidence against accused-appellant for these are “fruits questionedcheckpoints may be considered as a security measure to enable the NCRDC to pursue
of a poisoned tree” and, therefore, must be rejected, pursuant to Article III, Sec. 3(2) of the Constitution. itsmission of establishing effective territorial defense and maintaining peace and order for thebenefit of
the public. Checkpoints may not also be regarded as measures to thwart plots todestabilize the govt, in
NOTES: the interest of public security. Between the inherent right of thestate to protect its existence and promote
public welfare and an individuals right against awarrantless search w/c is, however, reasonably
When is a warrantless search allowed? conducted, the former should prevail. True,the manning of checkpoints by the military is susceptible of
abuse by the military in thesame manner that all governmental power is susceptible of abuse. But, at the
1. Warrantless search incidental to a lawful arrest recognized under Section 12, Rule 126 of the Rules of cost of occasional inconvenience, discomfort and even irritation to the citizen, the checkpointsduring
Court 8 and by prevailing jurisprudence; these abnormal times, when conducted w/in reasonable limits, are part of the pricewe pay for an orderly
society and a peaceful community.
2. Seizure of evidence in “plain view,” the elements of which are:
Pp. vs. Salvatierra
(a) a prior valid intrusion based on the valid warrantless arrest in which the police are legally present in ‘Facts:
the pursuit of their official duties; While Charlie Fernandez was walking towards Quiapo, appellant and the 2 accused lunged a
pointedinstrument at Charlie, hitting the latter at the left breast. This resulted to his death.On Nov. 15,
(b) the evidence was inadvertently discovered by the police who had the right to be where they are; 1990 the police received a complaint that appellant was creating a commotion. He wasthereafter taken in
custody.Appellant put up the defense of alibi alleging that he was having merienda with his wife and
(c) the evidence must be immediately apparent, and children whenthe incident occurred. He also alleged that he had an altercation with a woman on that day
who caused his arrestfor the crime of malicious mischief, wherein he was detained after. Then, when
(d) “plain view” justified mere seizure of evidence without further search; police arrived, they brought him tothe Homicide Section where he was investigated for the stabbing of
Fernandez.Appellant claimed that the arrest was made almost 3 months after the commission of the
3. Search of a moving vehicle. Highly regulated by the government, the vehicle’s inherent mobility crime and nowarrant had been obtained during the 3-month intervening period between the commission
reduces expectation of privacy especially when its transit in public thoroughfares furnishes a highly of the crime and hisapprehension, thus making the arrest illegal.
reasonable suspicion amounting to probable cause that the occupant committed a criminal activity; Issue:
W/N the arrest is violative of his constitutional rights?
4. Consented warrantless search; Held:
No. Appellant is estopped from questioning the legality of his arrest considering that he never raised
5. Customs search; itbefore entering his plea. Any objection involving a warrant of arrest or the procedure in the acquisition of
jurisdiction over the person of an accused must be made before he enters his plea, otherwise, the
6. Stop and Frisk; and objection isdeemed waived.

7. Exigent and Emergency Circumstances. Umil v. Ramos, 202 SCRA 251 (1991)
At the same time Oplan Hackle I was discovered, which constitutes plans of bombings and attacks on
Facts: PMA Alumni Homecoming in Baguio, the same event where the President was invited. The next morning
after the alumni homecoming celebration, a bomb was found inside the campus.
Separate motions before the Court, seeking reconsideration.
PNP Chief Arturo Lomibao also intercepted information that PNP- SAF members are planning to defect
In the Umil case, the arresting officers had good reason to believe that an NPA member (Rolando Dural, from the administration, while on the same view Congressman Peping Cojuanco plotted moves to bring
although using a fictitious name) was indeed being treated at St. Agnes Hospital, QC for gunshot wounds. down the Arroyo Administration.
The information was from the attending doctor and hospital management, and therefore came from
reliable sources. Huge number of soldiers joined the rallies to provide critical mass and armed component to Anti- Arroyo
protests.
In the case of Wilfredo Buenaobra, the same admitted that he was an NPA courier.
Bombings of telephone communication towers and cell sites in Bulacaan and Bataan was also considered
In the case of Amelia Roque, subversive documents and live ammunition were found at the time of her as an additional factual basis after the issuance of PP 1017 and GO 5.
arrest, and she admitted to owning such documents.
Because of these incidental series of events which clearly presents a critical situation, President Arroyo
As regards Domingo Anonuevo & Ramon Casiple, agents frisked them and found subversive documents cancelled all activities related to EDSA People Power I. Mike Arroyo, then Executive Secretary,
& loaded guns without permits. announced that warrantless arrest and takeover of facilities can be implemented.

With regard toVicky Ocaya, she arrived at a house subject to a search warrant. Ammunition & subversive Succeeding this announcement was the arrest of Randy David, a Filipino journalist and UP professor due
documents were found in her car. to a mistake of fact that he was actually involved in the street rallies. Seizure of Daily Tribune, Malaya
and Abante-- all local news publication, took place which, according to the PNP, was meant to show a
In the Nazareno case, Narciso Nazareno was identified by Ramil Regala as the latter’s strong presence to tell the media outlets not to connive or do anything that would help rebels in bringing
companion in killing Romulo Bunye II. down the government. Police also arrested Congressman Crispin Beltran, who then represented the
Issue: Anakpawis Party.
Whether or not Rolando Dural (and other petitioners in the other consolidated cases) was lawfully
arrested
Issue:
Ruling: Whether or not the issuance of Presidential Proclamation PP 1017 is unconstitutional? Whether or not the
Dural and the other petitioners were lawfully arrested for being members of the New People’s arrest of Randy David and the seizure of Daily Tribune et. al., is unconstitutional?
Army (mere membership is penalized), and for subversion (a continuing offense).
Ruling of the court:
Subversion and rebellion are anchored on an ideological base which compels the repetition of the same
acts of lawlessness & violence until the objective of overthrowing organized government is attained. Respondents claim that such petition is moot and academic based on the issuance of PP 1017, but the
Likewise, the arresting officers had personal knowledge of facts indicating that the person to be arrested Court rejects such contention. A moot and academic case is one that ceases to present a justiciable
is the one who committed the offense (based on actual facts), coupled with good faith in making the controversy. In this case, the Court is convinced that the President was justified in issuing PP 1017 which
arrest. calls for military aid.
The Court reiterates that mere suspicion of being a Communist Party member or a subversive is
absolutely not a ground for the arrest without warrant of the suspect. The Court predicated the validity of Most people then equate it to martial law, but such case is different wherein the basis then was the 1973
the arrests on the compliance with the requirements of a long existing law; probable cause and good faith Constitution. Under the present 1987 Constitution, the President may summon armed forces to aid him in
of the arresting peace officers; and that the arrest be on the basis of supporting lawless violence.
actual facts and circumstances.
The President's declaration of state rebellion was merely an act declaring a status or conduction of a
David vs. Arroyo G.R. No. 171396 May 3, 2006 public moment of interest. State of national emergency, however, is the prerogative of the President. Her
Facts of the case: exercise of emergency powers such as the taking over of privately owned utility requires delegation from
the Congress, which is entirely different from the martial law.
During the celebration of People Power I, President Arroyo issued Presidential Proclamation 1017 (PP
1017 for brevity) declaring a state of national emergency. The President also issued General Order As to the seizure of the Daily Tribune and the arrest of Randy David, the Court considers those actions
(G.O.) No. 5 implementing PP 1017. unlawful based on the fact that it violates the constitutional mandate of freedom of expression.

The President stated that over the past months, elements in political opposition have conspired with Pollo v. Constantino-David, G.R. No. 181881, 18 October 2011
extreme left represented by NDF- CCP- NPA and military adventurists, which caused her to declare such Facts
order. The President considered aims to oust the President and take- over reigns of government as clear
and present danger. Respondent CSC Chair Constantino-David received an anonymous letter complaint alleging of an
anomaly taking place in the Regional Office of the CSC. The respondent then formed a team and issued
On March 3, President Arroyo lifted PP 1017. a memo directing the team “to back up all the files in the computers found in the Mamamayan Muna
(PALD) and Legal divisions.”
Solicitor General argued that the basis of declaring PP 1017 was that the intent of the Constitution is to
give full discretionary powers to the President in determining the necessity of calling out the AFP. Several diskettes containing the back-up files sourced from the hard disk of PALD and LSD computers
were turned over to Chairperson David. The contents of the diskettes were examined by the CSC’s Office
However despite the contentions of the Solicitor General, the Magdalo group indicted the Oakwood for Legal Affairs (OLA). It was found that most of the files in the 17 diskettes containing files copied from
mutiny and called to wear red bands on their left arms to show disgust. the computer assigned to and being used by the petitioner, numbering about 40 to 42 documents, were
draft pleadings or lettersin connection with administrative cases in the CSC and other tribunals. On the
basis of this finding, Chairperson David issued the Show-Cause Order, requiring the petitioner, who had
gone on extended leave, to submit his explanation or counter-affidavit within five days from notice.
were retrieved is a government-issued computer, hence government property the use of which the CSC
In his Comment, petitioner denied the accusations against him and accused the CSC Officials of “fishing has absolute right to regulate and monitor.
expedition” when they unlawfully copied and printed personal files in his computer.
O'CONNOR v. ORTEGA, 480 U.S. 709 (1987)
He was charged of violating R.A. No. 6713 (Code of Conduct and Ethical Standards for Public Officials
and Employees). He assailed the formal charge and filed an Omnibus Motion ((For Reconsideration, to Facts of the Case
Dismiss and/or to Defer) assailing the formal charge as without basis having proceeded from an illegal In 1981, officials at a hospital, including Executive Director Dr. Dennis O'Connor, suspected improprieties
search which is beyond the authority of the CSC Chairman, such power pertaining solely to the court. in Dr. Ortega's management of a residency program. The officials conducted an investigation of Ortega,
which included multiple searches of his office and seizure of a number of items. The items were later
The CSC denied the omnibus motion and treated the motion as the petitioner’s answer to the charge. In used in proceedings before the California State Personnel Board to impeach the credibility of witnesses
view of the absence of petitioner and his counsel, and upon the motion of the prosecution, petitioner was that testified on Dr. Ortega's behalf.
deemed to have waived his right to the formal investigation which then proceeded ex parte.
Question
The petitioner was dismissed from service. He filed a petition to the CA which was dismissed by the latter Did the supervisor's search of the office violate Dr. Ortega's "reasonable expectation of privacy"
on the ground that it found no grave abuse of discretion on the part of the respondents. He filed a motion guaranteed by the Fourth Amendment?
for reconsideration which was further denied by the appellate court. Hence, this petition.
Issue Conclusion
WON the search conducted by the CSC on the computer of the petitioner constituted an illegal search Decision: 5 votes for O'Connor, 4 vote(s) against
and was a violation of his constitutional right to privacy Legal provision: Amendment 4: Fourth Amendment
In a 5-to-4 decision, the Court held that the search did not violate the Fourth Amendment. The Court held
Ruling that "the realities of the workplace" made some expectations of privacy among public employees
The search conducted on his office computer and the copying of his personal files was lawful and did not unreasonable when the intrusion was by a supervisor rather than a law enforcement official. Work- related
violate his constitutional right. searches, the Court found, were "merely incident to the primary business of the agency," and a warrant
requirement would "seriously disrupt the routine conduct of business." The Court thus held that a
Ratio Decidendi standard of "reasonableness" was sufficient for work-related intrusions by public employers.

In this case, the Court had the chance to present the cases illustrative of the issue raised by the Griswold v. Connecticut
petitioner.
Brief Fact Summary. Appellants were charged with violating a statute preventing the distribution of advice
Katz v. United States 389 U.S. 437 (1967), the US Supreme Court held that the act of FBI agents in to married couples regarding the prevention of conception. Appellants claimed that the statute violated the
electronically recording a conversation made by petitioner in an enclosed public telephone booth violated 14th Amendment to the United States Constitution.
his right to privacy and constituted a “search and seizure”. Because the petitioner had a reasonable
expectation of privacy in using the enclosed booth to make a personal telephone call, the protection of the Synopsis of Rule of Law. The right of a married couple to privacy is protected by the Constitution.
Fourth Amendment extends to such area. Moreso, the concurring opinion of Mr. Justice Harlan noted that
the existence of privacy right under prior decisions involved a two-fold requirement: first, that a person Facts. Appellant Griswold, Executive Director of the Planned Parenthood League of Connecticut and
has exhibited an actual (subjective) expectation of privacy; and second, that the expectation be one that Appellant Buxton, a licensed physician who served as Medical Director for the League at its Center in
society is prepared to recognize as reasonable (objective). New Haven, were arrested and charged with giving information, instruction, and medical advice to
married persons on means of preventing conception. Appellants were found guilty as accessories and
Mancusi v. DeForte 392 U.S. 364, 88 S.Ct. 2120, 20 L.Ed2d 1154 (1968),thus “recognized that fined $100 each. Appellants appealed on the theory that the accessory statute as applied violated the
employees may have a reasonable expectation of privacy against intrusions by police.” 14th Amendment to the United States Constitution. Appellants claimed standing based on their
professional relationship with the married people they advised.
O’Connor v. Ortega 480 U.S. 709 (1987), the Court categorically declared that “[i]ndividuals do not lose
Fourth Amendment rights merely because they work for the government instead of a private employer.” In Issue. Does the Constitution provide for a privacy right for married couples?
O’Connor the Court recognized that “special needs” authorize warrantless searches involving public
employees for work-related reasons. The Court thus laid down a balancing test under which government Held. The First Amendment has a penumbra where privacy is protected from governmental intrusion,
interests are weighed against the employee’s reasonable expectation of privacy. This reasonableness test which although not expressly included in the Amendment, is necessary to make the express guarantees
implicates neither probable cause nor the warrant requirement, which are related to law enforcement. meaningful. The association of marriage is a privacy right older than the Bill of Rights, and the State’s
effort to control marital activities in this case is unnecessarily broad and therefore impinges on protected
Social Justice Society (SJS) v. Dangerous Drugs Board G.R. Nos. 157870, 158633 and 161658, Constitutional freedoms.
November 3, 2008, 570 SCRA 410, 427, (citing Ople v. Torres, G.R. No. 127685, July 23, 1998, 293
SCRA 141, 169), recognized the fact that there may be such legitimate intrusion of privacy in the Dissent. Justice Stewart and Justice Black. Although the law is silly, it is not unconstitutional. The citizens
workplace. of Connecticut should use their rights under the 9th and 10th Amendment to convince their elected
representatives to repeal it if the law does not conform to their community standards.
The Court ruled that the petitioner did not have a reasonable expectation of privacy in his office and
computer files. Concurrence. Justice Goldberg, the Chief Justice, and Justice Brennan. The right to privacy in marriage is
so basic and fundamental that to allow it to be infringed because it is not specifically addressed in the first
As to the second point of inquiry, the Court answered in the affirmative. The search authorized by the eight amendments is to give the 9th Amendment no effect.
CSC Chair, the copying of the contents of the hard drive on petitioner’s computer reasonable in its Justice Harlan. The relevant statute violates the Due Process Clause of the 14th Amendment because if
inception and scope. violates the basic values implicit in the concept of ordered liberty.

The Court noted that unlike in the case of Anonymous Letter-Complaint against Atty. Miguel Morales, Discussion. The right to privacy in marriage is not specifically protected in either the Bill of Rights or the
Clerk of Court, Metropolitan Trial Court of Manila A.M. Nos. P-08-2519 and P-08-2520, November 19, Constitution. Nonetheless, it is a right so firmly rooted in tradition that its protection is mandated by
2008, 571 SCRA 361, the case at bar involves the computer from which the personal files of the petitioner various Constitutional Amendments, including the 1st, 9th and 14th Amendments.
MORFE V MUTUC character. This is not to say that a public officer, by virtue of position he holds, is bereft of constitutional
Facts: protection; it is only to emphasize that in subjecting him to such a further compulsory revelation of his
assets and liabilities, including the statement of the amounts of personal and family expenses, and the
The Law: Anti-Graft and Corrupt Practices Act of 1960 (RA No. 3019) amount of income taxes paid for the next preceding calendar year, there is no unconstitutional intrusion
Every public officer within 30 days after its approval or after his assumption of office “and within the month into what otherwise would be a private sphere.”
of January of every year thereafter”, as well as upon termination of his position, shall prepare and file with Unreasonable Search and Seizure
the head of the office to which he belongs, “a true detailed and sworn statement of assets and liabilities, The constitutional guarantee against unreasonable search and seizure does not give freedom from
including a statement of the amounts and sources of his income, the amounts of his personal and family testimonial compulsion.
expenses and the amount of income taxes paid for the next preceding calendar year”. Right against self-incrimination
Plaintiff Morfe, a judge of a CFI, contends that the periodical submission “within the month of January of We are not aware of any constitutional provision designed to protect a man’s conduct from judicial inquiry,
every other year thereafter” of their sworn statement of assets and liabilities (SAL) is violative of due or aid him in fleeing from justice.
process as an oppressive exercise of police power and as an unlawful invasion of the constitutional right Insult to personal integrity and official dignity
to privacy implicit on the ban against unreasonable search and seizure construed together with the Only congressional power or competence, not the wisdom of the action taken, mey be the basis for
prohibition against self-incrimination. declaring a statute invalid.
Executive Secretary and DOJ Sec:
Acceptance of public position = voluntary assumption of obligation OPLE VS TORRES
Merely seeks to adopt a reasonable measure of insuring the interest of general welfare in honest and [G.R. No. 127685. July 23, 1998]293 SCRA 141FACTS: Petitioner Senator Blas F. Ople assailed the
clean public service and is therefore a legitimate exercise of police power. constitutionality of the
CFI of Pangasinan held that the requirement exceeds the permissible limit of the police power and is thus Administrative Order No. 308 entitled “Adoption of Computerized IdentificationReference System” on the
offensive to the due process clause following grounds:
1.) The administrative order issued by the executive is deemed to be a law and not amere administrative
Issue/s: order thus it is a usurpation of legislative power of the congress tomake laws, and
2.) It impermissibly intrudes the citizen‟s constitutional right of privacy.
Whether the periodical submission of SAL for public officers is: 1. An oppressive exercise of police power; ISSUE: Does the Administrative Order No. 308 violates the constitutional right toprivacy?HELD: Yes, the
2. Violative of due process and an unlawful invasion of the right to privacy implicit in the ban against Administrative Order violates the constitutional right to privacy because
unreasonable search and seizure construed together with the prohibition against self-incrimination; 3. An its scope is too broad and vague that will put people‟s right to privacy in clear and
insult to the personal integrity and official dignity of public officials. present danger if implemented. The A.O. 308 also lacks of proper safeguards for protecting the
information that will be gathered from people through biometrics and
Ruling: Decision reversed. other means. Thus, A.O. No. 308 may interfere with the individual‟s liberty of abode and
travel by enabling authorities to track down his movement; it may also enableunscrupulous persons to
Ratio: access confidential information and circumvent the rightagainst self-
incrimination; it may pave the way for “fishing expeditions” by government
Presumption of validity authorities and evade the right against unreasonable searches and seizures.
Plaintiff asserted that the submission of SAL was a reasonable requirement for employment so a public
officer can make of record his assets and liabilities upon assumption of office. Plaintiff did not present
evidence to rebut the presumption of validity. Gaanan v. IAC, 145 SCRA 112 (1986)Gaanan v. IAC, 145 SCRA 112 (1986)
F: Complainant Atty. Pintor and Montebon offered to withdraw the complaint for direct assaultthey filed
“If the liberty involved were freedom of the mind or the person, the standard for the validity of against Laconico after demanding P8,000 from him. This demand was heard by Atty. Gaanan through a
governmental acts is much more rigorous and exacting, but where the liberty curtailed affects the most telephone extensionas requested by Laconico so as to personally hear the proposed conditions for the
rights of property, the permissible scope of regulatory measure is wider.” (Ermita-Malate Hotel v. Mayor of settlement. Atty. Pintor was subsequently arrested inan entrapment operation upon receipt of the money.
Manila) Since Atty. Gaanan listened to the telephone conversation without complainant''sconsent, complainant
Exercise of Police power and the defense provided by the Due Process Clause charged Gaanan and Laconico with violation of the Anti- Wiretapping Act (RA 4200).
“inherent and plenary power in the state which enables it to prohibit all things hurtful to the comfort, safety ISSUE: W/N an extension telephone is among the prohibited devices in Sec. 1 of RA 4200 such that iuts
and welfare of society” (Justice Malcolm) use to overhear a privateconversation would constitute an unlawful interception of communication
The power of sovereignty, the power to govern men and things within the limits of its domain (Justice between 2 parties using a telephone line.
Taney, going beyond curtailment of rights) HELD: NO An extension tel. cannot be placed in the same category as a dictaphone, dictagraph, or
Anyone with an alleged grievance regarding the extension of police power to regulatory action affecting other devices enumerated in Sec. 1of the law as the use thereof cannot be considered as "tapping" the
persons in public or private life can invoke the protection of due process. wire or cable of a telephone line. This section refers to instrumentswhose installation or presence cannot
It has been held that due process may be relied upon by public official to protect the security of tenure be presumed by the party or parties being overheard because, by their very nature, they are notof
which in a limited sense is analogous to property. Therefore he could also use due process to strike down common usage and their purpose is precisely for tapping, intercepting, or recording a tel. conversation.
what he considers as an infringement of his liberty. The tel. extension in this casewas not installed for that purpose. It just happened to be there for ordinary
Under the Constitution, the challenged provision is allowable as long as due process is observed. office use. Furthermore, it is a general rule that penalstatutes must be construed strictly in favor of the
The standard for due process is REASONABLENESS. Test: Official action must not outrun the bounds of accused. Thus in the case of doubt as in this case, on WON an extension tel. isincluded in the phrase
reason and result in sheer oppression. "device or arrangement" the penal statute must be construed as not including an extension tel. A perusal
“It would be to dwell in the realm of abstractions and to ignore the harsh and compelling realities of public of theSenate Congressional Record shows that our lawmakers intended to discourage, through
service with its ever-present temptation to heed the call of greed and avarice to condemn as arbitrary and punishment, persons such as governmentauthorities or representatives of organized groups from
oppressive a requirement as that imposed upon public officials and employees to file such sworn installing devices in order to gather evidence for use in court or to intimidate,blackmail or gain some
statement of assets and liabilities every two years after having done so upon assuming office…There was unwarranted advantage over the tel. users. Consequently, the mere act of listening , in order to
therefore no unconstitutional exercise of police power.” be punishablemust strictly be with the use of the enumerated devices in RA 4200 or others of similar
Right to privacy nature.
Right to be let alone
“It cannot be said that the challenged statutory provision calls for disclosure of information which infringes Olmstead v. United States
on the right of a person to privacy. It cannot be denied that the rational relationship such a requirement Brief Fact Summary. The conversations of various individuals involved in illegal liquor sales were tapped.
possesses with the objective of a valid statute goes very far in precluding assent to an objection of such
Synopsis of Rule of Law. “A standard which would forbid the reception of evidence, if obtained by other . [The Court has] to choose, and for my part I think it a less evil that some criminals should escape than
than nice ethical conduct by government officials, would make society suffer and give criminals greater that the government should play an ignoble part.”
immunity than has been known heretofore. In the absence of controlling legislation by Congress, those
who realize the difficulties in bringing offenders to justice may well deem it wise that the exclusion of Dissent. Justice Brandeis filed a dissenting opinion and argued “[a]pplying to the Fourth and Fifth
evidence should be confined to cases where rights under the Constitution would be violated by admitting Amendments the established rule of construction, the defendants’ objections to the evidence obtained by
it.” wire tapping must, in my opinion, be sustained. It is, of course, immaterial where the physical connection
with the telephone wires leading into the defendants’ premises was made. And it is also immaterial that
Facts. Various individuals were convicted of liquor related crimes, including conspiracy. The operation the intrusion was in aid of law enforcement. Experience should teach us to be most on our guard to
grossed a substantial amount of money. The leading conspirator and the general manager of the protect liberty when the government’s purposes are beneficent. Men born to freedom are naturally alert to
business was one of the Petitioners, Olmstead (the “Petitioner”). The main office of the business was in repel invasion of their liberty by evil-minded rulers. The greatest dangers to liberty lurk in insidious
Seattle and there were three telephones in the office, each on a different line. There were also telephones encroachment by men of zeal, well-meaning but without understanding.”
in an office the Petitioner had in his own home, at the home of his associates and various other places in “When the government, having full knowledge, sought, through the Department of Justice, to avail itself of
Seattle. A lot of communication occurred between Seattle and Vancouver, British Columbia. the fruits of these acts in order to accomplish its own ends, it assumed moral responsibility for the officers’
“The information which led to the discovery of the conspiracy and its nature and extent was largely crimes and if this court should permit the government, by means of its officers’ crimes, to effect its
obtained by intercepting messages on the telephones of the conspirators by four federal prohibition purpose of punishing the defendants, there would seem to be present all the elements of a ratification. If
officers. Small wires were inserted along the ordinary telephone wires from the residences of four of the so, the government itself would become a lawbreaker.”
[suspects] and those leading from the chief office. The insertions were made without trespass upon any
property of the defendants. They were made in the basement of the large office building. The taps from
house lines were made in the streets near the houses.”
Various conversations were taped and testified to by government witnesses.

Issue. “[W]hether the use of evidence of private telephone conversations between the defendants and
others, intercepted by means of wire tapping, amounted to a violation of the Fourth and Fifth Goldman v. United States, 316 U.S. 129 (1942)
Amendments[?]” No. 962
Held. “The [Fourth] amendment does not forbid what was done here. There was no searching. There was
no seizure. The evidence was secured by the use of the sense of hearing and that only. There was no Argued February 5, 6, 1942
entry of the houses or offices of the defendants. By the invention of the telephone 50 years ago, and its
application for the purpose of extending communications, one can talk with another at a far distant place. Decided April 27, 1942
The language of the amendment cannot be extended and expanded to include telephone wires, reaching
to the whole world from the defendant’s house or office. The intervening wires are not part of his house or 316 U.S. 129
office, any more than are the highways along which they are stretched.”
“Congress may, of course, protect the secrecy of telephone messages by making them, when Syllabus
intercepted, inadmissible in evidence in federal criminal trials, by direct legislation, and thus depart from
the common law of evidence. But the courts may not adopt such a policy by attributing an enlarged and 1. Refusal of the judge in the trial of a criminal case in the federal court to allow defendant to inspect the
unusual meaning to the Fourth Amendment. The reasonable view is that one who installs in his house a memoranda of Government witnesses -- where the memoranda were not used by the witnesses in court,
telephone instrument with connecting wires intends to project his voice to those quite outside, and that but only to refresh their recollection prior to testifying, and were also part of the Government's files -- held
the wires beyond his house, and messages while passing over them, are not within the protection of the not an abuse of discretion. P. 316 U. S. 132.
Fourth Amendment. Here those who intercepted the projected voices were not in the house of either party
to the conversation.” 2. Divulgence of a person's telephone conversation, overheard as it was spoken into the telephone
“Neither the cases we have cited nor any of the many federal decisions brought to our attention hold the receiver, does not violate § 605 of the Federal Communications Act, as in such case there is neither a
Fourth Amendment to have been violated as against a defendant, unless there has been an official "communication" nor an "interception" within the meaning of the Act. P. 316 U. S. 133.
search and seizure of his person or such a seizure of his papers or his tangible material effects or an
actual physical invasion of his house ‘or curtilage’ for the purpose of making a seizure. [The court 3. Evidence obtained by federal agents by use of a detectaphone, applied to the wall of a room adjoining
thought], therefore, that the wire tapping here disclosed did not amount to a search or seizure within the the office of the defendant, held not unlawfully obtained as a consequence of a prior trespass committed
meaning of the Fourth Amendment.” by the agents in the defendant's office where such trespass, as found by the courts below, did not aid
Additionally, “[t]he common-law rule is that the admissibility of evidence is not affected by the illegality of materially in the use of the detectaphone. P. 316 U. S. 134.
the means by which it was obtained.”
“A standard which would forbid the reception of evidence, if obtained by other than nice ethical conduct by 4. The use by federal agents of a detectaphone, whereby conversations in the office of a defendant were
government officials, would make society suffer and give criminals greater immunity than has been known overheard through contact on the
heretofore. In the absence of controlling legislation by Congress, those who realize the difficulties in
bringing offenders to justice may well deem it wise that the exclusion of evidence should be confined to Page 316 U. S. 130
cases where rights under the Constitution would be violated by admitting it.”
wall of an adjoining room, did not violate the Fourth Amendment, and evidence thus obtained was
Dissent. Justice Holmes filed a dissenting opinion and argued “that apart from the Constitution the admissible in a federal court. P. 316 U. S. 135.
government ought not to use evidence obtained and only obtainable by a criminal act. There is no body of
precedents by which we are bound, and which confines us to logical deduction from established rules. 118 F.2d 310 affirmed.
Therefore [the Court] must consider the two objects of desire both of which [the Court] cannot have and
make up our minds which to choose. It is desirable that criminals should be detected, and to that end that Certiorari, 314 U.S. 701, to review the affirmance of convictions of conspiracy to violate the Bankruptcy
all available evidence should be used. It also is desirable that the government should not itself foster and Act.
pay for other crimes, when they are the means by which the evidence is to be obtained. If it pays its
officers for having got evidence by crime I do not see why it may not as well pay them for getting it in the Official Supreme Court case law is only found in the print version of the United States Reports. Justia
same way, and I can attach no importance to protestations of disapproval if it knowingly accepts and pays case law is provided for general informational purposes only, and may not reflect current legal
and announces that in future it will pay for the fruits developments, verdicts or settlements. We make no warranties or guarantees about the accuracy,
completeness, or adequacy of the information contained on this site or information linked to from this site.
Please check official sources.

Katz v. United States


Brief Fact Summary. The petitioner, Katz (the “petitioner”), was convicted of transmitting wagering
information over telephone lines in violation of federal law. The government had entered into evidence the
petitioner’s end of telephone conversations that the government had obtained by placing a listening
device to the phone booth that the petitioner used. The Court of Appeals rejected the petitioner’s
contention that the evidence should be suppressed.

Synopsis of Rule of Law. The protection of the Fourth Amendment of the United States Constitution
(”Constitution”), against unreasonable searches and seizures, follows the person and not the place.

Facts. The petitioner used a public telephone booth to transmit wagering information from Los Angeles to
Boston and Miami in violation of federal law. After extensive surveillance, the FBI placed a listening
device to the top of the telephone booth and recorded the petitioner’s end of the telephone conversations
which was then used as evidence against him at his trial. The petitioner moved to have the evidence
suppressed under the Fourth Amendment of the Constitution, and that motion was denied. The Court of
Appeals rejected the contention that the evidence is inadmissible. Certiorari was granted.

Issue. Whether the Fourth Amendment of the Constitution protects telephone conversations conducted in
a phone booth and secretly recorded from introduction as evidence against a person?
Held. Justice Potter Stewart filed the majority opinion. The petitioner strenuously asserted that the phone
booth was a constitutionally protected area. However, the Fourth Amendment protects persons and not
places from unreasonable intrusion. Even in a public place, a person may have a reasonable expectation
of privacy in his person. Although the petitioner did not seek to hide his self from public view when he
entered the telephone booth, he did seek to keep out the uninvited ear. He did not relinquish his right to
do so simply because he went to a place where he could be seen. A person who enters into a telephone
booth may expect the protection of the Fourth Amendment of the Constitution as he assumes that the
words he utters into the telephone will not be broadcast to the world. Once this is acknowledged, it is
clear that the Fourth Amendment of the Constitution protects persons and not areas from unreasonable
searches and seizures. The Government’s activities in electron
ically listening to and recording the petitioner’s telephone conversations constituted a search and seizure
under the Fourth Amendment and absent a search warrant predicated upon sufficient probable cause, all
evidence obtained is inadmissible.

Dissent. Justice Hugo Black (”J. Black”) filed a dissenting opinion. J. Black observed that eavesdropping
was an ancient practice that the Framers were certainly aware of when they drafted the United States
Constitution (”Constitution”). Had they wished to prohibit this activity under the Fourth Amendment of the
Constitution they would have added such language that would have effectively done so. By clever
wording, the Supreme Court finds it plausible to argue that language aimed specifically at searches and
seizures of things that can be searched and seized may, to protect privacy, be applied to eavesdropped
evidence of conversations.
Concurrence. Justice John Harlan (”J. Harlan”) filed a dissenting opinion. The Fourth Amendment of the
Constitution protects persons, not places. There is a twofold requirement for what protection is afforded to
those people. First, that a person has exhibited an actual expectation of privacy and, second, that the
expectation be one that society is prepared to recognize as reasonable. The critical fact in this case is
that a person who enters a telephone booth shuts the door behind him, pays the toll, and is surely entitled
to assume that his conversation is not being intercepted. On the other hand, conversations out in the
open public would not be protected against being overheard as the expectation of privacy would not be
reasonable.

Discussion. The Fourth Amendment of the Constitution provides constitutional protection to individuals
and not to particular places. The two-part test for this protection is introduced by J. Harlan. First, the
person must have exhibited an actual expectation of privacy and, second, that expectation must be
reasonable.

Вам также может понравиться